SlideShare una empresa de Scribd logo
1 de 220
Descargar para leer sin conexión
March 2010 MRCOG part 1 exam
MCQ, EMQ, Separate items to revise
• large scale study was carried out to assess the benefits of a new drug
which would prevent fractured neck of femur due to osteoporosois in
post menopausal women. the study was carried out correctly using a
treatment group and an appropriate control group. the number of
women who sustained a fracture neck of femur due to osteoporosis
was measured reliably in each group and result are
group with fracture without fracture
control n1 n3
study n2 n4
calculate the number of women who need to recieve new drug in order
to prevent one new fracture of femoral neck due to postmenopausal
osteoporosis for the items below
n1=50,n2=25,n3=9950,n4=9975
answer 400
• Successful lactation is
a maintained by oestrogen F
bmaintained by progesterone F
initiated by LH surge F
maintained by HPL F
inhibited by dopamine T
• Following are required for haemostatic clot formation
coversion of prothrombin to thrombin T
platelet phospholipid T
active conversion of plasminogen to plasmin F
fibrin degradation products F
antithrombin F
• severe oligohydramnios with limb defects
• 2.60-70% ASSOSIATED WITH CLEFT LIP/ cleft palate
• closure of neural groove - 30 days
• closure of mid gut - 14 weeks
• cobalt - 60
• tecnitium- 99
• The pelvic surface of the sacrum?
- gives origin to the piriform muscles
- gives origin to the levator ani muscles
- is broader in the male than in the female
- trnasmit dorsal rami of sacral nerves
- is in contact with the anal canal
• Prolactin.
- release is stimulated by TRH
- plasma levels are raised in the first trim of preg.
- release is increased by suckling.
- maybe produced by decidua.
- release is inhibited by metoclopromide.
• The foll disorders and org are correctly paired.
- opthalmia neonatorum: chlamydia trachomatis
- chancroid: Haemophilus ducreyi.
- sleeping sickness: Leishmania donovani.
- ringworm: Trichenella spiralis.
- non-specific urethritis: Toxoplasma gondii.
• The heart rate typically increases in response to:
- pain.
- hypoxia.
- ventilatory expiration
- increase in Intracranial pressure
- decrease baroreceptor activity
• Antibodies.
- are proteins.
- are formed in the fetus before 12 weeks of Intrauterine life.
- have an average molecular weight of around 10 000 daltons.
- of the rhesus type are genetically transmitted.
- are produced by the ribosome of plasma cells.
• The following disorders have an X linked pattern of inheritance.
- G6PD Deficeincy.
- Kleinfelter syndrome
- adrenogenital syndrome
- haemophilia B
- familial hypercholestroleamia.
• The following are derived from the urogenital sinus:
- the bladder trigone.
- the ureters.
- the female urethra.
- greater vestibular glands.
- paraurethral glands.
• In the fetal CVS
- the heart arises from endoderm
- the heart is formed by fusion of endocardial tubes.
- Cardiac pulsation is present by the 30th day after fertilization.
- oxygenated blood is tranferred to the left atrium through the foramen
ovale.
- the ductus arteriosus closes during the last 4 weeks of pregnancy.
• Arginine vasopressin
- reduces GFR.
- controls water loss in the Proximal renal tubule.
- is synthesised by the post pituitary gland.
- is released in response to rise in plasma osmolality.
- is released in response to fall in circulating plasma volume.
• Renin
- is secreted by the zona glomerulosa of the adrenal coretx.
- is a proteolytic enzyme.
- is secreted at an increased rate if the renal perfusion pressure falls.
- acts upon circulating angiotensinogen.
- is released in response to an increase in extracellular fluid volume
• Actinomyces israelii
- is a rickettsia.
- forms yellow granules in pus.
- is a commensal in the mouth.
- is a commensal in the vagina.
- is usually resistant to penicillin.
• Diseases caused by spirochaetes include.
- Weil's disease.
- lymphogranuloma venereum
- pinta
- Vincent's angina.
- bilharzias
• The following are inherited as autosomal recessive conditions:
- tuberous sclerosis.
- phenylketonuria.
- achondroplasia
- sickle cell anaemia.
- Von gierke's disease.
• Antibodies play an important part in the development of:
- phagocytosis.
- Mantoux responce.
- erythroblastosis fetalis.
- hyperemesis gravidarum
- anaphylaxis
• chemical mediators concerned in the production of an inflammatory
response include:
- 5-hydroxytryptamine
- aldosterone.
- glucocorticoids.
- bradykinin
- leukotreines
• The parathyroid gland.
- originate from the pharyngeal cleft ectoderm.
- secrete parathyroid hormone via the chief( principal) cells.
- secrete calcitonin via the oxyphil cells.
- may become hyperplastic in the presence of intestinal malabsorption.
- may develop adenomas in association with islet cell tumour of the
pancreas
• In the abdominal wall:
- the rectus abdominis muscle is attched to the crest of the pubis.
- the post border of the external oblique muscle ends in the linea
semilunaris.
- the aponeurosis of the external oblique muscle takes part in the
formation of the conjoint tendon.
- the inferior epigastric artery is a branch of the internal iliac artery.
- the conjoint tendon blends medially with the anterior layer of the
rectus sheath.
• The urogenital sinus in the female gives rise to the following:
- ureter
- paraurethral glands
- Bartholin's gland
- urachus
- Gartner's duct.
• Concerning sex hormone:
- the ovary secretes androstenedione.
- The ovary secretes testosterone
- The ovary secretes dihydrotestosterone.
- SHBG conc. are higher in women more than men
- Androgens bound to protein have high biological activity.
• Features of congenital rubella include:
- excretion of virus by the neonates.
- hepatomegaly
- excessive production of growth hormone.
- cataract
- deafness.
• Clomifene citrate:
- is an anti-androgens.
- does not stimulate ovulation directly.
- can produce visual disturbance.
- is genereally prescribed throughtout the proliferative phase of the
menstrual cycle.
- in the treatment of ovulation increases the risk of multiple pregnancy.
• The foll are cytotoxic alkylation agents:
- Cyclophosphamide.
- mercaptopurine
- chlorambucil
- fluorouracil
- methotrexate.
• If a distribution of results is markedly skewed to the left:
- the mean is the same as the 50th centile.
- the same number of values lie on either side of the median.
- the mode is equal to the median.
- the student's t test should be used to compare the distribution with
another.
- logarithmic transformation of the result will produce a distribution
closer to the normal.
• Corcerning the analysis of clinical trials:
- the 95% confidence interval indicates the range within which 19 out of
20 values will lie.
- The P value illustrates how often the result would be expected to occur
by chance.
- c)b The conventional level of statistical significance is set of P<0.005
- In a randomised trial, there must be equal numbers of results in each
arm of the study.
- A relative risk reduction of 60% is significant irrespective of the value
of P.
• The following substance are normally synthesized in the liver:
- glucagon
- vitamin A
- cholesterol.
- immunoglobulins
- prothrombin
• The pineal gland:
- is situated at the anterior end of the 3rd ventricles.
- is innervated by the parasymphathetic nervous system.
- produces melatonin.
- maybe calcified in the adults.
- is most active during daylight.
• In congenital adrenal cortical hyperplasia.
- The commonest deficiency is C18 hydroxylase.
- plasma cortisol concentration is raised.
- urinary excretion of 17 oxysteroids is elevated.
- dexamethasone will suppress the urinary excretion of 17 oxysteroids.
- there is no virilising effects.
• 28)The following are RNA containing virus:
- coxsackie
- influneza
- mumps
- herpes simplex.
- cytomegalovirus.
• Listeria monocytogenes:
- is a gram negative organism.
- is sensitive to ampicillin.
- may cause a transplacental infection.
- is sexually transmitted.
- can be cultured from a high vagina swab.
• actinomyces israelii.
- is a fungus.
- forms yellow granules in pus.
- is a mouth commensal
- occurs in association with IUCD.
- is resistant to penicillin.
• The following drugs may cause enlargement of the fetal thyroid gland:
- methyldopa
- thyroxine
- carbimazole
- propranolol
- propylthiouracil
• The following statements about anticoagulant are correct:
- Heparin inhibits the action of thrombin
- The action of heparin is antagonised by vitamin K
- Heparin increases antithrombin III activity.
- The effects of anticoagulants are decreased by metronidazole.
- Warfarin is greater than 80% protein bound in plasma
• The following drugs and side effects are associated:
- methydopa: depression
- paracetamol: thromboembolism
- indomethacin: peptic ulcer
- prednisolone: osteoporosis.
- ritodrine: hypoglycemia
• Haematopoiesis in the fetus:
- results in nucleated erythrocytes early in development.
- occurs in the yolk sac in the first month.
- does not occur in the bone marrow until term.
- is predominantly hepatic during the 4th month.
- does not require folic acid.
• Early blood borne dissemination is characterised feature of:
- carcinoma of the endometrium.
- osteosarcoma
- basal cell carcinoma
- carcinoma of the cervix
- choriocarcinoma.
• The following cells maybe phagocytic:
- neutrophils.
- kupffer cells
- monocytes
- Hofbauer cells.
- plasma cell.
• The following are premalignant conditions:
- diverticular disease of the large bowel.
- ulcerative colitis.
- pulmonary asbestosis.
- Paget's disease of the bone.
- condylomata of the vulva
• In the pituitary gland:
- the anterior lobe is smaller than the posterior lobe.
- the posterior lobe is ectodermal in origin.
- the acidophil cells produces oxytocin.
- the basophil cells produce growth hormone.
- the blood supply is derived from the internal carotis artery.
• The obturator artery:
- branches from the posterior trunk of the internal iliac artery.
- passes through the greater sciatic foramen.
- is crossed by the ureter.
- supplies the hip joint.
- may be replaced by a branch of the superior epigastric artery.
• In congenital adrenal hyperplasia:
- the commonest cause is a deficiency of 21 hydroxylase.
- the plasma cortisol conc is increased.
- there may be excessive secretion of 17 alpha hydroxyprogesterone.
- sodium retention is characteristic.
- blood cathecholamine conc are increased.
• In the fetal lung:
- bronchial cartilage formation commences at 18-24 weeks of gestation.
- type II alveolar cells first appear at 16-20 weeks gestation.
- sphingomyelin is the most common phospholipid present at term
- phospholipid release is increased by endogenous adrenaline.
- phospholipid production is decreased by exogenous corticosteroids.
• In radiotherapy
- 1 gray is equivalent to 1 joule/kg.
- the skin usually receives a greater dose of radiation than the
underlying tissues.
- the major effect of radiation energy is to damage the cytoplasm of the
cell.
- cells in tissues which are hypoxic are more vulnerable to radiation.
- Radiation induced changes in tissues may take 6 weeks to develop.
• Concerning the adrenal glands
- cortex is derived from neural crest cells.
- Zona fasiculata secretes aldosterone.
- Cortical adenomas may cause Cushing syndrome.
- Neuroblastoma arise in the medulla.
- Addison's disease may result from autoimmune destruction of the
cortex.
• The following are autosomal recessive:
- neurofibromatosis.
- cystic fibrosis.
- phenylketonuria
- polyposis coli
- sickle cell anaemia
• Uterine fibroids:
- are defines histologically as fibromyxomas.
- arise from endometrial stroma
- maybe associated with polycythamia.
- predispose to endometrial hyperplasia.
- are liable to sarcomatous change in about 5% of cases.
• growth of the foll tumors are hormone dependent:
- squamous cell carcinoma of the cervix.
- breast adenocarcinoma.
- uterine leiomyoma.
- prostatic adenocarcinoma
- testicular carcinoma
• Surfactants:
- is formed mainly in the placenta
- levels in amniotic fluid diminish after 33 weeks of gestation
- formation can be inferred from the lecithin-sphingomyelin ratio in
amniotic fluid.
- contains palmitic acid
- decreases the surface tension in pulmonary alveoli.
• Intracellular fluid differs from Extracellular fluids in that:
- it forms the major proportion of total body water.
- its volume can be measured easily.
- it has a higher concentration of potassium than of sodium.
- its volume is regulated primarily by the kidneys.
- it has a higher phosphate concentration.
• The Anal canal:
- has an upper part which is innervated by the inferior hypogastric
plexus.
- has a lower part which is supplied by the superior rectal artery
- drains lymph to the superficial inguinal nodes from its upperpart.
- has its internal sphincter innervated by the infecrior rectal nerve.
- has a superficial part of its external sphincter attached to the coccyx.
• The right ovarian artery:
- Arises from the abdominal aorta above the renal artery.
- passes posterior to the 3rd (horizontal) part of the duodenum
- passes post to the genitofemoral nerve.
- supplies the right ureter.
- anastomoses with the right uterine artery.
• Vulva supplied by:
- Internal pudendal artery.
- Inferior rectal artery.
- Genitofemoral artery
- obturator artery
- femoral artery..
• This is a question outside past papers..All r false Except internal
pudendal artery..
• The vagina:
- has an anterior wall longer than the post wall.
- contains mucus secreting glands in its epithelium.
- is related in its lower third to the bladder base.
- during reproductive life has an acid pH.
- is derived from mesonephric duct.
• The right ureter:
- is approx 50cm in length.
- ia partly covered by duodenum
- crosses the genitofemoral nerve.
- enters the bladder anteromedially.
- receives part of its blood supply from the uterine artery.
• After birth:
- allantois froms median umblical ligament
- umbilical vein forms medial umbilical ligament.
- umbilical artery forms superior vesical artery.
- ductus venosus forms the ligamentum teres.
- ductus arteriosus forms the arch of the aorta.
• In spermiogenesis:
- primary spermatocytes undergo reduction division.
- primary spermatocyte gives rise to 4 spermatids.
- whole process of spermatogenesis in man takes 6-7 days.
- grossly abnormal spermatozoa may be found in fertile semen.
- spermatids are haploid.
• In the human male, dihydrotestosterone:
- is a precursor of testosterone.
- has one-tenth of the ptency of testosterone.
- is responsible for involution of the Mullerain system.
- is responsible of the male external genitalia.
- binds to an intracellular receptor.
• In Human lactation:
- estrogens promote development of breast lobules.
- estrogen promotes milk producing effect of prolactin on the brest,
- human placental lactogen is essential for milk synthesis.
- prolactin stimualtes gonadotrophin release.
- oxytocin causes milk ejection,
• Concerning testicular hormones:
- testosterone reduces plasma LH conc.
- Inhibin stimulates LH production.
- Estrogen are formed in the testis.
- Testosterone is converted to dihyrotestosterone by 5 alpha reductase.
- Testosterone in plasma is predominantly bound to albumin.
• Epidermal growth factor:
- is mitogenic.
- synthesis is stimulated by estradiol.
- is a steroid molecule.
- is found in endometrium.
- binds to a receptor on the nuclear membrane.
• folic acid:
- deficiency causes megaloblastic bone marrow..
- is hydroxycobalamin.
- is present in green vegetables.
- is predominantly absorbed from the large intestine.
- is destroyed by boiling water.
• Doppler Ultrasound:
- is used to monitor fetal breathing.
- is used in fetal HR monitors.
- can be used to measure blood velocity in the fetus.
- measure proton relaxation times.
- requires injection of contrast agents.
• The following are structural aberration of chromosomes:
- deletions.
- inversions.
- aneuploidy.
- polyploidy.
- translocation.
• Messenger RNA
- synthesis is dependant on RNA polymerase.
- is an exact copy of sense DNA.
- contains exons.
- is measured by western analysis.
- translationoccurs in the nucleus.
• In tumors of the bones:
- primary malignancy is more common than 2ndry malignancy.
- osteoma rarely present in skull bones.
- osteosarcoma is associated with Paget's disease of bone.
- lymph node metastases are unusual.
- simple bone cysts have a strong tendency to recur.
• Concerning the rectus sheath.
- Above the costal margin rectus abdominis lies on the costal cartilages.
- Below the arcuate line the internal oblique splits to enclose rectus
abdominis.
- It contains the musculophrenic artery.
- It is innervated by the ilioinguinal nerve
- Pyramidalis is innervated by the subcostal nerve.
• The external iliac artery.
- Enters the thigh anterior to the inguinal ligament.
- at its origin is crossed by the ureter.
- at its origin is crossed by the ovarian vessels.
- Lies medial to the external iliac vein at its distal end.
- gives rise to the deep external pudendal artery.
• Action of Insulin include stimulation of:
- glycogenolysis by the liver.
- cellular uptake of amino acids.
- entry of glucose into neurons.
- d)entry of glucose into adipose tissue.
- cellular uptake of potassium.
• Metronidazole:
- is effective against Giardia lamblia.
- is effective when administered per rectum
- should not be administered intravenously.
- is usually effective against Entamoeba histolytica.
- interferes with ethanol metabolism.
• Amniotic fluid:
- at term is hyperosmolar compared to fetal plasma.
- normally contains maternal and fetal cells.
- contains a higher concentration of alpha feto protein at 16 weeks than
at term.
- contains bilirubin.
- contains phospholipids.
• Chromosomes:
- are found in the same numb in all mammalian cells.
- can be analysed more quickly from a blood sample than from an
amniotic fluid sample.
- can be reliably identified by their length.
- the Y chromosome is larger than the X chromosome.
- DNA content is doubled during the S phase of the cell cycle.
• In cystic fibrosis, abnormalities are seen in the
- pancreas
- salivary glands.
- brain
- kidneys
- ileum
• Neutrophil polymorphs at the site of inflammation are capable of the
following:
- Phagocytosis.
- production of oxygen free radicals.
- replication
- fusion to form giant cells.
- antibody production
• In the statistical analysis of any group of numericals
- the mean is always less than the mode
- standard deviation is always greater than the standard error
- the median value lies at the midpoint of the range
- the standard error of the mean is independent of the total
number of observations
- there are the same number of observations greater than and
less than the median value
• In a randomised double blind trial comparing a new drug with a
placebo
- the patient will be taking either of two drugs
- patients can choose their mode of treatment
- doctors prescribing treatment decide which patients take the
new drug
- a large trial is more likely to give a statistically significant result
than a small trial
- half of the patients will take the new drug
• In a trial of oral hypoglycaemic agents 42 patients were given drug A
and 38 drug B . blood glucose conc. were measured befor and after a
single dose of the drug. Drug B apparently caused greater fall in the
blood glucose conc. (P=0.06)
- these results reach an accepted level of statistical significants
- Non- parametric statistical analysis bshould be used if datd are
not normally distributed
- In biological terms drugs A and B have been shown to be eqully
effective
- 6% more patients responded to drug A than drug B
- Unequal numbers in the two groups invalidate the trial
• The following statistical statements are correct
- In the normal distribution the value of the mode is 1.73 * that
of the median
- In a distribution skewed to the right, the mean lies to the left to
the median
- In the series 2;7;5;2;3;2;5;8, the mode is 2
- Student's test is designed to correct for skewed distributions
- the Chi-squared test may be used when data are not normally
distributed
• pudendal nerve, nerve for perianal area, down syndrome, patau
syndrome, cobalt, technitium, dna virus, HIV virus, syphillis, heptitis b
and c virus, PG I2, ionic exchange assosiated with calcium and
potassium, positive predictive value, negative predictive value,
aspirin,mefanamic acid, insulin secretion and glucagon sec inhibited
and stimulated by what substances
• PUDENDAL NERVE ROOT VALUE S1,2,3 ANT PRIMARY RAMI
• N SUPPLY OF PERIANAL AREA
• CLOSURE OF NEURAL GROOVE ON -WK OF IUL
• LENGTH OF UMBILICAL CORD
• AUTOSOMAL RECESSIVE TAY SACH'S DISEASE
• AUTOSOMAL DOMINANT ACHONDROPLASIA
• ANTIINFLAMMATORY MUSCLE RELAXANT PROSTAGLANDIN
• STAPH AUREUS COAGULASE POSITIVE, PIGMENTED COLONY,GRAM
POSITIVE
• GENTAMYCIN BACTERICIDAL,TOXIC TO 8 TH NERVE
• DIURETIC ACTING ON DCT BENDROFLUAZIDE
• Successful lactation is
- A- maintained by oestrogen F
- B-maintained by progesterone F
- c -initiated by LH surge F
- d- maintained by HPL F
- e -inhibited by dopamine T
• The following are required for haemostatic clot formation
- a -coversion of prothrombin to thrombin T
- b -platelet phospholipid T
- c -active conversion of plasminogen to plasmin F
- d -fibrin degradation products F
- e -antithrombin F
• the following statements about vitamins are true
- vit k is water soluble
- vit D is poorly absorbed in cases of obstructive jaundice
- vit A is a fat soluble vit.
- vitamins supply the body wd energy
- vit D is bound to transport protein in the circulation.
• ANSWERS
- a.F- fat soluble
- b.T- also in cases of pancreatic disease where there is pancreatic lipase
deficiency.
- c.T-
- d.F- vitamins r organic dietry constituents necessary for life haelth n
growth that do not function by supplying the body wd energy.
- e- T- physiologically active form of vitamin D is known as calcitriol.
- calcitriol is released into the circulation, and by binding to a carrier
protein in the plasma, vitamin D binding protein (VDBP), it is
transported to various target organs.
• VITAMIN B12
- a fat soluble vit
- b-absorption takes place throughout the small intestine
- c-is essentail for the metabolism of folic acid in human body.
- d-deficiency leads to macrocytic anemia
- e-deficiency is common in strict vegetarian.
• ANSWERS
- a-F- water soluble
- b-F-absorbed mainly in lower ileum aided by gastric intrinsic factor.
- C-T
- d-T
- e-T- source is mainly from animal food stuff, vegetables alone are
inadequate source.
• FOLIC ACID
- a-water soluble
- b-requires gastric intrinsic factor
- c-is necessry for nucleic acid synthesis
- d-heat stable
- e-involved in krebs cycle
• ANSWERS
- a-T
- b-F
- c-T-tetrahydrofolate is essentaial for both purine and pyramidine
synthesis.
- d-F- the normal western diet contains 500-700 microgm/day of wwch
10-100% lost in cooking.
- e-F
• Folic acid (also known as vitamin B9 or folacin) and folate (the
naturally occurring form), as well as pteroyl-L-glutamic acid and
pteroyl-L-glutamate, are forms of the water-soluble vitamin B9.
• Folic acid is itself not biologically active, but its biological importance is
due to tetrahydrofolate and other derivatives after its conversion to
dihydrofolic acid in the liver.
• Vitamin B9 (folic acid and folate inclusive) is essential to numerous
bodily functions ranging from nucleotide biosynthesis to the
remethylation of homocysteine.
• The human body needs folate to synthesize DNA, repair DNA, and
methylate DNA as well as to act as a cofactor in biological reactions
involving folate.
• It is especially important during periods of rapid cell division and
growth.
• Both children and adults require folic acid to produce healthy red blood
cells and prevent anemia.
• Leafy vegetables such as spinach, asparagus, turnip greens, romaine
lettuces, dried or fresh beans and peas, fortified grain products (pasta,
cereal, bread), sunflower seeds and certain other fruits (orange juice,
canned pineapple juice, cantaloupe, honeydew melon, grapefruit juice,
banana, raspberry, grapefruit, strawberry) and vegetables (beets,
broccoli, corn, tomato juice, vegetable juice, brussels sprouts, bok
choy) are rich sources of folate.
• Liver and liver products also contain high amounts of folate, as does
baker's yeast.
• Some breakfast cereals (ready-to-eat and others) are fortified with
25% to 100% of the recommended dietary allowance (RDA) for folic
acid.
• Folic acid naturally found in food is susceptible to high heat, UV, and is
soluble in water.
• Folic acid is an important nutrient for women who may become
pregnant, because a woman's blood levels of folate fall during
pregnancy due to an increased maternal RBC synthesis in the first half
of the pregnancy and fetal demands in the second half.
• The first four weeks of pregnancy (when most women do not even
realize they are pregnant) require folic acid for proper development of
the brain, skull, and spinal cord.
• Serious birth defects like neural tube defects are less likely to occur
when women take 0.4 mg of folic acid daily.
• Neural tube defects (NTDs) result in malformations of the spine (spina
bifida), skull, and brain (anencephaly).
• The risk of neural tube defects is significantly reduced when
supplemental folic acid is consumed in addition to a healthy diet prior
to and during the first month following conception.
• Folate deficiency during pregnancy can increase the risk of preterm
delivery, infant low birth weight, and fetal growth retardation.
• Folate deficiency in the mother increases homocysteine level in the
blood which may lead to spontaneous abortion and pregnancy
complications such as placental abruption and preeclampsia.Folic acid
appears to reduce the risk of stroke.
• Folic acid supplements help relieve hot flushes in postmenopausal
women. Just like in estrogen hormone replacement therapy, folic acid
interacts with neurotransmitters (norepinephrine, serotonin) in the
brain to reduce hot flushes.
• IN SHORT: Folate deficiency may lead to glossitis, diarrhea,
depression, confusion, anemia, and fetal neural tube defects and brain
defects (during pregnancy).
• VITAMIN C
- a-is found only in animal food stuffs.
- b-is rapidly destroyed by heating.
- c-there are normally large stores in pancreas.
- d-impared wound healing is one of the characterstic features of severe
vit c defeciency.
- e-exess vit c can lead to formation of oxalate stones in urinary tract.
• ANSWERS
- a-F. citrus foods n leafy green veges r rich in vit c while animal sources
contain only traces.
- b-T
- c-F-eye n adrenal glands contain large quantities of vit c.
- d-T
- e-T
• Vitamin C or L-ascorbic acid is an essential nutrient for humans, in
which it functions as a vitamin.
• Ascorbate (an ion of ascorbic acid) is required for a range of essential
metabolic reactions in all animals and plants.
• Deficiency in this vitamin causes the disease scurvy in humans.
• Ascorbic acid is absorbed in the body by both active transport and
simple diffusion.
• Sodium Dependent Active Transport - Sodium-Ascorbate Co-
Transporters (SVCTs) and Hexose transporters (GLUTs) are the two
transporters required for absorption.
• Although the body's maximal store of vitamin C is largely determined
by the renal threshold for blood, there are many tissues which
maintain vitamin C concentrations far higher than in blood.
• Biological tissues that accumulate over 100 times the level in blood
plasma of vitamin C are the adrenal glands, pituitary, thymus, corpus
luteum, and retina.
• Those with 10 to 50 times the concentration present in blood plasma
include the brain, spleen, lung, testicle, lymph nodes, liver, thyroid,
small intestinal mucosa, leukocytes, pancreas, kidney and salivary
glands.
• Scurvy is an avitaminosis resulting from lack of vitamin C, since without
this vitamin, the synthesised collagen is too unstable to perform its
function.
• Scurvy leads to the formation of liver spots on the skin, spongy gums,
and bleeding from all mucous membranes.
• The spots are most abundant on the pages and legs, and a person
with the ailment looks pale, feels depressed, and is partially
immobilized.
• In advanced scurvy there are open, suppurating wounds and loss of
teeth and, eventually, death.
• The human body can store only a certain amount of vitamin C, and so
the body soon depletes itself if fresh supplies are not consumed.
• In humans, vitamin C is essential to a healthy diet as well as being a
highly effective antioxidant, acting to lessen oxidative stress; a
substrate for ascorbate peroxidase;and an enzyme cofactor for the
biosynthesis of many important biochemicals.
• Vitamin C acts as an electron donor for important enzymes:[
• Vitamin C acts as an electron donor for eight different enzymes:
• Three participate in collagen hydroxylation.
• These reactions add hydroxyl groups to the amino acids proline or
lysine in the collagen molecule via prolyl hydroxylase and lysyl
hydroxylase, both requiring vitamin C as a cofactor.
• Hydroxylation allows the collagen molecule to assume its triple helix
structure and making vitamin C essential to the development and
maintenance of scar tissue, blood vessels, and cartilage.
• are necessary for synthesis of carnitine.
• Carnitine is essential for the transport of fatty acids into mitochondria
for ATP generation.
• The remaining three have the following functions in common but do
not always do this:
• -dopamine beta hydroxylase participates in the biosynthesis of
norepinephrine from dopamine.
• -another enzyme adds amide groups to peptide hormones, greatly
increasing their stability.
• -one modulates tyrosine metabolism.
• DAILY REQUIREMENTS:
• 75 milligrams per day: the United Kingdom's Food Standards Agency
• 45 milligrams per day: the World Health Organization
• VITAMIN B.
- a.B1(thiamin)deficiency leads to impaired collagen formation
- b.B1 stores in the body are adequate for upto 9 months
- c.B2(riboflavin) concentration is higher in fetus than in mother
- d.B6(pyridoxine)requirement in pregnancy is 25mg/day
- e.niacin is synthesised in the body from tryptophan.
• ANSWERS
- a-T-
- b-F- the body contains only 30mg(average adult requirement is 1-1.5)
- c-T- as with other water soluble vit, vit B crosses the placenta by active
mechanisms, wch results in higher concentration in fetus
- d.F- the true figure is 2.5mg/day an din non pregnant adult is
2mg/day.
- e-T
• VITAMIN A
- a.requires bile for its absorption.
- b.deficiency leads to night blindness.
- c.excess leads to xerophthalmia
- d.stored in liver
- e.daily dietry requirement during pregnancy is 50mg/day
• ANSWERS
- a.F
- b.T
- c.F- xerophthalmia is due to its deficiency. hypervitaminosis A is
charachterised by anorexia , headache, hepatosplenomegaly,patchy
loss of hair and hyperostosis.
- d.T
- e.F-during pregnancy the requirement is 1000
- microgm/day.
• Vitamin A is a vitamin which is needed by the retina of the eye in the
form of a specific metabolite, the light-absorbing molecule retinal.
• This molecule is absolutely necessary for both scotopic and color
vision.
• Vitamin A also functions in a very different role, as an irreversibly
oxidized form retinoic acid, which is an important hormone-like growth
factor for epithelial and other cells.
• Vitamin A can be found in two principal forms in foods:
• -retinol
• -carotenes
• Vitamin A is found naturally in many foods:
• liver (beef, pork, chicken, turkey, fish)
• carrot
• broccoli leaf
• sweet potato
• kale
• butter
• spinach
• pumpkin
• collard greens
• cantaloupe melon
• egg
• apricot
• papaya
• mango
• pea
• broccoli
• Vitamin A plays a role in a variety of functions throughout the body,
such as:
• Vision
• Gene transcription
• Immune function
• Embryonic development and reproduction
• Bone metabolism
• Haematopoiesis
• Skin health
• Antioxidant Activity
• DEFICIENCY leads to
• -night blindness
• -xerophthalmia.
• First there is dryness of the conjunctiva (xerosis) as the normal
lacrimal and mucus secreting epithelium is replaced by a keratinized
epithelium.
• This is followed by the build-up of keratin debris in small opaque
plaques (Bitot's spots) and, eventually, erosion of the roughened
corneal surface with softening and destruction of the cornea
(keratomalacia) and total blindness.
• Other changes include impaired immunity, hypokeratosis (white lumps
at hair follicles), keratosis pilaris and squamous metaplasia of the
epithelium lining the upper respiratory passages and urinary bladder to
a keratinized epithelium.
• With relations to dentistry, a deficiency in Vitamin A leads to enamel
hypoplasia.
• Adequate supply of Vitamin A is especially important for pregnant and
breastfeeding women, since deficiencies cannot be compensated by
postnatal supplementation.
• However, excess Vitamin A, especially through vitamin
supplementation, can cause birth defects and should not exceed
recommended daily values.
• VITAMIN E
- a.present in animal food stuffs only.
- b.deficiency may cause intrauterine fetal death
- c.potentiates the action of coumarin anticoagulants
- d.used in the treatment of infertility
- e.dietry requirement is 10mg/day
• ANSWERS
- a.F-present in most foods
- b.F-proven only in animals and not in humans
- c.T
- d.F-there is no evidence that vit e increases virility, or plays any role in
treatment of infertility or recurrent abortion.
- e.T
• VIT D
- a.water soluble
- b.is stored in body fat
- c.is absorbed from large intestine
- d.deficiency leads to rickets
- e.dietry req is 10mg/day
• ANSWERS
- a.F
- b.T
- c.F-it is absorbed from small intestine
- d.T
- e.T
• VITAMIN K
- a.mainly found in green leafy veges
- b.in adults no external supplements necessary
- c.hypervitaminosis is charactererised by anemia
- d.exists in 2 forms k1 n k2
- e.is stored in large quantities in the liver
• ANSWERS
- a.T
- b.T-except in pregnant pts on anti epilactics who require vit k in last
months of pregnancy and in new borns.
- c.T
- d.T
- e-Falthough accumulates in liver initially but its hepatic concentration
declines rapidly.
• REGARDING metabolism
- a.the metabolic rate is the amount of energy liberated per unit of time
- b.anabolism is defined as the formation of substances which can store
the energy.
- c.basal metabolic rate is defined as the metabolic rate determined at
rest in a room at 12-14hrs after the last meal.
- d.the BMR of a man is about 500kcal/day
- e.the metabolic rate is decreased after consumption of a meal that is
rich in protein.
• ANSWERS
- a.T
- b.T
- c.T
- d.F-2000kcal/day
- e.F-the metabolic rate is increased after consumption of a meal rich in
protein or fat.
• ANABOLISM
- the set of metabolic pathways that construct molecules from smaller
units.
- These reactions require energy.
• BASAL METABOLIC RATE
• The primary organ responsible for regulating metabolism is the
hypothalamus.
• REGARDING METABOLISM
- a.oxidation is the comination of a substance with either oxygen or
hydrogen.
- b.Co-factors r essential for certain enzyme reactions.
- c.A co-enzyme is a protein substance wch acts as a carrier for products
of reaction.
- d.Co-enzyme A is a high energy compound wch is formed from
adenine,ribose pentotothenic acid and thioethanol amine.
- E.a calorie is defined as the amount of heat energy needed to
raise the temperature of 1gm of water by 1 degree ,from 15 to
16degreesC.
• ANSWERS
• a.F-oxidation is the combination of a substance with oxygen or loss of
hydrogen or loss of an electron. reduction is the reverse of this.
• b.T
• c.F-it is an organic non-protein substance.
• d.T
• e.T
• ENZYMES
- a.are proteins
- b.heating usually results in a complete loss of enzyme activity
- c.a change in pH has no effect on the activity of an ezyme.
- d.are present in all cell organelles.
- e.organic solvents will usually destroy an enzymes activity.
• ANSWERS
- a.T
- b.T
- c.F-a change in pH has an effect on enzyme activity.
- d.T
- e.T
• PROTEIN METABOLISM
- a.proteins contain abt 40%nitrogen
- b.chains containing>100 amino acid residues are called proteins
- c.proteins yield 4 calories /gm absorbed.
- d.during pregnancy there is a rise in the plasma concentration of
triglycerides.
- e.during pregnancy there is a rise in the plasma concentration of
albumin.
• ANSWERS
- a.F
- b.T
- c.F
- d.T
- e.F
EMQ:
• ant division of ant. remi of S2-4
• ant division of post.rami of S2-4
• genitofemoral nerve
• post division of ant. remi of S2-4
• post division of post. remi of S2-4
• sensory supply of perineal nerve
• inferior rectal nerve
• ischial tuberosity
• ischial spine
• A lady is about to deliver and you are about to give her a pudendal
block..
• Q1- what is the root value of pudendal nerve?
• Q2- what nerve supplies lower part of vagina (I think)?
• Q3- why do u give local skin infiltration before episiotomy?
• Q4- or what is the nerve supply of peranal area .?
• (2).. If we take the day of fertilization as Day’0’ then…
• Day2
• Day4
• Day 8
• Day 10
• Day12
• Day 14
• Day 18
• Day20
• Day 22
• Day 24
• Day 26
• Day 42 [
• Day 70
• Q1- Which represents 4 cell stage ?
• Q2- conceptus implants completely?
• Q3- fetal heart pulse seen on ultrasound?
• Q4- vertebra form completely?
• A Cytoplasmic transcription factor receptor
• B G protein coupled receptor on cell membrane
• C G protein coupled receptor on Golgi complex
• D Mega subunit ligand gated ion channel
• E Multisubunit antibody receptor on cell membrane
• F Multisubunit ligand gated ion channel on cell membrane
• G Nuclear protein kinase receptor
• H Nuclear transcription factor receptor
• I Protein kinase receptor on cell membrane
• J Receptor protein complex (intracytoplasmic)
• K Transcription factor receptor on cell membrane
• Select the receptor which binds the molecules referred to in the itemS
below. Each option may be used once, more than once or not at all.
- insulin .
- progesterone .
- ostradiol .
- prostaglandin .
• (4)
• PCR
• Fish test
• Chromosomal linkage analysis
• Telomer analysis
• Sum hereditary crosslinkage chromosomal option (dnt remember)
• Screen for some known gene mutation
• Sweat test
• Saliva test
• -A family with one kid presumed to have cystic fibrosis however cftr
gene mutation was not detected. Family members willing to give
samples if needed.mother wants definite diagnosis as she is 11wks and
wants to know diagnosis for her baby.parents are cousins with 1st
degree relatives having cystic fibrosis.
• Invasive diagnostics can be applied to reach final
results by?
• you have Guthrie spot of their son how will u
diagnose him for cystic fibrosis?
• families in UK with cystic fibrosis how will u find
f508?
• (5)
o Endoplasmic reticulum
o Golgi apparatus
o Lysosomes
o Microtubules
o Mitochondria
o Nucleolus
o Plasma membrane
o Ribosomes
• Select the most appropriate organelle that matches the following
descriptions
Contains enzymes capable of digesting cells and cellular
material
'Reads' the mRNA and builds protein .
Modification of lipids and proteins with storage of
material prior to export out of the cell.
• (6)[i][b] screening test for following
syphilis treponemal antibody test
syphilis hemaglutination test
syphilis immobilization test
HB-electrophorisis
mcv
mchc
MCh
serum iron
paul-bennel test[/b][/i]
• alpha thalessemia
• folic acid deficiency
• iron deficiency anemia
• primary syphilis in early untreated period
• (7) on taking a large protein meal?
• Decrease decrease
• Increase increase
• Increase decrease
• Deacrease increase
• Nochange no change
• Nochange increase
• Nochange decrease
• Insulin & Glucagon excretion what will happened ...?
• (8)
o 3-hydroxyisovaleric acid .
o 17-alpha hydroxyprogesterone .
o 17-delta hydroxyprogesterone .
o acetic acid .
o cortisol .
• The following level of certain metabolities in amniotic fluid changes
significantly of the fetus has an inborn error of the metabolism .select
the single metabolite from list of the options above whose level in
amniotic fluid is altered by inheretiance disease in item below.
• congentiat adrenal hyperplasia
• (9)
Adrenaline
Calcitonin
• C Cholecystokinin
Cortisol
Glucagon
Insulin
Progesterone
Somatostatin
• IT- estosterone
• Select the appropriate hormone from the list for the following
structures that produce it. Each answer may be used once, more than
once, or not at all.
Adrenal Cortex .
Adrenal Medulla .
Pancreatic alpha cell .
Pancreatic D Cells .
• (10)
• AAnterior cerebral artery
• BAnterior communicating artery
• CAnterior inferior cerebellar artery
• DBasilar artery
• EInternal auditory artery
• FInternal carotid artery
• GMiddle cerebral artery
• HPosterior cerebral artery
• IPosterior communicating artery
• JSuperior cerebellar artery
• KVertebral artery
• Complete the diagram of the circle of Willis using the options given:
• this question came with diagrame in september 2007
• so you should study circle of weils very well if I get the pictures I will
post it .
• Akeratinising stratified squamous epithelium
• Bnon-keratinising stratified squamous epithelium
• Cpseudostratified columnar epithelium
• Dsimple columnar epithelium
• Esimple cuboidal epithelium
• Ftransitional epithelium
• Which of the epithelial types described above are present in the
anatomical regions below:
ectocervix .
endocervical canal .
cervical ectropion .
transformation zone of cervix .
vagina .
uterus .
Bladder .
Trachea .
Labia Majora .
Bowel .
• 11-Fallopian tube .
Vulva .
anal canal .
• Autosomal co-dominant
• Autosomal dominant
• Autosomal recessive
• Polygenic
• Single gene defect
• X linked dominant
• X linked recessive
• [Select the most likely mode of inheritance for the following patients’
conditions:
- A 27 -year old female developed gestational diabetes mellitus.
Her uncle and grandmother also had diabetes mellitus2- A
mothr is concerned regarding her baby who has developed
fractures which appear to occur with minimal trauma. He has
blue sclera.
- An 18-year old female underwent caries tooth extraction and
developed profuse bleeding. On history she revealed
menorrhagia. Her mother and her grandfather had the same
disease.
WARFARIN .
HEPARIN .
AMPICILLIN .
METHYLDOPA .
PENCILLIN .
CARBIMAZEPINE .
METRONIDAZOLE .
NON OF THE ABOVE .
ALL OF THE ABOVE .
• For each of the following choose the single most appropirate statement
from the above list of options .
- Drug contraindicated in breast feeding .
- Drug does not cross the placenta .
• A.
• Calcitonin
• B.
• Cortisol
• C.
• Glucagon
• D.
• Growth hormone
• E.
• Insulin
• F.
• Oestradiol 17-¦Â¦Â
• G.
• Oxytocin
• H.
• Parathyroid hormone
• I.
• Prolactin
• J.
• Thyroxine
• Instructions: For each action described below, choose the single most
likely causative hormone from the above list of options. Each option
may be used once, more than once, or not at all.
• Question 3: Stimulates deposition of cartilage at the ends of bones
• Question 4: Raises blood glucose levels through the breakdown of fat
and protein
• Question 5: Antagonises the effect of parathyroid hormone to minimise
bone density loss
• Question 6: Stimulates the release of milk from the breast
• Question 7: Stimulates the urinary secretion of calcium
• Options
• A.
• Common iliac artery
• B.
• External iliac artery
• C.
• Inferior epigastric artery
• D.
• Inferior vesical artery
• E.
• Internal iliac artery
• F.
• Middle rectal artery
• G.
• Ovarian artery
• H.
• Superior vesical artery
• I.
• Umbilical artery
• J.
• Uterine artery
• K.
• Vaginal artery
• Instructions: After a forceps delivery a 30-year-old primigravida
sustains a primary post partum haemorrhage of four litres. Although
the uterus appears well contracted the bleeding continues, and a
decision is made to identify and treat the bleeding point radiologically.
For each question posed below, choose the single most appropriate
option from the above list. Each option may be used once, more than
once or not at all.
• Question 8: What vessel runs up the broad ligament?
• Question 9: Which other vessel arises from the uterine artery?
• Question 10: Which other vessel anastomoses with the uterine artery?
Question 11: Which vessel does the uterine artery arise from?
• Question 12: Which vessel runs anterior and superior to the ureter?
- vitamin A
• B- vitamin E
• C- vitamin D
• vitamin B12
• vitamin B6
• vitamin C
• folic acid
• riblflavine
• vitamin K
• thiamine
- fat soluble vitamin sysnthesised in the intestinal wall from the
beta carotene .
- fat soluble vitamin synthesised by large intestinal bacteria .
- deficiency of this vitamin in childhood causes rickets .
- fat soluble vitamin synthesis by the kidney is regulated by
parathyroid hormone .
- deficiency of this vitamins typicaly occurs in women with
hyperemesis gravidarum .
- water soluble vitamin with anti-oxidant effects .
- absorption of this vitamin requires the presence of intrinsic
factor .
- deficiency of this vitamin causes megaloblastic anaemia and
neurological disorders .
- fat soluble vitamin produced in the skin by photo-activiation of
7- dehydrocholesterol .
- fat soluble vitamin with anti-oxdant effects .
- fat soluble vitamin deficiency associated with raised serum
alkaline phosphatase .
- deficiency of this vitamin associated with osteomalasia .
- maternal intake of this vitamin around the time of coception has
been shown to reduce incidance of neural tube defects .
- fat soluble vitamin , deficiency occur within few a days of
cessation of bile secretion .
- this vitamin is a cofactor in synthesis of prothrombin .
Candida spp
Herpes simplex virus
Human immunodeficiency virus
Gardnerella vaginalis
Chlamydia trachomatis
Human papilloma virus
Treponema pallidum
Neisseria gonorrhoeae
Trichomonas vaginalis
• Select the micro-organism described in the following cases:
- An obligate intracellular gram negative bacterium ....
- The cause of lymphogranuloma venereum .........
- Neutrophils containing gram negative diplococci .....
Double Y Syndrome
Down’s Syndrome
Edward’s Syndrome
Fragile X Syndrome
Klinefelter’s Syndrome
Patau’s Syndrome
Turner’s Syndrome
• Please select the most appropriate syndrome from the options below.
You may use each option once, more than once, or not at all.
- An infant girl is born with a webbed neck, hypertension and
audible ejection systolic murmur.
- A neonate boy is born with cleft lip and palate, low set ears and
polydactyly. The child survives for only 3 weeks.
- An infant boy is born with hypotonia, epicanthic folds and single
palmer creases.
Adrenal arteries
Celiac trunk
Common iliac arteries
Gonadal (ovarian or testicular) arteries
Inferior mesenteric artery
Lumbar arteries
Median sacral artery
Renal arteries
Superior mesenteric artery
• Select the most appropriate option:
- Gives rise to the left gastric, splenic and hepatic arteries.
- Gives rise to the intestinal, middle colic and right colic arteries.
- Gives rise to the left colic, sigmoid and superior rectal arteries.
- An unpaired branch that arises from the aortic bifurcation.
- Arises from the aorta between the level of L1 and L2 vertebrae,
and forms five segmental arteries that do not anastomose.
• B- Chlamydia trachomatis
• C- Herpes simplex virus
• D- Human immunodeficiency virus
• E- Gardnerella vaginalis
• F- Human papilloma virus
Neisseria gonorrhoeae
Treponema pallidum
Trichomonas vaginalis
• Select the micro-organism described in the following cases:
- A thin motile, gram negative spiral shaped bacterium ...
- An anaerobic flagellated protozoan ...
- A single strand RNA virus ..
ACTH
CRH
Dopamine
FSH
GnRH
Growth Hormone
LH
Prolactin
Somatostatin
TSH
• Please select the correct hormone from the options for each of the
descriptions below. You may use each option once, more than once, or
not at all.
- This hormone acts on cartilage and liver to release IGF-1...
- This hormone promotes iodination of tyrosine residues...
- This hypothalamic hormone inhibits the secretion of growth
hormone...
- In males, this hormone facilitates the generation spermatozoa...
- This hormone inhibits galactorrhoea...
•
Deep inguinal
Inferior mesenteric
Internal iliac
Para-aortic
Superficial inguinal
• Lymph from the following structures passes to the above group of
lymph nodes first. Please choose the most appropriate answer from the
list above.
- Rectal .
- Vulva .
- Hallux .
- Cervix .
- Upper anal canal .
Bias
Correlation
Error
False positive
False negative
Null hypothesis
Power
Regression
Sensitivity
Specificity
• Which of the above statistical terms is described by the following
statements:
- A random source of inaccuracy...
- A type 2 error...
- Ability to exclude a true negative...
Clostridium perfringens
clostridium difficile
Escherichia coli
Haemophilus influenzae
Staphylococcus aureus
streptococcus pyogenes
none of the above
• Select the most likely causative organism for the following infections.
- Chronic osteomyelotis after implant surgery ..
- Gas gangrene ..
- Pseudomembranous colitis ..
Mean
Median
Mid-range
Mode
Range
Spread
Standard Deviation
Standard Error
• Select the appropriate term from the list for the following definitions.
Each answer may be used once, more than once, or not at all.
- Most frequent value ..
- Spread of estimates of sample means around the true
population mean .
- The measure of spread of values around the mean .
- The mid value when all values are listed in ascending order .
- The sum of all the values divided by the number of values .
endothelial growth factor
human chorionic gonadotrophin
human placental lactogen
IGF-1
Insulin
leptin
oestrogen
progesterone
relaxin
• Select the most appropriate hormone for the following statements.
- The hormone that is the basis of most pregnancy tests.
- Hormone produced by synctiotrophoblast to regulate nutrient
storage in the final stages of pregnancy.
- Hormone that is secreted by decidual cells of the placenta .
- The hormone that is lactogenic, regulates glucose metabolism
and promotes fat breakdown .
• 1)
• ant division of ant. remi of S2-4
• ant division of post.rami of S2-4
• b)genitofemoral nerve
• post division of ant. remi of S2-4
• post division of post. remi of S2-4
• sensory supply of perineal nerve
• inferior rectal nerve
• ischial tuberosity
• h)ischial spine
• A lady is about to deliver and you are about to give her a pudendal
block..
• Q1- what is the root value of pudendal nerve? a
• Q2- what nerve supplies lower part of vagina (I think)? e/a
• Q3- why do u give local skin infiltration before episiotomy? e
• Q4- or what is the nerve supply of peranal area .?
• (2).. If we take the day of fertilization as Day’0’ then…
• Day2
• Day4
• Day 8
• Day 10
• Day12
• Day 14
• Day 18
• Day20
• Day 22
• Day 24
• Day 26
• Day 42 [
• Day 70
• Q1- Which represents 4 cell stage ? day 2
• Q2- conceptus implants completely? Day 10/12
• Q3- fetal heart pulse seen on ultrasound? Day 22
• Q4- vertebra form completely? Day 42
• PCR
• Fish test
• Chromosomal linkage analysis
• Telomer analysis
• Sum hereditary crosslinkage chromosomal option (dnt remember)
• Screen for some known gene mutation
• Sweat test
• Saliva test
• -A family with one kid presumed to have cystic fibrosis however cftr
gene mutation was not detected. Family members willing to give
samples if needed.mother wants definite diagnosis as she is 11wks and
wants to know diagnosis for her baby.parents are cousins with 1st
degree relatives having cystic fibrosis.
- Invasive diagnostics can be applied to reach final results by? ???
- you have Guthrie spot of their son how will u diagnose him for
cystic fibrosis? PCR???
- families in UK with cystic fibrosis how will u find f508? SWEAT
TEST????
• 5)
Endoplasmic reticulum
Golgi apparatus
Lysosomes
Microtubules
Mitochondria
Nucleolus
Plasma membrane
Ribosomes
• Select the most appropriate organelle that matches the following
descriptions
- Contains enzymes capable of digesting cells and cellular material
c
- 'Reads' the mRNA and builds protein . h
- Modification of lipids and proteins with storage of material prior
to export out of the cell. a
• (6) screening test for following
- syphilis treponemal antibody test
- syphilis hemaglutination test
- syphilis immobilization test
- HB-electrophorisis
- mcv
- mchc
- MCh
- serum iron
- paul-bennel test
alpha thalessemia 4
folic acid deficiency 6
iron deficiency anemia 8
primary syphilis in early untreated period 1
• on taking a large protein meal?
Decrease decrease
Increase increase
Increase decrease
Deacrease increase
Nochange no change
Nochange increase
Nochange decrease
• Insulin & Glucagon excretion what will happened ...? both will increase
in secretions…
• (8)
• 3-hydroxyisovaleric acid .
• 17-alpha hydroxyprogesterone .
• 17-delta hydroxyprogesterone .
• acetic acid .
• cortisol .
• The following level of certain metabolities in amniotic fluid changes
significantly of the fetus has an inborn error of the metabolism .select
the single metabolite from list of the options above whose level in
amniotic fluid is altered by inheretiance disease in item below.
• congentiat adrenal hyperplasia b
• (9)
Adrenaline
Calcitonin
• C Cholecystokinin
Cortisol
Glucagon
Insulin
Progesterone
Somatostatin
• IT- estosterone
• Select the appropriate hormone from the list for the following
structures that produce it. Each answer may be used once, more than
once, or not at all.
o Adrenal Cortex . d
o Adrenal Medulla . a
o Pancreatic alpha cell . e
o Pancreatic D Cells . h
• Akeratinising stratified squamous epithelium
• Bnon-keratinising stratified squamous epithelium
• Cpseudostratified columnar epithelium
• Dsimple columnar epithelium
• Esimple cuboidal epithelium
• Ftransitional epithelium
• Which of the epithelial types described above are present in the
anatomical regions below:
- ectocervix .
- endocervical canal . simple columnar
- cervical ectropion .
- transformation zone of cervix .
- vagina . A
- uterus . E
- Bladder . F
- Trachea .F
- Labia Majora .
- Bowel . simple columnar
• 11-Fallopian tube . ciliated epi
- Vulva .
- anal canal . simpe columnar
Autosomal co-dominant
Autosomal dominant
Autosomal recessive
Polygenic
Single gene defect
X linked dominant
X linked recessive
• [Select the most likely mode of inheritance for the following patients’
conditions:
- A 27 -year old female developed gestational diabetes mellitus.
Her uncle and grandmother also had diabetes mellitus D
- A mother is concerned regarding her baby who has developed
fractures which appear to occur with minimal trauma. He has
blue sclera. B
- An 18-year old female underwent caries tooth extraction and
developed profuse bleeding. On history she revealed
menorrhagia. Her mother and her grandfather had the same
disease. G
• EMQ
• 1)
PGH2
• 2)PGG2
PGE2
PGF2alpha
Arachidonic acid
Calcium
Magnesium
• Q1) Synthesis of prostaglandins by?
• Q2) the first Prostaglandin produces?
• Q3) Oxytocin uses this as its intermediate precursor?
• 2) Shapes of the pelvic
• Round shaped pelvic brim with transverse
diameter more than AP
• Round shaped pelvic brim with AP diameter more
than the transverse diameter
• C)Oval shaped pelvic brim with transverse diameter more than AP
• Oval shaped pelvic brim with AP diameter more
than the transverse diameter
• Heart shaped pelvic brim with transverse diameter
more than AP
• Heart shaped pelvic brim with AP diameter more
than the transverse diameter
• Q1) Gynecoid
• Q2) Android
• 3) DNA and RNA
• DNA—Deoxyadenine------Double standed
• Deoxythymine
• Deoxycytosine
• Deoxyguanine
• DNA—Deoxyadenosine------Single standed
• Deoxytyrosine
• Deoxycytosine
• Deoxyguanine
• RNA—Adenine-----Double stranded
• Thymine
• Cytosine
• Uracil
• ) RNA—Adenine-----Single stranded
• Thymine
• Cytosine
• Uracil
• Q1) Messenger RNA?
• Q2) DNA?
• Q3) Human papilloma virus?
Separated recalled items:
• Hormone increased in Prolactinaemia
• Hormone produced by posterior pituatary
• Baby born with right sided umbilical hernia
• Baby born with stiff limbs and mother had oligohydramniosis
• Atomic mass of technetium (99)
• Atomic mass of cobalt (58, but the nearest possible on the list 60)
• Origins of pudendal nerve- ant s2 s4
• Functions of 21 and 17a hydroxlase (convery what to what)
• POstitive predictive value and negative predictive value
• Linear regression
• Hep B and Hep C- DNA/ RNA viruses
• Lidocaine- side effects, shorter that bupivicaine
• Anatomy of pelvic bones
• Fetal skull
• Arterial supply of anus
• Lasers- ? red light faster that UV, ? cut with red light
• MRI- not radioactive, movement of H ions, resolution
• Matching disease with organism- chancre,
• Cancers with serum markers
• Amyloidosis- ?RA ?intracellular
• Bronchodilators
• Staph aureus- coagulase, toxin
• Time closure of neural tube, time mid gut move from abdo wall
• origin of pudendal nerve
• nerve supply of the skin of perinium
• the structure that can differentiate between body of uterus and cervix
in c/s
• struture must be identified during c/s to avoid injury
• embryology: by this time if the mid gut not withdrawl to abdomen the
fetus will devolp exomphilus
• by this time the an and post neroun was closed ,they give many
options
• obstructive jaunduce.
• histopathology of endometrium, hyperplasia & adenocarcinoma.
• HIV, HBV.
• selective COX 2 inhibitors & antiprogesterone.
• steroidogenesis pathways & enzymes.
• tumour markers.
• sarcoidisis.
• asbestosis.
• PUDENDAL NERVE ROOT VALUE S1,2,3 ANT PRIMARY RAMI
• N SUPPLY OF PERIANAL AREA
• LENGTH OF UMBILICAL CORD
• AUTOSOMAL RECESSIVE TAY SACH'S DISEASE
• AUTOSOMAL DOMINANT ACHONDROPLASIA
• ANTIINFLAMMATORY MUSCLE RELAXANT PROSTAGLANDIN
• STAPH AUREUS COAGULASE POSITIVE, PIGMENTED COLONY,GRAM
POSITIVE
• GENTAMYCIN BACTERICIDAL,TOXIC TO 8 TH NERVE
DIURETIC ACTING ON DCT BENDROFLUAZIDE
MCQ questions that actually appeared on MRCOG
part 1exams between 2005-2009.
1) The pelvic surface of the sacrum?
a) gives origin to the piriform muscles
b) gives origin to the levator ani muscles
c) is broader in the male than in the female
d) trnasmit dorsal rami of sacral nerves
e) is in contact with the anal canal
2)Prolactin.
a) release is stimulated by TRH
b) plasma levels are raised in the first trim of preg.
c) release is increased by suckling.
d) maybe produced by decidua.
e) release is inhibited by metoclopromide.
3) The foll disorders and org are correctly paired.
a) opthalmia neonatorum: chlamydia trachomatis
b) chancroid: Haemophilus ducreyi.
c) sleeping sickness: Leishmania donovani.
d) ringworm: Trichenella spiralis.
e) non-specific urethritis: Toxoplasma gondii.
4) The heart rate typically increases in response to:
a) pain.
b) hypoxia.
c)ventilatory expiration
d) increase in Intracranial pressure
e) decrease baroreceptor activity
5) Antibodies.
a) are proteins.
b) are formed in the fetus before 12 weeks of Intrauterine life.
c)have an average molecular weight of around 10 000 daltons.
d) of the rhesus type are genetically transmitted.
e) are produced by the ribosome of plasma cells.
6) the foll disorders have an X linked pattern of inheritance.
a) G6PD Deficeincy.
b) Kleinfelter syndrome
c) adrenogenital syndrome
d) haemophilia B
e) familial hypercholestroleamia.
7)The foll are derived from the urogenital sinus:
a) the bladder trigone.
b) the ureters.
c) the female urethra.
d)greater vestibular glands.
e) paraurethral glands.
In the fetal CVS
a) the heart arises from endoderm
b) the heart is formed by fusion of endocardial tubes.
c) Cardiac pulsation is present by the 30th day after fertilization.
d) oxygenated blood is tranferred to the left atrium through the foramen
ovale.
e) the ductus arteriosus closes during the last 4 weeks of pregnancy.
9)Arginine vasopressin
a) reduces GFR.
b) controls water loss in the Proximal renal tubule.
c) is synthesised by the post pituitary gland.
d) is released in response to rise in plasma osmolality.
e) is released in response to fall in circulating plasma volume.
10) Renin
a) is secreted by the zona glomerulosa of the adrenal coretx.
b) is a proteolytic enzyme.
c) is secreted at an increased rate if the renal perfusion pressure falls.
d) acts upon circulating angiotensinogen.
e) is released in response to an increase in extracellular fluid volume
11) Actinomyces israelii
a) is a rickettsia.
b) forms yellow granules in pus.
c) is a commensal in the mouth.
d) is a commensal in the vagina.
e) is usually resistant to penicillin.
12) Diseases caused by spirochaetes include.
a) Weil's disease.
b) lymphogranuloma venereum
c) pinta
d) Vincent's angina.
e) bilharzia
13) The foll are inherited as autosomal recessive conditions:
a) tuberous sclerosis.
b) phenylketonuria.
c) achondroplasia
d) sickle cell anaemia.
e) Von gierke's disease.
14) Antibodies play an important part in the development of:
a) phagocytosis.
b) Mantoux responce.
c) erythroblastosis fetalis.
d) hyperemesis gravidarum
e) anaphylaxis
15) chemical mediators concerned in the production of an inflammatory
response include:
a) 5-hydroxytryptamine
b) aldosterone.
c) glucocorticoids.
d) bradykinin
e) leukotreines
16) The parathyroid gland.
a) originate from the pharyngeal cleft ectoderm.
b) secrete parathyroid hormone via the chief( principal) cells.
c) secrete calcitonin via the oxyphil cells.
d) may become hyperplastic in the presence of intestinal malabsorption.
e) may develop adenomas in association with islet cell tumour of the
pancreas
17) In the abdominal wall:
a) the rectus abdominis muscle is attched to the crest of the pubis.
b) the post border of the external oblique muscle ends in the linea
semilunaris.
c) the aponeurosis of the external oblique muscle takes part in the
formation of the conjoint tendon.
d) the inferior epigastric artery is a branch of the internal iliac artery.
e) the conjoint tendon blends medially with the anterior layer of the rectus
sheath.
1 The urogenital sinus in the female gives rise to the following:
a) ureter
b) paraurethral glands
c) Bartholin's gland
d) urachus
e)Gartner's duct.
19) Concerning sex hormone:
a) the ovary secretes androstenedione.
b) The ovary secretes testosterone
c) The ovary secretes dihydrotestosterone.
d) SHBG conc. are higher in women more than men
e) Androgens bound to protein have high biological activity.
20) Features of congenital rubella include:
a) excretion of virus by the neonates.
b) hepatomegaly
c) excessive production of growth hormone.
d) cataract
e) deafness.
21) Clomifene citrate:
a) is an anti-androgens.
b) does not stimulate ovulation directly.
c) can produce visual disturbance.
d) is genereally prescribed throughtout the proliferative phase of the
menstrual cycle.
e) in the treatment of ovulation increases the risk of multiple pregnancy.
22) The foll are cytotoxic alkylation agents:
a) Cyclophosphamide.
b) mercaptopurine
c) chlorambucil
d) fluorouracil
e) methotrexate.
Most of the statistic questions posted are right. so will post only the ones
which are missing from wht i remember.
23) If a distribution of results is markedly skewed to the left:
a) the mean is the same as the 50th centile.
b) the same number of values lie on either side of the median.
c) the mode is equal to the median.
d) the student's t test should be used to compare the distribution with
another.
e) logarithmic transformation of the result will produce a distribution
closer to the normal.
24) Corcerning the analysis of clinical trials:
a) the 95% confidence interval indicates the range within which 19 out of
20 values will lie.
b) The P value illustrates how often the result would be expected to occur
by chance.
c)b The conventional level of statistical significance is set of P<0.005
d) In a randomised trial, there must be equal numbers of results in each
arm of the study.
e) A relative risk reduction of 60% is significant irrespective of the value
of P.
25) The following substance are normally synthesized in the liver:
a) glucagon
b) vitamin A
c) cholesterol.
d) immunoglobulins
e) prothrombin
26) The pineal gland:
a) is situated at the anterior end of the 3rd ventricles.
b) is innervated by the parasymphathetic nervous system.
c)produces melatonin.
d)maybe calcified in the adults.
e) is most active during daylight.
27)In congenital adrenal cortical hyperplasia.
a) The commonest deficiency is C18 hydroxylase.
b) plasma cortisol concentration is raised.
c) urinary excretion of 17 oxysteroids is elevated.
d) dexamethasone will suppress the urinary excretion of 17 oxysteroids.
e) there is no virilising effects.
28)Foll are RNA containing virus:
a) coxsackie
b) influneza
c) mumps
d) herpes simplex.
e) cytomegalovirus.
29) Listeria monocytogenes:
a) is a gram negative organism.
b) is sensitive to ampicillin.
c) may cause a transplacental infection.
d) is sexually transmitted.
e) can be cultured from a high vagina swab.
30) actinomyces israelii.
a)is a fungus.
b) forms yellow granules in pus.
c)is a mouth commensal
d) occurs in association with IUCD.
e) is resistant to penicillin.
31) The following drugs may cause enlargement of the fetal thyroid
gland:
a)methyldopa
b) thyroxine
c) carbimazole
d) propranolol
e) propylthiouracil
32) The foll statements about anticoagulant are correct:
a)Heparin inhibits the action of thrombin
b) The action of heparin is antagonised by vitamin K
c) Heparin increases antithrombin III activity.
d) The effects of anticoagulants are decreased by metronidazole.
e) Warfarin is greater than 80% protein bound in plasma
33) The following drugs and side effects are associated:
a) methydopa: depression
b) paracetamol: thromboembolism
c) indomethacin: peptic ulcer
d) prednisolone: osteoporosis.
e) ritodrine: hypoglycemia
34) Haematopoiesis in the fetus:
a) results in nucleated erythrocytes early in development.
b) occurs in the yolk sac in the first month.
c) does not occur in the bone marrow until term.
d) is predominantly hepatic during the 4th month.
e) does not require folic acid.
35)Early blood borne dissemination is characterised feature of:
a) carcinoma of the endometrium.
b) osteosarcoma
c) basal cell carcinoma
d) carcinoma of the cervix
e) choriocarcinoma.
36) The following cells maybe phagocytic:
a) neutrophils.
b) kupffer cells
c) monocytes
d) Hofbauer cells.
e) plasma cell.
37) The following are premalignant conditions:
a) diverticular disease of the large bowel.
b) ulcerative colitis.
c) pulmonary asbestosis.
d) Paget's disease of the bone.
e) condylomata of the vulva
3 In the pituitary gland:
a) the anterior lobe is smaller than the posterior lobe.
b) the posterior lobe is ectodermal in origin.
c) the acidophil cells produces oxytocin.
d) the basophil cells produce growth hormone.
e) the blood supply is derived from the internal carotis artery.
39)The obturator artery:
a)branches from the posterior trunk of the internal iliac artery.
b) passes through the greater sciatic foramen.
c) is crossed by the ureter.
d)supplies the hip joint.
e) may be replaced by a branch of the superior epigastric artery.
40) In congenital adrenal hyperplasia:
a) the commonest cause is a deficiency of 21 hydroxylase.
b) the plasma cortisol conc is increased.
c) there may be excessive secretion of 17 alpha hydroxyprogesterone.
d) sodium retention is characteristic.
e)blood cathecholamine conc are increased.
41) In the fetal lung:
a)bronchial cartilage formation commences at 18-24 weeks of gestation.
b) type II alveolar cells first appear at 16-20 weeks gestation.
c) sphingomyelin is the most common phospholipid present at term
d) phospholipid release is increased by endogenous adrenaline.
e) phospholipid production is decreased by exogenous corticosteroids.
42) In radiotherapy
a) 1 gray is equivalent to 1 joule/kg.
b) the skin usually receives a greater dose of radiation than the underlying
tissues.
c) the major effect of radiation energy is to damage the cytoplasm of the
cell.
d)cells in tissues which are hypoxic are more vulnerable to radiation.
e)Radiation induced changes in tissues may take 6 weeks to develop.
43) Concerning the adrenal glands
a) cortex is derived from neural crest cells.
b) Zona fasiculata secretes aldosterone.
c) Cortical adenomas may cause Cushing syndrome.
d) Neuroblastoma arise in the medulla.
e) Addison's disease may result from autoimmune destruction of the
cortex.
44)The following are autosomal recessive:
a) neurofibromatosis.
b) cystic fibrosis.
c) phenylketonuria
d) polyposis coli
e) sickle cell anaemia
45) Uterine fibroids:
a) are defines histologically as fibromyxomas.
b) arise from endometrial stroma
c) maybe associated with polycythamia.
d) predispose to endometrial hyperplasia.
e) are liable to sarcomatous change in about 5% of cases.
46) growth of the foll tumors are hormone dependent:
a) squamous cell carcinoma of the cervix.
b) breast adenocarcinoma.
c) uterine leiomyoma.
d) prostatic adenocarcinoma
e) testicular carcinoma
47)Surfactants:
a) is formed mainly in the placenta
b) levels in amniotic fluid diminish after 33 weeks of gestation
c) formation can be inferred from the lecithin-sphingomyelin ratio in
amniotic fluid.
d) contains palmitic acid
e) decreases the surface tension in pulmonary alveoli.
4 Intracellular fluid differs from Extracellular fluids in that:
a) it forms the major proportion of total body water.
b) its volume can be measured easily.
c) it has a higher concentration of potassium than of sodium.
d) its volume is regulated primarily by the kidneys.
e) it has a higher phosphate concentration.
49) The Anal canal:
a) has an upper part which is innervated by the inferior hypogastric
plexus.
b) has a lower part which is supplied by the superior rectal artery
c) drains lymph to the superficial inguinal nodes from its upperpart.
d) has its internal sphincter innervated by the infecrior rectal nerve.
e) has a superficial part of its external sphincter attached to the coccyx.
50) The right ovarian artery:
a) Arises from the abdominal aorta above the renal artery.
b) passes posterior to the 3rd (horizontal) part of the duodenum
c) passes post to the genitofemoral nerve.
d) supplies the right ureter.
e) anastomoses with the right uterine artery.
51) Vulva supplied by:
a) Internal pudendal artery.
b) Inferior rectal artery.
c) Genitofemoral artery
d) obturator artery
e) femoral artery..
This is a question outside past papers..All r false Except internal pudendal
artery..
52) The vagina:
a) has an anterior wall longer than the post wall.
b) contains mucus secreting glands in its epithelium.
c) is related in its lower third to the bladder base.
d) during reproductive life has an acid pH.
e) is derived from mesonephric duct.
53) The right ureter:
a) is approx 50cm in length.
b) ia partly covered by duodenum
c) crosses the genitofemoral nerve.
d) enters the bladder anteromedially.
e) receives part of its blood supply from the uterine artery.
54) After birth:
a) allantois froms median umblical ligament
b) umbilical vein forms medial umbilical ligament.
c) umbilical artery forms superior vesical artery.
d) ductus venosus forms the ligamentum teres.
e) ductus arteriosus forms the arch of the aorta.
55)In spermiogenesis:
a) primary spermatocytes undergo reduction division.
b) primary spermatocyte gives rise to 4 spermatids.
c) whole process of spermatogenesis in man takes 6-7 days.
d) grossly abnormal spermatozoa may be found in fertile semen.
e) spermatids are haploid.
56) In the human male, dihydrotestosterone:
a) is a precursor of testosterone.
b) has one-tenth of the ptency of testosterone.
c) is responsible for involution of the Mullerain system.
d) is responsible of the male external genitalia.
e) binds to an intracellular receptor.
57) In Human lactation:
a) estrogens promote development of breast lobules.
b) estrogen promotes milk producing effect of prolactin on the brest,
c) human placental lactogen is essential for milk synthesis.
d) prolactin stimualtes gonadotrophin release.
e) oxytocin causes milk ejection,
58)Concerning testicular hormones:
a) testosterone reduces plasma LH conc.
b) Inhibin stimulates LH production.
c) Estrogen are formed in the testis.
d) Testosterone is converted to dihyrotestosterone by 5 alpha reductase.
e) Testosterone in plasma is predominantly bound to albumin.
59) Epidermal growth factor:
a) is mitogenic.
b) synthesis is stimulated by estradiol.
c) is a steroid molecule.
d) is found in endometrium.
e) binds to a receptor on the nuclear membrane.
60) folic acid:
a) deficiency causes megaloblastic bone marrow..
b) is hydroxycobalamin.
c) is present in green vegetables.
d) is predominantly absorbed from the large intestine.
e) is destroyed by boiling water.
61) Doppler Ultrasound:
a) is used to monitor fetal breathing.
b) is used in fetal HR monitors.
c) can be used to measure blood velocity in the fetus.
d) measure proton relaxation times.
e) requires injection of contrast agents.
62) The following are structural abberation of chromosomes:
a) deletions.
b) inversions.
c) aneuploidy.
d) polyploidy.
e) translocation.
63) Messenger RNA
a) synthesis is dependant on RNA polymerase.
b) is an exact copy of sense DNA.
c) contains exons.
d) is measured by western analysis.
e) translationoccurs in the nucleus.
64) In tumors of the bones:
a) primary malignancy is more common than 2ndry malignancy.
b) osteoma rarely present in skull bones.
c) osteosarcoma is associated with Paget's disease of bone.
d) lymph node metastases are unusual.
e) simple bone cysts have a strong tendency to recur.
Questions about
• endometrial hyperplasia
• side effect of drug (carboplatin)-(taxal)
• arias stella if its specific in pregnancy
• chlamydia
• varicella
• hpv ,wart
• hcg concentration in pregnancy
• citric acid cycle diagram
• hormones diagram levels of, fsh,lh,estrogen,testosterone
• by transvaginal u/s yolk sac appear in which week
• genetic about pcr, fish test
• laser
• totipotent cell,tumour like mass
• voiding presure
• ca requirement in pregnancy
• test used to diagnose folic acid deficency
• thalasthmia
• cervical ectropion cells are columnar cell
• mri contraindicated in early pregnancy
• stellate instability in dna
• transverse abdominus ms attached to lumbar transverse process
• cut s1 lead to autonomic bladder
• cancer which are hormone dependent
-breast
-prostate
-thyroid
• guthre test
• cystic fibrosis and diagnosis
-swet test -karyotyping-fish test
• disinfection and sterilization
• complication of contraceptive pills
• innervation of all pelvic organs.
• lining of ureter
• erythropoeitin and renin q from john duthie
• complications of cisplatin paclitaxel
• statistics -std error calculation mean in a normal distribution
• occipito frontal diameter
• hcg titre at detection by tv u/s 10 at 3 days after fertilization
• crl in scan is 60mm what is gestational age ,crl at term ?
• parietal suture is between
• kuffer cells are –phagocytic
• estradiol receptors
• progestrone receptors -we know that these are intra nuclear but
there were confusing options with intranuclear , one option had
intranuclear kinase .
• anticonvulsant with description of phenytoin
• vitamin deficiency causing hyperemesis
• macrocytic anemia
• xerophtalmia
• cystic fibrosis
• complement causes
• b thalasemia detection
• folate def detection
• basal cell ca - local malignant i think
• tumor like –hamaroma
• cystic fibrosis guthic spot test
• receptors in apoptosis caspa
• arias stella reaction
• blood picture in PET and in preg is inflam like
• in statistics mostly repeated questions..example given..calculate
sensitivity,specificity,+ve predictive value..in right sided skewed
distribution curve median will be on which side of mean?
• microsatellites alleles
- cd4
-nk cells
• down syndrome
• pulmonary embolism
• apoptosis or endometrial receptors.
• anal canal
• vagina
• ovary
• Phenylketonuria
• inhibin
• passive transport
• erythropoietin
• renin
• hcg levels
• syphylis
• rubella
• hormone dependent malignancies(testicular carc._sertoli leydig-
clear cellcarc.ofkidney-thyroid carc.)
• pigd
• congenital heart
• ring Y chromosome
• passive diffussion not depend on .
• concentration gradient
• molecular size
• following tissue are capable of regeneration
spinal cord
liver
epidermis
myocardium
bone marrow
• double blind trial
• level of HCG in the urine pregnancy test
• APOPTOSIS
• methods of disinfection and
• BOWIE DICK test
• progesterone receptors
• syphilis , toxoplasma , hpv , immune responses
• down syndrome
associated with duodenal atresia true
associated with ambig genetalia false
only maternal chromosome F
only paternal chromosome F
• oogonia
• miosis starts at puberty
• miosis before mitosis
• during S phase chromosomes are doubled.
• to calculate sensitivity and specificity.
• standard deviation
• standard error of the mean.
• mean, mode median in normal distribution and skewed data.
• T test
• MRI... type of radiation,
causes effect in pregnancy
women should be placed left tilted during scan.
• dose of radiation
. 1 gray = joules/ kg
. absorbed dose.
• dopplers USG
•
EMQ questions that actually appeared on MRCOG
part 1exams between 2007-2009.
EMQ about
• vitamins (hyperemesis), vit B6 and (xerophthalmia).
• paclitaxel carboplatin side effects
• complex partial seiizure gum hypertrophy,acne ,facial
coarsening,vit K TO BE given..?drug used
• oestrogen progesterone receptor site
• vitamins-xerophthalmia..morning sickness..macrocytic anaemia.
• rubella incubation period,specific immunity within 15 days..
• HEPATOCELLULAR carcinoma,..??
• lining epithelium of ureter,..?
• mantoux rection..
• ovulatory DUB
• aneurysm- 10% due 2 inflam.,syphilitic aneu,thoracic
vessels,marfan recessive.
• if a mother has a child with cystic fibrosis ,she is pregnant and will
do amniocentesis
a. which test to be done (every possibility mentioned)
b. all relatives will volunteer to do gutic spot test ,which test you
will carry on the blood .
• crown heel lenghth at birth
crown/rump length 6 cm =? Weeks
• type of hpv causing benign wart
type of hpv causing cancer cx
• .B cell
.plasma cell
.dendritic cell
.mast cells
. t cell
QUESTION
. ANTIBODY PRODUCTION?
. ANTIGEN PRESENTATION?
• Options
1. oval inlet transversal diameter longer than anterposterior .
2. oval inlet anterposterior dia longer than trans.
3. heart shape inlet anterposterior dia. longer than trans.
4. heart shape inlet trans. diam. longer than anterposterior
5. rounded inlet .. T>A.
6. rounded inlet ..A > T
Qus.
a. Gynaecoid pelvis.
b. Android pelvis .
------------------------------------------------------------
a. Adenine monophosphate
Guanine monophosphate
Cytosine monophosphate duple helix
Thymine monophosphate
b. Adenine monophosphate
Guanine monophosphate
Cytosine monophosphate duple helix
Uracil monophosphate
c. Adenine monophosphate
Guanine monophosphate
Cytosine monophosphate single helix
Uracil monophosphate
d. Adenine monophosphate
Guanine monophosphate
Cytosine monophosphate single helix
Thymine monophosphate
e. Adenine monophosphate
Guanine monophosphate
Cytosine monophosphate duple helix
Thymine monophosphate
f. Diadenine monophosphate
Diguanine monophosphate
Dicytosine monophosphate duple helix
Diuracil monophosphate
c. DiAdenine monophosphate
DIGuanine monophosphate
DICytosine monophosphate single helix
DIUracil monophosphate
d. DIAdenine monophosphate
DIGuanine monophosphate
DICytosine monophosphate single helix
DIThymine monophosphate
e. Adenine monophosphate
Guanine monophosphate
Cytosine monophosphate duple helix
Thymine monophosphate
b. Adenine monophosphate
Guanine monophosphate
Cytosine monophosphate duple helix
Uracil monophosphate
c. Adenine monophosphate
Guanine monophosphate
Cytosine monophosphate single helix
Uracil monophosphate
d. Adenine monophosphate
Guanine monophosphate
Cytosine monophosphate single helix
Thymine monophosphate
1. DNA.
2. mRNA
3. Nucliac acid o HPV.
• Options
Somatotroph, acidophil
Somatotroph, basophil
lactotroph, acidophil
lactotroph, basophil
corticotroph, acidophil
corticotroph, basophil
gonadotroph, acidophil
gonadotroph, basophil
thyrotroph, acidophil
thyrotroph, basophil
Q1. Growth hormone
Q2. Prolactin
number 1:
a. pelvic brim oval in shape, transverse diameter more than anteropost
diameter.
b. pelvic brim oval in shape, anteropost diameter more than transverse
diameter.
c. pelvic brim heart shaped, transverse diameter more than anteropost
diameter.
d. pelvic briim heart shaped, anteropost diameter more than transverse
diameter.
e. pelvic brim circular, transverse diameter more than anteropost
diameter.
f. pelvic brim circular, anteroposr diameter more than transverse.
1) gynecoid pelvis.
2) android pelvis.
number 2:
a.double stranded, adenosine monophosphate, guanine monophosphate,
cytosine monophosphate, uracil monophosphate.
b. double stranded, adenosine monoph, guanine monph, cytosine
monoph, thymine monoph.
c. single stranded, adenine monoph, guanine monoph, cytosine monoph,
thymine monoph.
d. single stranded, adenine monoph, guanine monoph, cytosine monoph,
uracil monoph.
e. single stranded, deoxyadenine monoph, deoxyguanine monoph,
deoxycytosine monoph, deoxyuracil monoph.
f. single stranded, deoxyadenine monoph, deoxyguanine monoph,
deoxycytosine monoph, deoxythymine monoph.
g. double stranded, deoxyadenine monoph, deoxyguanine monoph,
deoxycytosine monoph, deoxyuracil monoph
h. double stranded, deoxyadenine monoph, deoxyguanine monoph,
deoxycytosine monoph, deoxythymine monoph,
1) DNA.
2) RNA.
3) genome of HPV.
number 3:
a. edwards syndrome.
b. exomphalus.
c. gastroschiasis.
d. patau's syndrome.
e. down's syndrome.
f. kleinfeltr's syndrome.
1) abnormality in the anterior abdominal wall, usually to the right of the
umbilicus, other genetic abnormalities rarely associated.
2) abnormality in chromosome 18.
number 4:
a. azithromycin.
b. amoxycillin.
c. cefuroxime.
d. benzylpenicillin.
e. metronidazole.
1) best treatment of acute upper UTI in third trimester.
2)treatment of chlamydia in a non-pregnant woman.
number 5:
a. L1,2,3
b. L1
c. L2,3,4
d. L1,2
e. L3,4,5
1) ilioinguinal nerve.
2) genitofemoral nerve.
number 6:
a. streptococus, aerobic, G+ve.
b. streptococus, anaerobic, G+ve.
c. staphylococus, aerobic, G+ve.
d. staphlococus, anaeobic, G-ve.
e. pseudomonas, aerobic, G+ve.
f. pseudomonas, aerobic G-ve.
g.pseudomonas, anaerobic, G-ve.
1) toxic shock syndrome.
2) hospital cross infection.
number 7:
a. CMV.
b. listeria monocytogens.
c. treponame pallidum.
d. HIV.
e. human leucocytic virus.
f. staphlococus aureus.
g. streptococus.
h. varicella zoster.
1) a pregnant woman developed a "flue-like" illness with fever and
general malaise, her baby was born with hepatosplenomegaly and
jaundice.
2) a woman has a history of 2 previous stillbirths, is now pregnant and at
37 weeks complained of fever and rash, her son was born healthy but at
age of one year he had abnormal incisors and later deafness.
number 8:
a. hepatitis C, DNA.
b. hepatitis C, RNA.
c. herpes simplex.
d. HPV.
e. i cant recall the other options.
1) associated with high incidence of hepatocellular carcinoma.
2) papular skin rash.
number 9:
a. estrogen.
b. progesterone.
c. DHEA-sulphate.
d. cortisol.
e. cortisone.
1) hormone maintains uterine quisence throughout pregnancy.
2) hormone produced by the placenta and fetal adrenal gland, promotes
fetal lung maturation.
number 10:
a. allantois.
b. cloaca.
c. mesonephros.
d. yolk sac.
1) rectum develops from the posterior portion if this structure.
2) germ cells arise from it.
number 11:
a. B cell
b. T CD4 cell
c. T CD8 cell
d. dendritic dells.
1) major antigan presenting cell.
2) could not recall it.
EMQ
1)
1) PGH2
2)PGG2
3) PGE2
4) PGF2alpha
5) Arachidonic acid
6) Calcium
7) Magnesium
Q1) Synthesis of prostaglandins
Q2) the first Prostaglandin produces.
Q3) Oxytocin uses this as its intermediate precursor
2) Shapes of the pelvic
A) Round shaped pelvic brim with transverse diameter more than AP
B) Round shaped pelvic brim with AP diameter more than the transverse
diameter
C)Oval shaped pelvic brim with transverse diameter more than AP
D) Oval shaped pelvic brim with AP diameter more than the transverse
diameter
E) Heart shaped pelvic brim with transverse diameter more than AP
F) Heart shaped pelvic brim with AP diameter more than the transverse
diameter
Q1) Gynecoid
Q2) Android
3) DNA and RNA
A) DNA—Deoxyadenine------Double standed
Deoxythymine
Deoxycytosine
Deoxyguanine
B) DNA—Deoxyadenosine------Single standed
Deoxytyrosine
Deoxycytosine
Deoxyguanine
C) RNA—Adenine-----Double stranded
Thymine
Cytosine
Uracil
D) ) RNA—Adenine-----Single stranded
Thymine
Cytosine
Uracil
Q1) Messenger RNA
Q2) DNA
Q3) Human papilloma virus
4)
1) L1
2) L1, L2
3) L1, L2, L3
4) L1, L2, L3, L4
5) L2, L3, L4
6) L3, L4
Q1) Ilioinguinal
Q2) Genitofemoral.
) Given in a single dose
A) AZITHROMYCIN
B) Amoxicillin
C) Cephalosporin
D) Vancomycin
E) Gentamycin
F) Benzylpenicillin
Q1) A female with a chlamydial infection, and non-pregnant, which is the
best medication?
Q2) A female presents with acute upper UTI?
6)Infections
A) Staph
B) Strept
C) Pseudomonas aeruginosa
Q1) most common cause wound infection in hospital
Q2) hospital acquired infection
7) Statistics
Another question on calculation for specificity and sensitivity.. People
preparing for March 2010, please learn the formulas well.
8-THE FOLLOWING STATEMENTS ABOUT THE ADRENAL
GLAND ARE CORRECT
a-they lie anterior to the diaphragm ...........True.
b-the left adrenal gland lies behind the pancreas.....True.
C-lymphatic drainage is to the superficial inguinal nodes.....False
d-the adrenal cortex contains chromaffin cells .........False.
e-the adrenal medulla is derived from mesoderm ....False
the ovary
a- is attached to the ant surface of the broad ligament
b- lies on the genitofemoral nerve
c- lies in the angle between the ureter and the external iliac vessel
d- - has visceral afferent fibres from the pelvic splanchnic nerve
e- has lymphatic drainage to the superficial inguinal lymph nodes
concerning the uterus
a- it is formed from the mesonephric duct
b- it has a lymphatic drainage in part to the inguinal glands
c- the uterine artery passes below the ureter
d- the uterine vein communicate with the vesical plexus of veins
e-pain sensation from the body of uterus is carried by the pelvic
splanchnic nerve
about this stem am not sure if it was like that in the exam
Obturator artery
a- branches from the posterior trunk of the internal iliac artery
b- passes through the greater sciatic foramen
c- is crossed by the ureter
d- supplies the hip joint
e- may be replaced by a branch of the superior epigastric artery
The external iliac artery
a-enters the thigh anterior to the inguinal ligament
b- at its origin is crossed by ureter
c- at its origin is crossed by ovarian vessels
d-lies medial to the external iliac vein at its distal end
e-give rise to the deep external pudendal artery .
The pelvic splanchnic nerves..
A- Are derived from the posterior rami of the sacral spinal nerves
B- Supply afferent fibres
C- Unite with branches of the synpathetic pelvic plexus
D- Supply the ascending colon with motor fibres
E- Supply the uterus with parasympathetic fibres
In the anterior abdominal wall...
A- Rectal muscle is intersected tranversely by three bands
B- The posterior rectus sheath below the arcuate line consists of
transversalis fascia only
C- Above the costal margin the posterior rectus sheath is deficient
D- The superior epigastric artery arises from the internal thoracic artery.
E- The inferior epigastric artery arises from the femoral artery
The pelvic surface of the sacrum.
A- Gives origin to the piriforms muscle
B- Gives origin to the levator ani muscle
C- Is broader in the male than in the female
D- Transmits the dorsal remi of sacral nerves
E- Is in contact with the anal canal.
The rectum
A- Is supplied in part by the inferior rectal artery
B- Is innervated by the inferior rectal nerve
C- Is lined by stratified squamous epithelium
D- Has its lymphatic drainage to the superficial inguinal nodes
E- Possesse a complete layer of longitudinal muscle
The Spleen
A- has a notched posterior border
B- lies in front of the costo-diaphragmatic recess
C- is in contact with the body of the pancrease
D- lies under the cover of the 9th to the 11th ribs
E- is innervated from the renal plexus
THE VULVA
a-internal pudendal nerve
b-anterior cutaneous of thigh
c-inferior rectal
d-illioinguinal
e-obturator
the pineal gland
a-is situated at the anterior end of the third ventricle
b-is innervated by parasympathetic nervous system
c-produce melatonin
d-may be calcified in the adult
e-is most active during daylight
screening test for following
1- syphilis treponemal antibody test
2- syphilis hemaglutination test
3- syphilis immobilization test
4- HB-electrophorisis
5- mcv
6- mchc
7- MCh
8- serum iron
9- paul-bennel test
Q1) Thalassemia
Q2) Syphilis
Sep. 2009
Options
1. oval inlet transversal diameter longer than anterposterior .
2. oval inlet anterposterior dia longer than trans.
3. heart shape inlet anterposterior dia. longer than trans.
4. heart shape inlet trans. diam. longer than anterposterior
5. rounded inlet .. T>A.
6. rounded inlet ..A > T
Qus.
a. Gynaecoid pelvis.
b. Android pelvis .
------------------------------------------------------------
a. Adenine monophosphate
Guanine monophosphate
Cytosine monophosphate duple helix
Thymine monophosphate
b. Adenine monophosphate
Guanine monophosphate
Cytosine monophosphate duple helix
Uracil monophosphate
c. Adenine monophosphate
Guanine monophosphate
Cytosine monophosphate single helix
Uracil monophosphate
d. Adenine monophosphate
Guanine monophosphate
Cytosine monophosphate single helix
Thymine monophosphate
e. Diadenine monophosphate
Diguanine monophosphate
Dicytosine monophosphate duple helix
Diuracil monophosphate
f. Diadenine monophosphate
Diguanine monophosphate
Dicytosine monophosphate duple helix
Dithymine monophosphate
g. DiAdenine monophosphate
DIGuanine monophosphate
DICytosine monophosphate single helix
DIUracil monophosphate
h. DIAdenine monophosphate
DIGuanine monophosphate
DICytosine monophosphate single helix
DIThymine monophosphate
1. DNA.
2. mRNA
3. Nucliac acid o HPV.
STRUCTURE OF BASES AND NOT THE NUCLEOTIDE OR
NUCLEOSIDES
gynecoid and android pelvis and the DNA
1)
a) ant division of ant. remi of S2-4
b) ant division of post.rami of S2-4
b)genitofemoral nerve
c) post division of ant. remi of S2-4
d) post division of post. remi of S2-4
e) sensory supply of perineal nerve
f) inferior rectal nerve
g) ischial tuberosity
h)ischial spine
A lady is about to deliver and you are about to give her a pudendal
block..
Q1- what is the root value of pudendal nerve?
Q2- what nerve supplies lower part of vagina (I think)?
Q3- why do u give local skin infiltration before episiotomy?
Q4- or what is the nerve supply of peranal area .?
(2).. If we take the day of fertilization as Day’0’ then…
Day2
Day4
Day 8
Day 10
Day12
Day 14
Day 18
Day20
Day 22
Day 24
Day 26
Day 42 [
Day 70
Q1- Which represents 4 cell stage ?
Q2- conceptus implants completely?
Q3- fetal heart pulse seen on ultrasound?
Q4- vertebra form completely?
(3)
A Cytoplasmic transcription factor receptor
B G protein coupled receptor on cell membrane
C G protein coupled receptor on Golgi complex
D Mega subunit ligand gated ion channel
E Multisubunit antibody receptor on cell membrane
F Multisubunit ligand gated ion channel on cell membrane
G Nuclear protein kinase receptor
H Nuclear transcription factor receptor
I Protein kinase receptor on cell membrane
J Receptor protein complex (intracytoplasmic)
K Transcription factor receptor on cell membrane
Select the receptor which binds the molecules referred to in the
itemS below. Each option may be used once, more than once or not
at all.
1- insulin .
2- progesterone .
3- ostradiol .
4- prostaglandin .
(4)
PCR
Fish test
Chromosomal linkage analysis
Telomer analysis
Sum hereditary crosslinkage chromosomal option (dnt remember)
Screen for some known gene mutation
Sweat test
Saliva test
-A family with one kid presumed to have cystic fibrosis however
cftr gene mutation was not detected. Family members willing to
give samples if needed.mother wants definite diagnosis as she is
11wks and wants to know diagnosis for her baby.parents are
cousins with 1st degree relatives having cystic fibrosis.
1- Invasive diagnostics can be applied to reach final results by?
2- you have Guthrie spot of their son how will u diagnose him for
cystic fibrosis?
3- families in UK with cystic fibrosis how will u find f508?
(5)
A- Endoplasmic reticulum
B- Golgi apparatus
C- Lysosomes
D- Microtubules
E- Mitochondria
F- Nucleolus
G- Plasma membrane
H- Ribosomes
Select the most appropriate organelle that matches the following
descriptions
1- Contains enzymes capable of digesting cells and cellular
material
2- 'Reads' the mRNA and builds protein .
3- Modification of lipids and proteins with storage of material prior
to export out of the cell.
(6)[i][b] screening test for following
1- syphilis treponemal antibody test
2- syphilis hemaglutination test
3- syphilis immobilization test
4- HB-electrophorisis
5- mcv
6- mchc
7- MCh
8- serum iron
9- paul-bennel test[/b][/i]
A- alpha thalessemia
B- folic acid deficiency
C- iron deficiency anemia
D- primary syphilis in early untreated period
(7) on taking a large protein meal?
A- Decrease decrease
B- Increase increase
C- Increase decrease
D- Deacrease increase
E- Nochange no change
F- Nochange increase
G- Nochange decrease
Insulin & Glucagon excretion what will happened ...?
(8)
a- 3-hydroxyisovaleric acid .
b- 17-alpha hydroxyprogesterone .
c- 17-delta hydroxyprogesterone .
d- acetic acid .
e- cortisol .
The following level of certain metabolities in amniotic fluid
changes significantly of the fetus has an inborn error of the
metabolism .select the single metabolite from list of the options
above whose level in amniotic fluid is altered by inheretiance
disease in item below.
congentiat adrenal hyperplasia
(9)
A- Adrenaline
B- Calcitonin
C Cholecystokinin
D- Cortisol
E- Glucagon
F- Insulin
G- Progesterone
H- Somatostatin
IT- estosterone
Select the appropriate hormone from the list for the following
structures that produce it. Each answer may be used once, more
than once, or not at all.
1- Adrenal Cortex .
2- Adrenal Medulla .
3- Pancreatic alpha cell .
4- Pancreatic D Cells .
(10)
AAnterior cerebral artery
BAnterior communicating artery
CAnterior inferior cerebellar artery
DBasilar artery
EInternal auditory artery
FInternal carotid artery
GMiddle cerebral artery
HPosterior cerebral artery
IPosterior communicating artery
JSuperior cerebellar artery
KVertebral artery
Complete the diagram of the circle of Willis using the options
given:
this question came with diagrame in september 2007
so you should study circle of weils very well if I get the pictures I
will post it .
Akeratinising stratified squamous epithelium
Bnon-keratinising stratified squamous epithelium
Cpseudostratified columnar epithelium
Dsimple columnar epithelium
Esimple cuboidal epithelium
Ftransitional epithelium
Which of the epithelial types described above are present in the
anatomical regions below:
1- ectocervix .
2- endocervical canal .
3- cervical ectropion .
4- transformation zone of cervix .
5- vagina .
6- uterus .
7- Bladder .
8- Trachea .
9- Labia Majora .
10- Bowel .
11-Fallopian tube .
12- Vulva .
13- anal canal .
A- Autosomal co-dominant
B- Autosomal dominant
C- Autosomal recessive
D- Polygenic
E- Single gene defect
F- X linked dominant
G- X linked recessive
[Select the most likely mode of inheritance for the following patients’
conditions:
1- A 27 -year old female developed gestational diabetes mellitus. Her
uncle and grandmother also had diabetes mellitus2- A mothr is concerned
regarding her baby who has developed fractures which appear to occur
with minimal trauma. He has blue sclera.
3- An 18-year old female underwent caries tooth extraction and
developed profuse bleeding. On history she revealed menorrhagia. Her
mother and her grandfather had the same disease.
A- WARFARIN .
B- HEPARIN .
C- AMPICILLIN .
D- METHYLDOPA .
E- PENCILLIN .
F- CARBIMAZEPINE .
G- METRONIDAZOLE .
H- NON OF THE ABOVE .
I- ALL OF THE ABOVE .
For each of the following choose the single most appropirate statement
from the above list of options .
1- Drug contraindicated in breast feeding .
2- Drug does not cross the placenta .
A.
Calcitonin
B.
Cortisol
C.
Glucagon
D.
Growth hormone
E.
Insulin
F.
Oestradiol 17-¦Â¦Â
G.
Oxytocin
H.
Parathyroid hormone
I.
Prolactin
J.
Thyroxine
Instructions: For each action described below, choose the single
most likely causative hormone from the above list of options. Each
option may be used once, more than once, or not at all.
Question 3: Stimulates deposition of cartilage at the ends of bones
Question 4: Raises blood glucose levels through the breakdown of
fat and protein
Question 5: Antagonises the effect of parathyroid hormone to
minimise bone density loss
Question 6: Stimulates the release of milk from the breast
Question 7: Stimulates the urinary secretion of calcium
Options
A.
Common iliac artery
B.
External iliac artery
C.
Inferior epigastric artery
D.
Inferior vesical artery
E.
Internal iliac artery
F.
Middle rectal artery
G.
Ovarian artery
H.
Superior vesical artery
I.
Umbilical artery
J.
Uterine artery
K.
Vaginal artery
Instructions: After a forceps delivery a 30-year-old primigravida sustains
a primary post partum haemorrhage of four litres. Although the uterus
appears well contracted the bleeding continues, and a decision is made to
identify and treat the bleeding point radiologically. For each question
posed below, choose the single most appropriate option from the above
list. Each option may be used once, more than once or not at all.
148618503 2005-2010 obstetrics-gynecology-text-book-part-1-suhail-march-mrcog-part-1-pastpapers-recall-mrcog-total-guide
148618503 2005-2010 obstetrics-gynecology-text-book-part-1-suhail-march-mrcog-part-1-pastpapers-recall-mrcog-total-guide
148618503 2005-2010 obstetrics-gynecology-text-book-part-1-suhail-march-mrcog-part-1-pastpapers-recall-mrcog-total-guide
148618503 2005-2010 obstetrics-gynecology-text-book-part-1-suhail-march-mrcog-part-1-pastpapers-recall-mrcog-total-guide
148618503 2005-2010 obstetrics-gynecology-text-book-part-1-suhail-march-mrcog-part-1-pastpapers-recall-mrcog-total-guide
148618503 2005-2010 obstetrics-gynecology-text-book-part-1-suhail-march-mrcog-part-1-pastpapers-recall-mrcog-total-guide
148618503 2005-2010 obstetrics-gynecology-text-book-part-1-suhail-march-mrcog-part-1-pastpapers-recall-mrcog-total-guide
148618503 2005-2010 obstetrics-gynecology-text-book-part-1-suhail-march-mrcog-part-1-pastpapers-recall-mrcog-total-guide
148618503 2005-2010 obstetrics-gynecology-text-book-part-1-suhail-march-mrcog-part-1-pastpapers-recall-mrcog-total-guide
148618503 2005-2010 obstetrics-gynecology-text-book-part-1-suhail-march-mrcog-part-1-pastpapers-recall-mrcog-total-guide
148618503 2005-2010 obstetrics-gynecology-text-book-part-1-suhail-march-mrcog-part-1-pastpapers-recall-mrcog-total-guide
148618503 2005-2010 obstetrics-gynecology-text-book-part-1-suhail-march-mrcog-part-1-pastpapers-recall-mrcog-total-guide
148618503 2005-2010 obstetrics-gynecology-text-book-part-1-suhail-march-mrcog-part-1-pastpapers-recall-mrcog-total-guide
148618503 2005-2010 obstetrics-gynecology-text-book-part-1-suhail-march-mrcog-part-1-pastpapers-recall-mrcog-total-guide
148618503 2005-2010 obstetrics-gynecology-text-book-part-1-suhail-march-mrcog-part-1-pastpapers-recall-mrcog-total-guide
148618503 2005-2010 obstetrics-gynecology-text-book-part-1-suhail-march-mrcog-part-1-pastpapers-recall-mrcog-total-guide
148618503 2005-2010 obstetrics-gynecology-text-book-part-1-suhail-march-mrcog-part-1-pastpapers-recall-mrcog-total-guide
148618503 2005-2010 obstetrics-gynecology-text-book-part-1-suhail-march-mrcog-part-1-pastpapers-recall-mrcog-total-guide
148618503 2005-2010 obstetrics-gynecology-text-book-part-1-suhail-march-mrcog-part-1-pastpapers-recall-mrcog-total-guide
148618503 2005-2010 obstetrics-gynecology-text-book-part-1-suhail-march-mrcog-part-1-pastpapers-recall-mrcog-total-guide
148618503 2005-2010 obstetrics-gynecology-text-book-part-1-suhail-march-mrcog-part-1-pastpapers-recall-mrcog-total-guide
148618503 2005-2010 obstetrics-gynecology-text-book-part-1-suhail-march-mrcog-part-1-pastpapers-recall-mrcog-total-guide
148618503 2005-2010 obstetrics-gynecology-text-book-part-1-suhail-march-mrcog-part-1-pastpapers-recall-mrcog-total-guide
148618503 2005-2010 obstetrics-gynecology-text-book-part-1-suhail-march-mrcog-part-1-pastpapers-recall-mrcog-total-guide
148618503 2005-2010 obstetrics-gynecology-text-book-part-1-suhail-march-mrcog-part-1-pastpapers-recall-mrcog-total-guide
148618503 2005-2010 obstetrics-gynecology-text-book-part-1-suhail-march-mrcog-part-1-pastpapers-recall-mrcog-total-guide
148618503 2005-2010 obstetrics-gynecology-text-book-part-1-suhail-march-mrcog-part-1-pastpapers-recall-mrcog-total-guide
148618503 2005-2010 obstetrics-gynecology-text-book-part-1-suhail-march-mrcog-part-1-pastpapers-recall-mrcog-total-guide
148618503 2005-2010 obstetrics-gynecology-text-book-part-1-suhail-march-mrcog-part-1-pastpapers-recall-mrcog-total-guide
148618503 2005-2010 obstetrics-gynecology-text-book-part-1-suhail-march-mrcog-part-1-pastpapers-recall-mrcog-total-guide
148618503 2005-2010 obstetrics-gynecology-text-book-part-1-suhail-march-mrcog-part-1-pastpapers-recall-mrcog-total-guide
148618503 2005-2010 obstetrics-gynecology-text-book-part-1-suhail-march-mrcog-part-1-pastpapers-recall-mrcog-total-guide
148618503 2005-2010 obstetrics-gynecology-text-book-part-1-suhail-march-mrcog-part-1-pastpapers-recall-mrcog-total-guide
148618503 2005-2010 obstetrics-gynecology-text-book-part-1-suhail-march-mrcog-part-1-pastpapers-recall-mrcog-total-guide
148618503 2005-2010 obstetrics-gynecology-text-book-part-1-suhail-march-mrcog-part-1-pastpapers-recall-mrcog-total-guide
148618503 2005-2010 obstetrics-gynecology-text-book-part-1-suhail-march-mrcog-part-1-pastpapers-recall-mrcog-total-guide
148618503 2005-2010 obstetrics-gynecology-text-book-part-1-suhail-march-mrcog-part-1-pastpapers-recall-mrcog-total-guide
148618503 2005-2010 obstetrics-gynecology-text-book-part-1-suhail-march-mrcog-part-1-pastpapers-recall-mrcog-total-guide
148618503 2005-2010 obstetrics-gynecology-text-book-part-1-suhail-march-mrcog-part-1-pastpapers-recall-mrcog-total-guide
148618503 2005-2010 obstetrics-gynecology-text-book-part-1-suhail-march-mrcog-part-1-pastpapers-recall-mrcog-total-guide
148618503 2005-2010 obstetrics-gynecology-text-book-part-1-suhail-march-mrcog-part-1-pastpapers-recall-mrcog-total-guide
148618503 2005-2010 obstetrics-gynecology-text-book-part-1-suhail-march-mrcog-part-1-pastpapers-recall-mrcog-total-guide
148618503 2005-2010 obstetrics-gynecology-text-book-part-1-suhail-march-mrcog-part-1-pastpapers-recall-mrcog-total-guide
148618503 2005-2010 obstetrics-gynecology-text-book-part-1-suhail-march-mrcog-part-1-pastpapers-recall-mrcog-total-guide
148618503 2005-2010 obstetrics-gynecology-text-book-part-1-suhail-march-mrcog-part-1-pastpapers-recall-mrcog-total-guide
148618503 2005-2010 obstetrics-gynecology-text-book-part-1-suhail-march-mrcog-part-1-pastpapers-recall-mrcog-total-guide
148618503 2005-2010 obstetrics-gynecology-text-book-part-1-suhail-march-mrcog-part-1-pastpapers-recall-mrcog-total-guide
148618503 2005-2010 obstetrics-gynecology-text-book-part-1-suhail-march-mrcog-part-1-pastpapers-recall-mrcog-total-guide
148618503 2005-2010 obstetrics-gynecology-text-book-part-1-suhail-march-mrcog-part-1-pastpapers-recall-mrcog-total-guide
148618503 2005-2010 obstetrics-gynecology-text-book-part-1-suhail-march-mrcog-part-1-pastpapers-recall-mrcog-total-guide
148618503 2005-2010 obstetrics-gynecology-text-book-part-1-suhail-march-mrcog-part-1-pastpapers-recall-mrcog-total-guide
148618503 2005-2010 obstetrics-gynecology-text-book-part-1-suhail-march-mrcog-part-1-pastpapers-recall-mrcog-total-guide
148618503 2005-2010 obstetrics-gynecology-text-book-part-1-suhail-march-mrcog-part-1-pastpapers-recall-mrcog-total-guide
148618503 2005-2010 obstetrics-gynecology-text-book-part-1-suhail-march-mrcog-part-1-pastpapers-recall-mrcog-total-guide
148618503 2005-2010 obstetrics-gynecology-text-book-part-1-suhail-march-mrcog-part-1-pastpapers-recall-mrcog-total-guide
148618503 2005-2010 obstetrics-gynecology-text-book-part-1-suhail-march-mrcog-part-1-pastpapers-recall-mrcog-total-guide
148618503 2005-2010 obstetrics-gynecology-text-book-part-1-suhail-march-mrcog-part-1-pastpapers-recall-mrcog-total-guide
148618503 2005-2010 obstetrics-gynecology-text-book-part-1-suhail-march-mrcog-part-1-pastpapers-recall-mrcog-total-guide
148618503 2005-2010 obstetrics-gynecology-text-book-part-1-suhail-march-mrcog-part-1-pastpapers-recall-mrcog-total-guide
148618503 2005-2010 obstetrics-gynecology-text-book-part-1-suhail-march-mrcog-part-1-pastpapers-recall-mrcog-total-guide
148618503 2005-2010 obstetrics-gynecology-text-book-part-1-suhail-march-mrcog-part-1-pastpapers-recall-mrcog-total-guide
148618503 2005-2010 obstetrics-gynecology-text-book-part-1-suhail-march-mrcog-part-1-pastpapers-recall-mrcog-total-guide
148618503 2005-2010 obstetrics-gynecology-text-book-part-1-suhail-march-mrcog-part-1-pastpapers-recall-mrcog-total-guide
148618503 2005-2010 obstetrics-gynecology-text-book-part-1-suhail-march-mrcog-part-1-pastpapers-recall-mrcog-total-guide
148618503 2005-2010 obstetrics-gynecology-text-book-part-1-suhail-march-mrcog-part-1-pastpapers-recall-mrcog-total-guide
148618503 2005-2010 obstetrics-gynecology-text-book-part-1-suhail-march-mrcog-part-1-pastpapers-recall-mrcog-total-guide
148618503 2005-2010 obstetrics-gynecology-text-book-part-1-suhail-march-mrcog-part-1-pastpapers-recall-mrcog-total-guide
148618503 2005-2010 obstetrics-gynecology-text-book-part-1-suhail-march-mrcog-part-1-pastpapers-recall-mrcog-total-guide
148618503 2005-2010 obstetrics-gynecology-text-book-part-1-suhail-march-mrcog-part-1-pastpapers-recall-mrcog-total-guide
148618503 2005-2010 obstetrics-gynecology-text-book-part-1-suhail-march-mrcog-part-1-pastpapers-recall-mrcog-total-guide
148618503 2005-2010 obstetrics-gynecology-text-book-part-1-suhail-march-mrcog-part-1-pastpapers-recall-mrcog-total-guide
148618503 2005-2010 obstetrics-gynecology-text-book-part-1-suhail-march-mrcog-part-1-pastpapers-recall-mrcog-total-guide
148618503 2005-2010 obstetrics-gynecology-text-book-part-1-suhail-march-mrcog-part-1-pastpapers-recall-mrcog-total-guide
148618503 2005-2010 obstetrics-gynecology-text-book-part-1-suhail-march-mrcog-part-1-pastpapers-recall-mrcog-total-guide
148618503 2005-2010 obstetrics-gynecology-text-book-part-1-suhail-march-mrcog-part-1-pastpapers-recall-mrcog-total-guide
148618503 2005-2010 obstetrics-gynecology-text-book-part-1-suhail-march-mrcog-part-1-pastpapers-recall-mrcog-total-guide
148618503 2005-2010 obstetrics-gynecology-text-book-part-1-suhail-march-mrcog-part-1-pastpapers-recall-mrcog-total-guide
148618503 2005-2010 obstetrics-gynecology-text-book-part-1-suhail-march-mrcog-part-1-pastpapers-recall-mrcog-total-guide
148618503 2005-2010 obstetrics-gynecology-text-book-part-1-suhail-march-mrcog-part-1-pastpapers-recall-mrcog-total-guide
148618503 2005-2010 obstetrics-gynecology-text-book-part-1-suhail-march-mrcog-part-1-pastpapers-recall-mrcog-total-guide
148618503 2005-2010 obstetrics-gynecology-text-book-part-1-suhail-march-mrcog-part-1-pastpapers-recall-mrcog-total-guide
148618503 2005-2010 obstetrics-gynecology-text-book-part-1-suhail-march-mrcog-part-1-pastpapers-recall-mrcog-total-guide
148618503 2005-2010 obstetrics-gynecology-text-book-part-1-suhail-march-mrcog-part-1-pastpapers-recall-mrcog-total-guide
148618503 2005-2010 obstetrics-gynecology-text-book-part-1-suhail-march-mrcog-part-1-pastpapers-recall-mrcog-total-guide
148618503 2005-2010 obstetrics-gynecology-text-book-part-1-suhail-march-mrcog-part-1-pastpapers-recall-mrcog-total-guide
148618503 2005-2010 obstetrics-gynecology-text-book-part-1-suhail-march-mrcog-part-1-pastpapers-recall-mrcog-total-guide
148618503 2005-2010 obstetrics-gynecology-text-book-part-1-suhail-march-mrcog-part-1-pastpapers-recall-mrcog-total-guide
148618503 2005-2010 obstetrics-gynecology-text-book-part-1-suhail-march-mrcog-part-1-pastpapers-recall-mrcog-total-guide
148618503 2005-2010 obstetrics-gynecology-text-book-part-1-suhail-march-mrcog-part-1-pastpapers-recall-mrcog-total-guide
148618503 2005-2010 obstetrics-gynecology-text-book-part-1-suhail-march-mrcog-part-1-pastpapers-recall-mrcog-total-guide
148618503 2005-2010 obstetrics-gynecology-text-book-part-1-suhail-march-mrcog-part-1-pastpapers-recall-mrcog-total-guide
148618503 2005-2010 obstetrics-gynecology-text-book-part-1-suhail-march-mrcog-part-1-pastpapers-recall-mrcog-total-guide
148618503 2005-2010 obstetrics-gynecology-text-book-part-1-suhail-march-mrcog-part-1-pastpapers-recall-mrcog-total-guide
148618503 2005-2010 obstetrics-gynecology-text-book-part-1-suhail-march-mrcog-part-1-pastpapers-recall-mrcog-total-guide
148618503 2005-2010 obstetrics-gynecology-text-book-part-1-suhail-march-mrcog-part-1-pastpapers-recall-mrcog-total-guide
148618503 2005-2010 obstetrics-gynecology-text-book-part-1-suhail-march-mrcog-part-1-pastpapers-recall-mrcog-total-guide
148618503 2005-2010 obstetrics-gynecology-text-book-part-1-suhail-march-mrcog-part-1-pastpapers-recall-mrcog-total-guide
148618503 2005-2010 obstetrics-gynecology-text-book-part-1-suhail-march-mrcog-part-1-pastpapers-recall-mrcog-total-guide
148618503 2005-2010 obstetrics-gynecology-text-book-part-1-suhail-march-mrcog-part-1-pastpapers-recall-mrcog-total-guide
148618503 2005-2010 obstetrics-gynecology-text-book-part-1-suhail-march-mrcog-part-1-pastpapers-recall-mrcog-total-guide
148618503 2005-2010 obstetrics-gynecology-text-book-part-1-suhail-march-mrcog-part-1-pastpapers-recall-mrcog-total-guide
148618503 2005-2010 obstetrics-gynecology-text-book-part-1-suhail-march-mrcog-part-1-pastpapers-recall-mrcog-total-guide
148618503 2005-2010 obstetrics-gynecology-text-book-part-1-suhail-march-mrcog-part-1-pastpapers-recall-mrcog-total-guide
148618503 2005-2010 obstetrics-gynecology-text-book-part-1-suhail-march-mrcog-part-1-pastpapers-recall-mrcog-total-guide
148618503 2005-2010 obstetrics-gynecology-text-book-part-1-suhail-march-mrcog-part-1-pastpapers-recall-mrcog-total-guide
148618503 2005-2010 obstetrics-gynecology-text-book-part-1-suhail-march-mrcog-part-1-pastpapers-recall-mrcog-total-guide
148618503 2005-2010 obstetrics-gynecology-text-book-part-1-suhail-march-mrcog-part-1-pastpapers-recall-mrcog-total-guide
148618503 2005-2010 obstetrics-gynecology-text-book-part-1-suhail-march-mrcog-part-1-pastpapers-recall-mrcog-total-guide
148618503 2005-2010 obstetrics-gynecology-text-book-part-1-suhail-march-mrcog-part-1-pastpapers-recall-mrcog-total-guide
148618503 2005-2010 obstetrics-gynecology-text-book-part-1-suhail-march-mrcog-part-1-pastpapers-recall-mrcog-total-guide
148618503 2005-2010 obstetrics-gynecology-text-book-part-1-suhail-march-mrcog-part-1-pastpapers-recall-mrcog-total-guide
148618503 2005-2010 obstetrics-gynecology-text-book-part-1-suhail-march-mrcog-part-1-pastpapers-recall-mrcog-total-guide
148618503 2005-2010 obstetrics-gynecology-text-book-part-1-suhail-march-mrcog-part-1-pastpapers-recall-mrcog-total-guide
148618503 2005-2010 obstetrics-gynecology-text-book-part-1-suhail-march-mrcog-part-1-pastpapers-recall-mrcog-total-guide
148618503 2005-2010 obstetrics-gynecology-text-book-part-1-suhail-march-mrcog-part-1-pastpapers-recall-mrcog-total-guide
148618503 2005-2010 obstetrics-gynecology-text-book-part-1-suhail-march-mrcog-part-1-pastpapers-recall-mrcog-total-guide
148618503 2005-2010 obstetrics-gynecology-text-book-part-1-suhail-march-mrcog-part-1-pastpapers-recall-mrcog-total-guide
148618503 2005-2010 obstetrics-gynecology-text-book-part-1-suhail-march-mrcog-part-1-pastpapers-recall-mrcog-total-guide
148618503 2005-2010 obstetrics-gynecology-text-book-part-1-suhail-march-mrcog-part-1-pastpapers-recall-mrcog-total-guide

Más contenido relacionado

La actualidad más candente

La actualidad más candente (20)

Ahmed elmelhat neonatal thrombocytopenia
Ahmed elmelhat neonatal thrombocytopeniaAhmed elmelhat neonatal thrombocytopenia
Ahmed elmelhat neonatal thrombocytopenia
 
Gaucher disease
Gaucher diseaseGaucher disease
Gaucher disease
 
Congenital adrenal hyperplasia
Congenital adrenal hyperplasiaCongenital adrenal hyperplasia
Congenital adrenal hyperplasia
 
MRCOG Part 1 Practice MCQs
MRCOG Part 1 Practice MCQsMRCOG Part 1 Practice MCQs
MRCOG Part 1 Practice MCQs
 
vte path and rx
vte path and rxvte path and rx
vte path and rx
 
Chronic Myeloid Leukemia
Chronic Myeloid LeukemiaChronic Myeloid Leukemia
Chronic Myeloid Leukemia
 
Niemann pick disease
Niemann pick diseaseNiemann pick disease
Niemann pick disease
 
Hemophagocyitic histiocytosis
Hemophagocyitic histiocytosisHemophagocyitic histiocytosis
Hemophagocyitic histiocytosis
 
Gaucher's disease - Pathology
Gaucher's disease - PathologyGaucher's disease - Pathology
Gaucher's disease - Pathology
 
NEONATAL JAUNDICE- ALL YOU NEED TO KNOW
NEONATAL JAUNDICE- ALL YOU NEED TO  KNOWNEONATAL JAUNDICE- ALL YOU NEED TO  KNOW
NEONATAL JAUNDICE- ALL YOU NEED TO KNOW
 
Acute lymphoblastic leukemia
Acute lymphoblastic leukemiaAcute lymphoblastic leukemia
Acute lymphoblastic leukemia
 
Cmp
CmpCmp
Cmp
 
Non immuine hydrops fetalis
Non immuine hydrops fetalisNon immuine hydrops fetalis
Non immuine hydrops fetalis
 
NAIT
NAITNAIT
NAIT
 
Minimal Change Diseas
Minimal Change DiseasMinimal Change Diseas
Minimal Change Diseas
 
Chronic Myelogenous Leukemia
Chronic Myelogenous LeukemiaChronic Myelogenous Leukemia
Chronic Myelogenous Leukemia
 
A case of Chronic Myeloid Leukemia
A case of Chronic Myeloid LeukemiaA case of Chronic Myeloid Leukemia
A case of Chronic Myeloid Leukemia
 
Haemorrhagic and Haemolytic of Newborn Diseases
Haemorrhagic and Haemolytic of Newborn DiseasesHaemorrhagic and Haemolytic of Newborn Diseases
Haemorrhagic and Haemolytic of Newborn Diseases
 
Fabry disease
Fabry diseaseFabry disease
Fabry disease
 
Congenital Adrenal Hyperplasia (CAH)
Congenital Adrenal Hyperplasia (CAH)Congenital Adrenal Hyperplasia (CAH)
Congenital Adrenal Hyperplasia (CAH)
 

Destacado (7)

Obstetics simplified el-mowafi
Obstetics simplified el-mowafiObstetics simplified el-mowafi
Obstetics simplified el-mowafi
 
DR.Mona Part 1 MRCOG
DR.Mona Part 1 MRCOGDR.Mona Part 1 MRCOG
DR.Mona Part 1 MRCOG
 
Obstetrics Quiz
Obstetrics QuizObstetrics Quiz
Obstetrics Quiz
 
Gynae mcq
Gynae mcqGynae mcq
Gynae mcq
 
MRCOG PART 1 PRACTICE MCQ
MRCOG PART 1 PRACTICE MCQMRCOG PART 1 PRACTICE MCQ
MRCOG PART 1 PRACTICE MCQ
 
Obstetric hemorrhage cases and MCQ for undergraduate
Obstetric hemorrhage cases and MCQ for undergraduateObstetric hemorrhage cases and MCQ for undergraduate
Obstetric hemorrhage cases and MCQ for undergraduate
 
O&g01.williams obstetrics & gynecology selected questions 2
O&g01.williams obstetrics & gynecology   selected questions 2O&g01.williams obstetrics & gynecology   selected questions 2
O&g01.williams obstetrics & gynecology selected questions 2
 

Similar a 148618503 2005-2010 obstetrics-gynecology-text-book-part-1-suhail-march-mrcog-part-1-pastpapers-recall-mrcog-total-guide

5919003-230817133315-c77bnnnhhhca1c8.pdf
5919003-230817133315-c77bnnnhhhca1c8.pdf5919003-230817133315-c77bnnnhhhca1c8.pdf
5919003-230817133315-c77bnnnhhhca1c8.pdfMitikuTeka1
 
Pathology of Endocrine system.ppt
Pathology of Endocrine system.pptPathology of Endocrine system.ppt
Pathology of Endocrine system.pptDARMAUSADA
 
Pituitary april-12 د فائزة
Pituitary april-12 د فائزة Pituitary april-12 د فائزة
Pituitary april-12 د فائزة eliasmawla
 
Paraneoplastic Endocrine Syndrome
Paraneoplastic Endocrine SyndromeParaneoplastic Endocrine Syndrome
Paraneoplastic Endocrine SyndromeDJ CrissCross
 
Lecture 1. hyperpituitarism
Lecture 1. hyperpituitarismLecture 1. hyperpituitarism
Lecture 1. hyperpituitarismAyub Abdi
 
Pediatric endocrinology review MCQs- part 6
Pediatric endocrinology review MCQs- part 6Pediatric endocrinology review MCQs- part 6
Pediatric endocrinology review MCQs- part 6Abdulmoein AlAgha
 
Lecture 9. adrenal medulla diseases
Lecture 9. adrenal medulla diseasesLecture 9. adrenal medulla diseases
Lecture 9. adrenal medulla diseasesAyub Abdi
 
Pulmonery oedema in pregnancy case reporet
Pulmonery oedema in pregnancy case reporetPulmonery oedema in pregnancy case reporet
Pulmonery oedema in pregnancy case reporetWaled Abohatab
 
Final paraneoplastic syndromes
Final paraneoplastic syndromesFinal paraneoplastic syndromes
Final paraneoplastic syndromesDrRashmiBudha
 
Phaeochromocytoma a case
Phaeochromocytoma a casePhaeochromocytoma a case
Phaeochromocytoma a caseREKHAKHARE
 
Amenorrhea - Define, Cause, Sign and Symptoms, Type- Pathological and Physiol...
Amenorrhea - Define, Cause, Sign and Symptoms, Type- Pathological and Physiol...Amenorrhea - Define, Cause, Sign and Symptoms, Type- Pathological and Physiol...
Amenorrhea - Define, Cause, Sign and Symptoms, Type- Pathological and Physiol...sonal patel
 
Hypertensive disorders in pregnancy
Hypertensive disorders in pregnancyHypertensive disorders in pregnancy
Hypertensive disorders in pregnancyPriti Patil
 

Similar a 148618503 2005-2010 obstetrics-gynecology-text-book-part-1-suhail-march-mrcog-part-1-pastpapers-recall-mrcog-total-guide (20)

5919003-230817133315-c77bnnnhhhca1c8.pdf
5919003-230817133315-c77bnnnhhhca1c8.pdf5919003-230817133315-c77bnnnhhhca1c8.pdf
5919003-230817133315-c77bnnnhhhca1c8.pdf
 
Pathology of Endocrine system.ppt
Pathology of Endocrine system.pptPathology of Endocrine system.ppt
Pathology of Endocrine system.ppt
 
Pituitary april-12 د فائزة
Pituitary april-12 د فائزة Pituitary april-12 د فائزة
Pituitary april-12 د فائزة
 
Male infertility 2 2018)
Male infertility 2 2018)Male infertility 2 2018)
Male infertility 2 2018)
 
Prolactinoma
ProlactinomaProlactinoma
Prolactinoma
 
PHEOCHROMOCYTOMA
PHEOCHROMOCYTOMAPHEOCHROMOCYTOMA
PHEOCHROMOCYTOMA
 
Paraneoplastic Endocrine Syndrome
Paraneoplastic Endocrine SyndromeParaneoplastic Endocrine Syndrome
Paraneoplastic Endocrine Syndrome
 
Pheochromocytoma
Pheochromocytoma Pheochromocytoma
Pheochromocytoma
 
Congenital adrenal hyperplasia
Congenital adrenal hyperplasiaCongenital adrenal hyperplasia
Congenital adrenal hyperplasia
 
Lecture 1. hyperpituitarism
Lecture 1. hyperpituitarismLecture 1. hyperpituitarism
Lecture 1. hyperpituitarism
 
Adrenal glands hegazy
Adrenal glands hegazyAdrenal glands hegazy
Adrenal glands hegazy
 
Adrenal glands hegazy
Adrenal glands hegazyAdrenal glands hegazy
Adrenal glands hegazy
 
Pediatric endocrinology review MCQs- part 6
Pediatric endocrinology review MCQs- part 6Pediatric endocrinology review MCQs- part 6
Pediatric endocrinology review MCQs- part 6
 
Lecture 9. adrenal medulla diseases
Lecture 9. adrenal medulla diseasesLecture 9. adrenal medulla diseases
Lecture 9. adrenal medulla diseases
 
Copy Of Obs
Copy Of ObsCopy Of Obs
Copy Of Obs
 
Pulmonery oedema in pregnancy case reporet
Pulmonery oedema in pregnancy case reporetPulmonery oedema in pregnancy case reporet
Pulmonery oedema in pregnancy case reporet
 
Final paraneoplastic syndromes
Final paraneoplastic syndromesFinal paraneoplastic syndromes
Final paraneoplastic syndromes
 
Phaeochromocytoma a case
Phaeochromocytoma a casePhaeochromocytoma a case
Phaeochromocytoma a case
 
Amenorrhea - Define, Cause, Sign and Symptoms, Type- Pathological and Physiol...
Amenorrhea - Define, Cause, Sign and Symptoms, Type- Pathological and Physiol...Amenorrhea - Define, Cause, Sign and Symptoms, Type- Pathological and Physiol...
Amenorrhea - Define, Cause, Sign and Symptoms, Type- Pathological and Physiol...
 
Hypertensive disorders in pregnancy
Hypertensive disorders in pregnancyHypertensive disorders in pregnancy
Hypertensive disorders in pregnancy
 

Más de Bharti Gahtori

ROLE OF ULTRASOUND IN MULTIFETAL GESTATION - WHAT AN OBSTETRICIAN SHOULD KNOW ?
ROLE OF ULTRASOUND IN MULTIFETAL GESTATION - WHAT AN OBSTETRICIAN SHOULD KNOW ?ROLE OF ULTRASOUND IN MULTIFETAL GESTATION - WHAT AN OBSTETRICIAN SHOULD KNOW ?
ROLE OF ULTRASOUND IN MULTIFETAL GESTATION - WHAT AN OBSTETRICIAN SHOULD KNOW ?Bharti Gahtori
 
Role of ultrasound in the Infertility management
Role of ultrasound in the Infertility  management Role of ultrasound in the Infertility  management
Role of ultrasound in the Infertility management Bharti Gahtori
 
Ultrasound report the black box of fetal status
Ultrasound report the black box of fetal status Ultrasound report the black box of fetal status
Ultrasound report the black box of fetal status Bharti Gahtori
 
Kalpavriksha final questionnaire 2017 class 8 th pdf
Kalpavriksha final questionnaire 2017 class 8 th pdfKalpavriksha final questionnaire 2017 class 8 th pdf
Kalpavriksha final questionnaire 2017 class 8 th pdfBharti Gahtori
 
Recent advances in gynecologic usg
Recent advances in gynecologic  usg Recent advances in gynecologic  usg
Recent advances in gynecologic usg Bharti Gahtori
 
Intrauterine growth restriction
Intrauterine growth restrictionIntrauterine growth restriction
Intrauterine growth restrictionBharti Gahtori
 
Final second round junior ppt 2016
Final second round junior ppt 2016Final second round junior ppt 2016
Final second round junior ppt 2016Bharti Gahtori
 
148012541 90397156-homemade-recipes-book-soaps-ointments-candles-shampoos-and...
148012541 90397156-homemade-recipes-book-soaps-ointments-candles-shampoos-and...148012541 90397156-homemade-recipes-book-soaps-ointments-candles-shampoos-and...
148012541 90397156-homemade-recipes-book-soaps-ointments-candles-shampoos-and...Bharti Gahtori
 
144923688 danforth-obstetrics-and-gynecology-9ed
144923688 danforth-obstetrics-and-gynecology-9ed144923688 danforth-obstetrics-and-gynecology-9ed
144923688 danforth-obstetrics-and-gynecology-9edBharti Gahtori
 
90774117 aims-of-obstetric-critical-care-management
90774117 aims-of-obstetric-critical-care-management90774117 aims-of-obstetric-critical-care-management
90774117 aims-of-obstetric-critical-care-managementBharti Gahtori
 
NCERT BASED GEOGRAPHY QUIZ
NCERT BASED GEOGRAPHY QUIZNCERT BASED GEOGRAPHY QUIZ
NCERT BASED GEOGRAPHY QUIZBharti Gahtori
 
Focused approach to antenatal care - First trimester screening
Focused approach to antenatal care - First trimester screeningFocused approach to antenatal care - First trimester screening
Focused approach to antenatal care - First trimester screeningBharti Gahtori
 
SWACCH KASHIPUR ABHIYAAN -MY VISION
SWACCH KASHIPUR ABHIYAAN -MY VISIONSWACCH KASHIPUR ABHIYAAN -MY VISION
SWACCH KASHIPUR ABHIYAAN -MY VISIONBharti Gahtori
 
Obstetrics doppler ultrasound
Obstetrics doppler ultrasoundObstetrics doppler ultrasound
Obstetrics doppler ultrasoundBharti Gahtori
 

Más de Bharti Gahtori (15)

ROLE OF ULTRASOUND IN MULTIFETAL GESTATION - WHAT AN OBSTETRICIAN SHOULD KNOW ?
ROLE OF ULTRASOUND IN MULTIFETAL GESTATION - WHAT AN OBSTETRICIAN SHOULD KNOW ?ROLE OF ULTRASOUND IN MULTIFETAL GESTATION - WHAT AN OBSTETRICIAN SHOULD KNOW ?
ROLE OF ULTRASOUND IN MULTIFETAL GESTATION - WHAT AN OBSTETRICIAN SHOULD KNOW ?
 
Role of ultrasound in the Infertility management
Role of ultrasound in the Infertility  management Role of ultrasound in the Infertility  management
Role of ultrasound in the Infertility management
 
Ultrasound report the black box of fetal status
Ultrasound report the black box of fetal status Ultrasound report the black box of fetal status
Ultrasound report the black box of fetal status
 
Kalpavriksha final questionnaire 2017 class 8 th pdf
Kalpavriksha final questionnaire 2017 class 8 th pdfKalpavriksha final questionnaire 2017 class 8 th pdf
Kalpavriksha final questionnaire 2017 class 8 th pdf
 
Recent advances in gynecologic usg
Recent advances in gynecologic  usg Recent advances in gynecologic  usg
Recent advances in gynecologic usg
 
Intrauterine growth restriction
Intrauterine growth restrictionIntrauterine growth restriction
Intrauterine growth restriction
 
Final second round junior ppt 2016
Final second round junior ppt 2016Final second round junior ppt 2016
Final second round junior ppt 2016
 
148012541 90397156-homemade-recipes-book-soaps-ointments-candles-shampoos-and...
148012541 90397156-homemade-recipes-book-soaps-ointments-candles-shampoos-and...148012541 90397156-homemade-recipes-book-soaps-ointments-candles-shampoos-and...
148012541 90397156-homemade-recipes-book-soaps-ointments-candles-shampoos-and...
 
144923688 danforth-obstetrics-and-gynecology-9ed
144923688 danforth-obstetrics-and-gynecology-9ed144923688 danforth-obstetrics-and-gynecology-9ed
144923688 danforth-obstetrics-and-gynecology-9ed
 
90774117 aims-of-obstetric-critical-care-management
90774117 aims-of-obstetric-critical-care-management90774117 aims-of-obstetric-critical-care-management
90774117 aims-of-obstetric-critical-care-management
 
NCERT BASED GEOGRAPHY QUIZ
NCERT BASED GEOGRAPHY QUIZNCERT BASED GEOGRAPHY QUIZ
NCERT BASED GEOGRAPHY QUIZ
 
Focused approach to antenatal care - First trimester screening
Focused approach to antenatal care - First trimester screeningFocused approach to antenatal care - First trimester screening
Focused approach to antenatal care - First trimester screening
 
SWACCH KASHIPUR ABHIYAAN -MY VISION
SWACCH KASHIPUR ABHIYAAN -MY VISIONSWACCH KASHIPUR ABHIYAAN -MY VISION
SWACCH KASHIPUR ABHIYAAN -MY VISION
 
Doppler in pregnancy
Doppler in pregnancyDoppler in pregnancy
Doppler in pregnancy
 
Obstetrics doppler ultrasound
Obstetrics doppler ultrasoundObstetrics doppler ultrasound
Obstetrics doppler ultrasound
 

Último

Call Girls Service Chennai Jiya 7001305949 Independent Escort Service Chennai
Call Girls Service Chennai Jiya 7001305949 Independent Escort Service ChennaiCall Girls Service Chennai Jiya 7001305949 Independent Escort Service Chennai
Call Girls Service Chennai Jiya 7001305949 Independent Escort Service ChennaiNehru place Escorts
 
Housewife Call Girls Bangalore - Call 7001305949 Rs-3500 with A/C Room Cash o...
Housewife Call Girls Bangalore - Call 7001305949 Rs-3500 with A/C Room Cash o...Housewife Call Girls Bangalore - Call 7001305949 Rs-3500 with A/C Room Cash o...
Housewife Call Girls Bangalore - Call 7001305949 Rs-3500 with A/C Room Cash o...narwatsonia7
 
Call Girls Thane Just Call 9910780858 Get High Class Call Girls Service
Call Girls Thane Just Call 9910780858 Get High Class Call Girls ServiceCall Girls Thane Just Call 9910780858 Get High Class Call Girls Service
Call Girls Thane Just Call 9910780858 Get High Class Call Girls Servicesonalikaur4
 
Call Girls Service in Bommanahalli - 7001305949 with real photos and phone nu...
Call Girls Service in Bommanahalli - 7001305949 with real photos and phone nu...Call Girls Service in Bommanahalli - 7001305949 with real photos and phone nu...
Call Girls Service in Bommanahalli - 7001305949 with real photos and phone nu...narwatsonia7
 
Call Girls Hsr Layout Just Call 7001305949 Top Class Call Girl Service Available
Call Girls Hsr Layout Just Call 7001305949 Top Class Call Girl Service AvailableCall Girls Hsr Layout Just Call 7001305949 Top Class Call Girl Service Available
Call Girls Hsr Layout Just Call 7001305949 Top Class Call Girl Service Availablenarwatsonia7
 
Mumbai Call Girls Service 9910780858 Real Russian Girls Looking Models
Mumbai Call Girls Service 9910780858 Real Russian Girls Looking ModelsMumbai Call Girls Service 9910780858 Real Russian Girls Looking Models
Mumbai Call Girls Service 9910780858 Real Russian Girls Looking Modelssonalikaur4
 
Housewife Call Girls Hsr Layout - Call 7001305949 Rs-3500 with A/C Room Cash ...
Housewife Call Girls Hsr Layout - Call 7001305949 Rs-3500 with A/C Room Cash ...Housewife Call Girls Hsr Layout - Call 7001305949 Rs-3500 with A/C Room Cash ...
Housewife Call Girls Hsr Layout - Call 7001305949 Rs-3500 with A/C Room Cash ...narwatsonia7
 
Call Girl Koramangala | 7001305949 At Low Cost Cash Payment Booking
Call Girl Koramangala | 7001305949 At Low Cost Cash Payment BookingCall Girl Koramangala | 7001305949 At Low Cost Cash Payment Booking
Call Girl Koramangala | 7001305949 At Low Cost Cash Payment Bookingnarwatsonia7
 
Russian Call Girl Brookfield - 7001305949 Escorts Service 50% Off with Cash O...
Russian Call Girl Brookfield - 7001305949 Escorts Service 50% Off with Cash O...Russian Call Girl Brookfield - 7001305949 Escorts Service 50% Off with Cash O...
Russian Call Girl Brookfield - 7001305949 Escorts Service 50% Off with Cash O...narwatsonia7
 
Call Girls Service Noida Maya 9711199012 Independent Escort Service Noida
Call Girls Service Noida Maya 9711199012 Independent Escort Service NoidaCall Girls Service Noida Maya 9711199012 Independent Escort Service Noida
Call Girls Service Noida Maya 9711199012 Independent Escort Service NoidaPooja Gupta
 
Low Rate Call Girls Mumbai Suman 9910780858 Independent Escort Service Mumbai
Low Rate Call Girls Mumbai Suman 9910780858 Independent Escort Service MumbaiLow Rate Call Girls Mumbai Suman 9910780858 Independent Escort Service Mumbai
Low Rate Call Girls Mumbai Suman 9910780858 Independent Escort Service Mumbaisonalikaur4
 
College Call Girls Vyasarpadi Whatsapp 7001305949 Independent Escort Service
College Call Girls Vyasarpadi Whatsapp 7001305949 Independent Escort ServiceCollege Call Girls Vyasarpadi Whatsapp 7001305949 Independent Escort Service
College Call Girls Vyasarpadi Whatsapp 7001305949 Independent Escort ServiceNehru place Escorts
 
VIP Call Girls Lucknow Nandini 7001305949 Independent Escort Service Lucknow
VIP Call Girls Lucknow Nandini 7001305949 Independent Escort Service LucknowVIP Call Girls Lucknow Nandini 7001305949 Independent Escort Service Lucknow
VIP Call Girls Lucknow Nandini 7001305949 Independent Escort Service Lucknownarwatsonia7
 
Call Girls Service Nandiambakkam | 7001305949 At Low Cost Cash Payment Booking
Call Girls Service Nandiambakkam | 7001305949 At Low Cost Cash Payment BookingCall Girls Service Nandiambakkam | 7001305949 At Low Cost Cash Payment Booking
Call Girls Service Nandiambakkam | 7001305949 At Low Cost Cash Payment BookingNehru place Escorts
 
Book Call Girls in Yelahanka - For 7001305949 Cheap & Best with original Photos
Book Call Girls in Yelahanka - For 7001305949 Cheap & Best with original PhotosBook Call Girls in Yelahanka - For 7001305949 Cheap & Best with original Photos
Book Call Girls in Yelahanka - For 7001305949 Cheap & Best with original Photosnarwatsonia7
 
97111 47426 Call Girls In Delhi MUNIRKAA
97111 47426 Call Girls In Delhi MUNIRKAA97111 47426 Call Girls In Delhi MUNIRKAA
97111 47426 Call Girls In Delhi MUNIRKAAjennyeacort
 
Call Girls ITPL Just Call 7001305949 Top Class Call Girl Service Available
Call Girls ITPL Just Call 7001305949 Top Class Call Girl Service AvailableCall Girls ITPL Just Call 7001305949 Top Class Call Girl Service Available
Call Girls ITPL Just Call 7001305949 Top Class Call Girl Service Availablenarwatsonia7
 
Call Girls Service In Shyam Nagar Whatsapp 8445551418 Independent Escort Service
Call Girls Service In Shyam Nagar Whatsapp 8445551418 Independent Escort ServiceCall Girls Service In Shyam Nagar Whatsapp 8445551418 Independent Escort Service
Call Girls Service In Shyam Nagar Whatsapp 8445551418 Independent Escort Serviceparulsinha
 
Call Girl Lucknow Mallika 7001305949 Independent Escort Service Lucknow
Call Girl Lucknow Mallika 7001305949 Independent Escort Service LucknowCall Girl Lucknow Mallika 7001305949 Independent Escort Service Lucknow
Call Girl Lucknow Mallika 7001305949 Independent Escort Service Lucknownarwatsonia7
 

Último (20)

Call Girls Service Chennai Jiya 7001305949 Independent Escort Service Chennai
Call Girls Service Chennai Jiya 7001305949 Independent Escort Service ChennaiCall Girls Service Chennai Jiya 7001305949 Independent Escort Service Chennai
Call Girls Service Chennai Jiya 7001305949 Independent Escort Service Chennai
 
Housewife Call Girls Bangalore - Call 7001305949 Rs-3500 with A/C Room Cash o...
Housewife Call Girls Bangalore - Call 7001305949 Rs-3500 with A/C Room Cash o...Housewife Call Girls Bangalore - Call 7001305949 Rs-3500 with A/C Room Cash o...
Housewife Call Girls Bangalore - Call 7001305949 Rs-3500 with A/C Room Cash o...
 
Call Girls Thane Just Call 9910780858 Get High Class Call Girls Service
Call Girls Thane Just Call 9910780858 Get High Class Call Girls ServiceCall Girls Thane Just Call 9910780858 Get High Class Call Girls Service
Call Girls Thane Just Call 9910780858 Get High Class Call Girls Service
 
Call Girls Service in Bommanahalli - 7001305949 with real photos and phone nu...
Call Girls Service in Bommanahalli - 7001305949 with real photos and phone nu...Call Girls Service in Bommanahalli - 7001305949 with real photos and phone nu...
Call Girls Service in Bommanahalli - 7001305949 with real photos and phone nu...
 
Call Girls Hsr Layout Just Call 7001305949 Top Class Call Girl Service Available
Call Girls Hsr Layout Just Call 7001305949 Top Class Call Girl Service AvailableCall Girls Hsr Layout Just Call 7001305949 Top Class Call Girl Service Available
Call Girls Hsr Layout Just Call 7001305949 Top Class Call Girl Service Available
 
Mumbai Call Girls Service 9910780858 Real Russian Girls Looking Models
Mumbai Call Girls Service 9910780858 Real Russian Girls Looking ModelsMumbai Call Girls Service 9910780858 Real Russian Girls Looking Models
Mumbai Call Girls Service 9910780858 Real Russian Girls Looking Models
 
Housewife Call Girls Hsr Layout - Call 7001305949 Rs-3500 with A/C Room Cash ...
Housewife Call Girls Hsr Layout - Call 7001305949 Rs-3500 with A/C Room Cash ...Housewife Call Girls Hsr Layout - Call 7001305949 Rs-3500 with A/C Room Cash ...
Housewife Call Girls Hsr Layout - Call 7001305949 Rs-3500 with A/C Room Cash ...
 
Call Girl Koramangala | 7001305949 At Low Cost Cash Payment Booking
Call Girl Koramangala | 7001305949 At Low Cost Cash Payment BookingCall Girl Koramangala | 7001305949 At Low Cost Cash Payment Booking
Call Girl Koramangala | 7001305949 At Low Cost Cash Payment Booking
 
Russian Call Girl Brookfield - 7001305949 Escorts Service 50% Off with Cash O...
Russian Call Girl Brookfield - 7001305949 Escorts Service 50% Off with Cash O...Russian Call Girl Brookfield - 7001305949 Escorts Service 50% Off with Cash O...
Russian Call Girl Brookfield - 7001305949 Escorts Service 50% Off with Cash O...
 
sauth delhi call girls in Bhajanpura 🔝 9953056974 🔝 escort Service
sauth delhi call girls in Bhajanpura 🔝 9953056974 🔝 escort Servicesauth delhi call girls in Bhajanpura 🔝 9953056974 🔝 escort Service
sauth delhi call girls in Bhajanpura 🔝 9953056974 🔝 escort Service
 
Call Girls Service Noida Maya 9711199012 Independent Escort Service Noida
Call Girls Service Noida Maya 9711199012 Independent Escort Service NoidaCall Girls Service Noida Maya 9711199012 Independent Escort Service Noida
Call Girls Service Noida Maya 9711199012 Independent Escort Service Noida
 
Low Rate Call Girls Mumbai Suman 9910780858 Independent Escort Service Mumbai
Low Rate Call Girls Mumbai Suman 9910780858 Independent Escort Service MumbaiLow Rate Call Girls Mumbai Suman 9910780858 Independent Escort Service Mumbai
Low Rate Call Girls Mumbai Suman 9910780858 Independent Escort Service Mumbai
 
College Call Girls Vyasarpadi Whatsapp 7001305949 Independent Escort Service
College Call Girls Vyasarpadi Whatsapp 7001305949 Independent Escort ServiceCollege Call Girls Vyasarpadi Whatsapp 7001305949 Independent Escort Service
College Call Girls Vyasarpadi Whatsapp 7001305949 Independent Escort Service
 
VIP Call Girls Lucknow Nandini 7001305949 Independent Escort Service Lucknow
VIP Call Girls Lucknow Nandini 7001305949 Independent Escort Service LucknowVIP Call Girls Lucknow Nandini 7001305949 Independent Escort Service Lucknow
VIP Call Girls Lucknow Nandini 7001305949 Independent Escort Service Lucknow
 
Call Girls Service Nandiambakkam | 7001305949 At Low Cost Cash Payment Booking
Call Girls Service Nandiambakkam | 7001305949 At Low Cost Cash Payment BookingCall Girls Service Nandiambakkam | 7001305949 At Low Cost Cash Payment Booking
Call Girls Service Nandiambakkam | 7001305949 At Low Cost Cash Payment Booking
 
Book Call Girls in Yelahanka - For 7001305949 Cheap & Best with original Photos
Book Call Girls in Yelahanka - For 7001305949 Cheap & Best with original PhotosBook Call Girls in Yelahanka - For 7001305949 Cheap & Best with original Photos
Book Call Girls in Yelahanka - For 7001305949 Cheap & Best with original Photos
 
97111 47426 Call Girls In Delhi MUNIRKAA
97111 47426 Call Girls In Delhi MUNIRKAA97111 47426 Call Girls In Delhi MUNIRKAA
97111 47426 Call Girls In Delhi MUNIRKAA
 
Call Girls ITPL Just Call 7001305949 Top Class Call Girl Service Available
Call Girls ITPL Just Call 7001305949 Top Class Call Girl Service AvailableCall Girls ITPL Just Call 7001305949 Top Class Call Girl Service Available
Call Girls ITPL Just Call 7001305949 Top Class Call Girl Service Available
 
Call Girls Service In Shyam Nagar Whatsapp 8445551418 Independent Escort Service
Call Girls Service In Shyam Nagar Whatsapp 8445551418 Independent Escort ServiceCall Girls Service In Shyam Nagar Whatsapp 8445551418 Independent Escort Service
Call Girls Service In Shyam Nagar Whatsapp 8445551418 Independent Escort Service
 
Call Girl Lucknow Mallika 7001305949 Independent Escort Service Lucknow
Call Girl Lucknow Mallika 7001305949 Independent Escort Service LucknowCall Girl Lucknow Mallika 7001305949 Independent Escort Service Lucknow
Call Girl Lucknow Mallika 7001305949 Independent Escort Service Lucknow
 

148618503 2005-2010 obstetrics-gynecology-text-book-part-1-suhail-march-mrcog-part-1-pastpapers-recall-mrcog-total-guide

  • 1. March 2010 MRCOG part 1 exam MCQ, EMQ, Separate items to revise • large scale study was carried out to assess the benefits of a new drug which would prevent fractured neck of femur due to osteoporosois in post menopausal women. the study was carried out correctly using a treatment group and an appropriate control group. the number of women who sustained a fracture neck of femur due to osteoporosis was measured reliably in each group and result are group with fracture without fracture control n1 n3 study n2 n4 calculate the number of women who need to recieve new drug in order to prevent one new fracture of femoral neck due to postmenopausal osteoporosis for the items below n1=50,n2=25,n3=9950,n4=9975 answer 400 • Successful lactation is a maintained by oestrogen F bmaintained by progesterone F initiated by LH surge F maintained by HPL F inhibited by dopamine T • Following are required for haemostatic clot formation coversion of prothrombin to thrombin T platelet phospholipid T active conversion of plasminogen to plasmin F fibrin degradation products F antithrombin F • severe oligohydramnios with limb defects • 2.60-70% ASSOSIATED WITH CLEFT LIP/ cleft palate
  • 2. • closure of neural groove - 30 days • closure of mid gut - 14 weeks • cobalt - 60 • tecnitium- 99 • The pelvic surface of the sacrum? - gives origin to the piriform muscles - gives origin to the levator ani muscles - is broader in the male than in the female - trnasmit dorsal rami of sacral nerves - is in contact with the anal canal • Prolactin. - release is stimulated by TRH - plasma levels are raised in the first trim of preg. - release is increased by suckling. - maybe produced by decidua. - release is inhibited by metoclopromide. • The foll disorders and org are correctly paired. - opthalmia neonatorum: chlamydia trachomatis - chancroid: Haemophilus ducreyi. - sleeping sickness: Leishmania donovani. - ringworm: Trichenella spiralis. - non-specific urethritis: Toxoplasma gondii. • The heart rate typically increases in response to: - pain.
  • 3. - hypoxia. - ventilatory expiration - increase in Intracranial pressure - decrease baroreceptor activity • Antibodies. - are proteins. - are formed in the fetus before 12 weeks of Intrauterine life. - have an average molecular weight of around 10 000 daltons. - of the rhesus type are genetically transmitted. - are produced by the ribosome of plasma cells. • The following disorders have an X linked pattern of inheritance. - G6PD Deficeincy. - Kleinfelter syndrome - adrenogenital syndrome - haemophilia B - familial hypercholestroleamia. • The following are derived from the urogenital sinus: - the bladder trigone. - the ureters. - the female urethra. - greater vestibular glands. - paraurethral glands. • In the fetal CVS - the heart arises from endoderm - the heart is formed by fusion of endocardial tubes. - Cardiac pulsation is present by the 30th day after fertilization.
  • 4. - oxygenated blood is tranferred to the left atrium through the foramen ovale. - the ductus arteriosus closes during the last 4 weeks of pregnancy. • Arginine vasopressin - reduces GFR. - controls water loss in the Proximal renal tubule. - is synthesised by the post pituitary gland. - is released in response to rise in plasma osmolality. - is released in response to fall in circulating plasma volume. • Renin - is secreted by the zona glomerulosa of the adrenal coretx. - is a proteolytic enzyme. - is secreted at an increased rate if the renal perfusion pressure falls. - acts upon circulating angiotensinogen. - is released in response to an increase in extracellular fluid volume • Actinomyces israelii - is a rickettsia. - forms yellow granules in pus. - is a commensal in the mouth. - is a commensal in the vagina. - is usually resistant to penicillin. • Diseases caused by spirochaetes include. - Weil's disease. - lymphogranuloma venereum - pinta - Vincent's angina. - bilharzias • The following are inherited as autosomal recessive conditions:
  • 5. - tuberous sclerosis. - phenylketonuria. - achondroplasia - sickle cell anaemia. - Von gierke's disease. • Antibodies play an important part in the development of: - phagocytosis. - Mantoux responce. - erythroblastosis fetalis. - hyperemesis gravidarum - anaphylaxis • chemical mediators concerned in the production of an inflammatory response include: - 5-hydroxytryptamine - aldosterone. - glucocorticoids. - bradykinin - leukotreines • The parathyroid gland. - originate from the pharyngeal cleft ectoderm. - secrete parathyroid hormone via the chief( principal) cells. - secrete calcitonin via the oxyphil cells. - may become hyperplastic in the presence of intestinal malabsorption. - may develop adenomas in association with islet cell tumour of the pancreas • In the abdominal wall: - the rectus abdominis muscle is attched to the crest of the pubis. - the post border of the external oblique muscle ends in the linea semilunaris.
  • 6. - the aponeurosis of the external oblique muscle takes part in the formation of the conjoint tendon. - the inferior epigastric artery is a branch of the internal iliac artery. - the conjoint tendon blends medially with the anterior layer of the rectus sheath. • The urogenital sinus in the female gives rise to the following: - ureter - paraurethral glands - Bartholin's gland - urachus - Gartner's duct. • Concerning sex hormone: - the ovary secretes androstenedione. - The ovary secretes testosterone - The ovary secretes dihydrotestosterone. - SHBG conc. are higher in women more than men - Androgens bound to protein have high biological activity. • Features of congenital rubella include: - excretion of virus by the neonates. - hepatomegaly - excessive production of growth hormone. - cataract - deafness. • Clomifene citrate: - is an anti-androgens. - does not stimulate ovulation directly. - can produce visual disturbance. - is genereally prescribed throughtout the proliferative phase of the menstrual cycle. - in the treatment of ovulation increases the risk of multiple pregnancy. • The foll are cytotoxic alkylation agents:
  • 7. - Cyclophosphamide. - mercaptopurine - chlorambucil - fluorouracil - methotrexate. • If a distribution of results is markedly skewed to the left: - the mean is the same as the 50th centile. - the same number of values lie on either side of the median. - the mode is equal to the median. - the student's t test should be used to compare the distribution with another. - logarithmic transformation of the result will produce a distribution closer to the normal. • Corcerning the analysis of clinical trials: - the 95% confidence interval indicates the range within which 19 out of 20 values will lie. - The P value illustrates how often the result would be expected to occur by chance. - c)b The conventional level of statistical significance is set of P<0.005 - In a randomised trial, there must be equal numbers of results in each arm of the study. - A relative risk reduction of 60% is significant irrespective of the value of P. • The following substance are normally synthesized in the liver: - glucagon - vitamin A - cholesterol. - immunoglobulins - prothrombin • The pineal gland: - is situated at the anterior end of the 3rd ventricles. - is innervated by the parasymphathetic nervous system. - produces melatonin.
  • 8. - maybe calcified in the adults. - is most active during daylight. • In congenital adrenal cortical hyperplasia. - The commonest deficiency is C18 hydroxylase. - plasma cortisol concentration is raised. - urinary excretion of 17 oxysteroids is elevated. - dexamethasone will suppress the urinary excretion of 17 oxysteroids. - there is no virilising effects. • 28)The following are RNA containing virus: - coxsackie - influneza - mumps - herpes simplex. - cytomegalovirus. • Listeria monocytogenes: - is a gram negative organism. - is sensitive to ampicillin. - may cause a transplacental infection. - is sexually transmitted. - can be cultured from a high vagina swab. • actinomyces israelii. - is a fungus. - forms yellow granules in pus. - is a mouth commensal - occurs in association with IUCD. - is resistant to penicillin. • The following drugs may cause enlargement of the fetal thyroid gland: - methyldopa
  • 9. - thyroxine - carbimazole - propranolol - propylthiouracil • The following statements about anticoagulant are correct: - Heparin inhibits the action of thrombin - The action of heparin is antagonised by vitamin K - Heparin increases antithrombin III activity. - The effects of anticoagulants are decreased by metronidazole. - Warfarin is greater than 80% protein bound in plasma • The following drugs and side effects are associated: - methydopa: depression - paracetamol: thromboembolism - indomethacin: peptic ulcer - prednisolone: osteoporosis. - ritodrine: hypoglycemia • Haematopoiesis in the fetus: - results in nucleated erythrocytes early in development. - occurs in the yolk sac in the first month. - does not occur in the bone marrow until term. - is predominantly hepatic during the 4th month. - does not require folic acid. • Early blood borne dissemination is characterised feature of: - carcinoma of the endometrium. - osteosarcoma - basal cell carcinoma - carcinoma of the cervix - choriocarcinoma. • The following cells maybe phagocytic: - neutrophils. - kupffer cells
  • 10. - monocytes - Hofbauer cells. - plasma cell. • The following are premalignant conditions: - diverticular disease of the large bowel. - ulcerative colitis. - pulmonary asbestosis. - Paget's disease of the bone. - condylomata of the vulva • In the pituitary gland: - the anterior lobe is smaller than the posterior lobe. - the posterior lobe is ectodermal in origin. - the acidophil cells produces oxytocin. - the basophil cells produce growth hormone. - the blood supply is derived from the internal carotis artery. • The obturator artery: - branches from the posterior trunk of the internal iliac artery. - passes through the greater sciatic foramen. - is crossed by the ureter. - supplies the hip joint. - may be replaced by a branch of the superior epigastric artery. • In congenital adrenal hyperplasia: - the commonest cause is a deficiency of 21 hydroxylase. - the plasma cortisol conc is increased. - there may be excessive secretion of 17 alpha hydroxyprogesterone. - sodium retention is characteristic. - blood cathecholamine conc are increased. • In the fetal lung: - bronchial cartilage formation commences at 18-24 weeks of gestation. - type II alveolar cells first appear at 16-20 weeks gestation.
  • 11. - sphingomyelin is the most common phospholipid present at term - phospholipid release is increased by endogenous adrenaline. - phospholipid production is decreased by exogenous corticosteroids. • In radiotherapy - 1 gray is equivalent to 1 joule/kg. - the skin usually receives a greater dose of radiation than the underlying tissues. - the major effect of radiation energy is to damage the cytoplasm of the cell. - cells in tissues which are hypoxic are more vulnerable to radiation. - Radiation induced changes in tissues may take 6 weeks to develop. • Concerning the adrenal glands - cortex is derived from neural crest cells. - Zona fasiculata secretes aldosterone. - Cortical adenomas may cause Cushing syndrome. - Neuroblastoma arise in the medulla. - Addison's disease may result from autoimmune destruction of the cortex. • The following are autosomal recessive: - neurofibromatosis. - cystic fibrosis. - phenylketonuria - polyposis coli - sickle cell anaemia • Uterine fibroids: - are defines histologically as fibromyxomas. - arise from endometrial stroma - maybe associated with polycythamia. - predispose to endometrial hyperplasia. - are liable to sarcomatous change in about 5% of cases. • growth of the foll tumors are hormone dependent:
  • 12. - squamous cell carcinoma of the cervix. - breast adenocarcinoma. - uterine leiomyoma. - prostatic adenocarcinoma - testicular carcinoma • Surfactants: - is formed mainly in the placenta - levels in amniotic fluid diminish after 33 weeks of gestation - formation can be inferred from the lecithin-sphingomyelin ratio in amniotic fluid. - contains palmitic acid - decreases the surface tension in pulmonary alveoli. • Intracellular fluid differs from Extracellular fluids in that: - it forms the major proportion of total body water. - its volume can be measured easily. - it has a higher concentration of potassium than of sodium. - its volume is regulated primarily by the kidneys. - it has a higher phosphate concentration. • The Anal canal: - has an upper part which is innervated by the inferior hypogastric plexus. - has a lower part which is supplied by the superior rectal artery - drains lymph to the superficial inguinal nodes from its upperpart. - has its internal sphincter innervated by the infecrior rectal nerve. - has a superficial part of its external sphincter attached to the coccyx. • The right ovarian artery: - Arises from the abdominal aorta above the renal artery. - passes posterior to the 3rd (horizontal) part of the duodenum - passes post to the genitofemoral nerve. - supplies the right ureter. - anastomoses with the right uterine artery.
  • 13. • Vulva supplied by: - Internal pudendal artery. - Inferior rectal artery. - Genitofemoral artery - obturator artery - femoral artery.. • This is a question outside past papers..All r false Except internal pudendal artery.. • The vagina: - has an anterior wall longer than the post wall. - contains mucus secreting glands in its epithelium. - is related in its lower third to the bladder base. - during reproductive life has an acid pH. - is derived from mesonephric duct. • The right ureter: - is approx 50cm in length. - ia partly covered by duodenum - crosses the genitofemoral nerve. - enters the bladder anteromedially. - receives part of its blood supply from the uterine artery. • After birth: - allantois froms median umblical ligament - umbilical vein forms medial umbilical ligament. - umbilical artery forms superior vesical artery. - ductus venosus forms the ligamentum teres. - ductus arteriosus forms the arch of the aorta. • In spermiogenesis: - primary spermatocytes undergo reduction division. - primary spermatocyte gives rise to 4 spermatids. - whole process of spermatogenesis in man takes 6-7 days.
  • 14. - grossly abnormal spermatozoa may be found in fertile semen. - spermatids are haploid. • In the human male, dihydrotestosterone: - is a precursor of testosterone. - has one-tenth of the ptency of testosterone. - is responsible for involution of the Mullerain system. - is responsible of the male external genitalia. - binds to an intracellular receptor. • In Human lactation: - estrogens promote development of breast lobules. - estrogen promotes milk producing effect of prolactin on the brest, - human placental lactogen is essential for milk synthesis. - prolactin stimualtes gonadotrophin release. - oxytocin causes milk ejection, • Concerning testicular hormones: - testosterone reduces plasma LH conc. - Inhibin stimulates LH production. - Estrogen are formed in the testis. - Testosterone is converted to dihyrotestosterone by 5 alpha reductase. - Testosterone in plasma is predominantly bound to albumin. • Epidermal growth factor: - is mitogenic. - synthesis is stimulated by estradiol. - is a steroid molecule. - is found in endometrium. - binds to a receptor on the nuclear membrane. • folic acid: - deficiency causes megaloblastic bone marrow.. - is hydroxycobalamin. - is present in green vegetables.
  • 15. - is predominantly absorbed from the large intestine. - is destroyed by boiling water. • Doppler Ultrasound: - is used to monitor fetal breathing. - is used in fetal HR monitors. - can be used to measure blood velocity in the fetus. - measure proton relaxation times. - requires injection of contrast agents. • The following are structural aberration of chromosomes: - deletions. - inversions. - aneuploidy. - polyploidy. - translocation. • Messenger RNA - synthesis is dependant on RNA polymerase. - is an exact copy of sense DNA. - contains exons. - is measured by western analysis. - translationoccurs in the nucleus. • In tumors of the bones: - primary malignancy is more common than 2ndry malignancy. - osteoma rarely present in skull bones. - osteosarcoma is associated with Paget's disease of bone. - lymph node metastases are unusual. - simple bone cysts have a strong tendency to recur. • Concerning the rectus sheath. - Above the costal margin rectus abdominis lies on the costal cartilages.
  • 16. - Below the arcuate line the internal oblique splits to enclose rectus abdominis. - It contains the musculophrenic artery. - It is innervated by the ilioinguinal nerve - Pyramidalis is innervated by the subcostal nerve. • The external iliac artery. - Enters the thigh anterior to the inguinal ligament. - at its origin is crossed by the ureter. - at its origin is crossed by the ovarian vessels. - Lies medial to the external iliac vein at its distal end. - gives rise to the deep external pudendal artery. • Action of Insulin include stimulation of: - glycogenolysis by the liver. - cellular uptake of amino acids. - entry of glucose into neurons. - d)entry of glucose into adipose tissue. - cellular uptake of potassium. • Metronidazole: - is effective against Giardia lamblia. - is effective when administered per rectum - should not be administered intravenously. - is usually effective against Entamoeba histolytica. - interferes with ethanol metabolism. • Amniotic fluid: - at term is hyperosmolar compared to fetal plasma. - normally contains maternal and fetal cells. - contains a higher concentration of alpha feto protein at 16 weeks than at term. - contains bilirubin. - contains phospholipids. • Chromosomes:
  • 17. - are found in the same numb in all mammalian cells. - can be analysed more quickly from a blood sample than from an amniotic fluid sample. - can be reliably identified by their length. - the Y chromosome is larger than the X chromosome. - DNA content is doubled during the S phase of the cell cycle. • In cystic fibrosis, abnormalities are seen in the - pancreas - salivary glands. - brain - kidneys - ileum • Neutrophil polymorphs at the site of inflammation are capable of the following: - Phagocytosis. - production of oxygen free radicals. - replication - fusion to form giant cells. - antibody production • In the statistical analysis of any group of numericals - the mean is always less than the mode - standard deviation is always greater than the standard error - the median value lies at the midpoint of the range - the standard error of the mean is independent of the total number of observations - there are the same number of observations greater than and less than the median value • In a randomised double blind trial comparing a new drug with a placebo - the patient will be taking either of two drugs - patients can choose their mode of treatment - doctors prescribing treatment decide which patients take the new drug
  • 18. - a large trial is more likely to give a statistically significant result than a small trial - half of the patients will take the new drug • In a trial of oral hypoglycaemic agents 42 patients were given drug A and 38 drug B . blood glucose conc. were measured befor and after a single dose of the drug. Drug B apparently caused greater fall in the blood glucose conc. (P=0.06) - these results reach an accepted level of statistical significants - Non- parametric statistical analysis bshould be used if datd are not normally distributed - In biological terms drugs A and B have been shown to be eqully effective - 6% more patients responded to drug A than drug B - Unequal numbers in the two groups invalidate the trial • The following statistical statements are correct - In the normal distribution the value of the mode is 1.73 * that of the median - In a distribution skewed to the right, the mean lies to the left to the median - In the series 2;7;5;2;3;2;5;8, the mode is 2 - Student's test is designed to correct for skewed distributions - the Chi-squared test may be used when data are not normally distributed • pudendal nerve, nerve for perianal area, down syndrome, patau syndrome, cobalt, technitium, dna virus, HIV virus, syphillis, heptitis b and c virus, PG I2, ionic exchange assosiated with calcium and potassium, positive predictive value, negative predictive value, aspirin,mefanamic acid, insulin secretion and glucagon sec inhibited and stimulated by what substances • PUDENDAL NERVE ROOT VALUE S1,2,3 ANT PRIMARY RAMI • N SUPPLY OF PERIANAL AREA • CLOSURE OF NEURAL GROOVE ON -WK OF IUL • LENGTH OF UMBILICAL CORD
  • 19. • AUTOSOMAL RECESSIVE TAY SACH'S DISEASE • AUTOSOMAL DOMINANT ACHONDROPLASIA • ANTIINFLAMMATORY MUSCLE RELAXANT PROSTAGLANDIN • STAPH AUREUS COAGULASE POSITIVE, PIGMENTED COLONY,GRAM POSITIVE • GENTAMYCIN BACTERICIDAL,TOXIC TO 8 TH NERVE • DIURETIC ACTING ON DCT BENDROFLUAZIDE • Successful lactation is - A- maintained by oestrogen F - B-maintained by progesterone F - c -initiated by LH surge F - d- maintained by HPL F - e -inhibited by dopamine T • The following are required for haemostatic clot formation - a -coversion of prothrombin to thrombin T - b -platelet phospholipid T - c -active conversion of plasminogen to plasmin F - d -fibrin degradation products F - e -antithrombin F • the following statements about vitamins are true - vit k is water soluble - vit D is poorly absorbed in cases of obstructive jaundice - vit A is a fat soluble vit. - vitamins supply the body wd energy - vit D is bound to transport protein in the circulation. • ANSWERS - a.F- fat soluble - b.T- also in cases of pancreatic disease where there is pancreatic lipase deficiency. - c.T- - d.F- vitamins r organic dietry constituents necessary for life haelth n growth that do not function by supplying the body wd energy. - e- T- physiologically active form of vitamin D is known as calcitriol.
  • 20. - calcitriol is released into the circulation, and by binding to a carrier protein in the plasma, vitamin D binding protein (VDBP), it is transported to various target organs. • VITAMIN B12 - a fat soluble vit - b-absorption takes place throughout the small intestine - c-is essentail for the metabolism of folic acid in human body. - d-deficiency leads to macrocytic anemia - e-deficiency is common in strict vegetarian. • ANSWERS - a-F- water soluble - b-F-absorbed mainly in lower ileum aided by gastric intrinsic factor. - C-T - d-T - e-T- source is mainly from animal food stuff, vegetables alone are inadequate source. • FOLIC ACID - a-water soluble - b-requires gastric intrinsic factor - c-is necessry for nucleic acid synthesis - d-heat stable - e-involved in krebs cycle • ANSWERS - a-T - b-F - c-T-tetrahydrofolate is essentaial for both purine and pyramidine synthesis. - d-F- the normal western diet contains 500-700 microgm/day of wwch 10-100% lost in cooking. - e-F
  • 21. • Folic acid (also known as vitamin B9 or folacin) and folate (the naturally occurring form), as well as pteroyl-L-glutamic acid and pteroyl-L-glutamate, are forms of the water-soluble vitamin B9. • Folic acid is itself not biologically active, but its biological importance is due to tetrahydrofolate and other derivatives after its conversion to dihydrofolic acid in the liver. • Vitamin B9 (folic acid and folate inclusive) is essential to numerous bodily functions ranging from nucleotide biosynthesis to the remethylation of homocysteine. • The human body needs folate to synthesize DNA, repair DNA, and methylate DNA as well as to act as a cofactor in biological reactions involving folate. • It is especially important during periods of rapid cell division and growth. • Both children and adults require folic acid to produce healthy red blood cells and prevent anemia. • Leafy vegetables such as spinach, asparagus, turnip greens, romaine lettuces, dried or fresh beans and peas, fortified grain products (pasta, cereal, bread), sunflower seeds and certain other fruits (orange juice, canned pineapple juice, cantaloupe, honeydew melon, grapefruit juice, banana, raspberry, grapefruit, strawberry) and vegetables (beets, broccoli, corn, tomato juice, vegetable juice, brussels sprouts, bok choy) are rich sources of folate.
  • 22. • Liver and liver products also contain high amounts of folate, as does baker's yeast. • Some breakfast cereals (ready-to-eat and others) are fortified with 25% to 100% of the recommended dietary allowance (RDA) for folic acid. • Folic acid naturally found in food is susceptible to high heat, UV, and is soluble in water. • Folic acid is an important nutrient for women who may become pregnant, because a woman's blood levels of folate fall during pregnancy due to an increased maternal RBC synthesis in the first half of the pregnancy and fetal demands in the second half. • The first four weeks of pregnancy (when most women do not even realize they are pregnant) require folic acid for proper development of the brain, skull, and spinal cord. • Serious birth defects like neural tube defects are less likely to occur when women take 0.4 mg of folic acid daily. • Neural tube defects (NTDs) result in malformations of the spine (spina bifida), skull, and brain (anencephaly). • The risk of neural tube defects is significantly reduced when supplemental folic acid is consumed in addition to a healthy diet prior to and during the first month following conception. • Folate deficiency during pregnancy can increase the risk of preterm delivery, infant low birth weight, and fetal growth retardation.
  • 23. • Folate deficiency in the mother increases homocysteine level in the blood which may lead to spontaneous abortion and pregnancy complications such as placental abruption and preeclampsia.Folic acid appears to reduce the risk of stroke. • Folic acid supplements help relieve hot flushes in postmenopausal women. Just like in estrogen hormone replacement therapy, folic acid interacts with neurotransmitters (norepinephrine, serotonin) in the brain to reduce hot flushes. • IN SHORT: Folate deficiency may lead to glossitis, diarrhea, depression, confusion, anemia, and fetal neural tube defects and brain defects (during pregnancy). • VITAMIN C - a-is found only in animal food stuffs. - b-is rapidly destroyed by heating. - c-there are normally large stores in pancreas. - d-impared wound healing is one of the characterstic features of severe vit c defeciency. - e-exess vit c can lead to formation of oxalate stones in urinary tract. • ANSWERS - a-F. citrus foods n leafy green veges r rich in vit c while animal sources contain only traces. - b-T - c-F-eye n adrenal glands contain large quantities of vit c. - d-T - e-T • Vitamin C or L-ascorbic acid is an essential nutrient for humans, in which it functions as a vitamin.
  • 24. • Ascorbate (an ion of ascorbic acid) is required for a range of essential metabolic reactions in all animals and plants. • Deficiency in this vitamin causes the disease scurvy in humans. • Ascorbic acid is absorbed in the body by both active transport and simple diffusion. • Sodium Dependent Active Transport - Sodium-Ascorbate Co- Transporters (SVCTs) and Hexose transporters (GLUTs) are the two transporters required for absorption. • Although the body's maximal store of vitamin C is largely determined by the renal threshold for blood, there are many tissues which maintain vitamin C concentrations far higher than in blood. • Biological tissues that accumulate over 100 times the level in blood plasma of vitamin C are the adrenal glands, pituitary, thymus, corpus luteum, and retina. • Those with 10 to 50 times the concentration present in blood plasma include the brain, spleen, lung, testicle, lymph nodes, liver, thyroid, small intestinal mucosa, leukocytes, pancreas, kidney and salivary glands. • Scurvy is an avitaminosis resulting from lack of vitamin C, since without this vitamin, the synthesised collagen is too unstable to perform its function. • Scurvy leads to the formation of liver spots on the skin, spongy gums, and bleeding from all mucous membranes. • The spots are most abundant on the pages and legs, and a person with the ailment looks pale, feels depressed, and is partially immobilized.
  • 25. • In advanced scurvy there are open, suppurating wounds and loss of teeth and, eventually, death. • The human body can store only a certain amount of vitamin C, and so the body soon depletes itself if fresh supplies are not consumed. • In humans, vitamin C is essential to a healthy diet as well as being a highly effective antioxidant, acting to lessen oxidative stress; a substrate for ascorbate peroxidase;and an enzyme cofactor for the biosynthesis of many important biochemicals. • Vitamin C acts as an electron donor for important enzymes:[ • Vitamin C acts as an electron donor for eight different enzymes: • Three participate in collagen hydroxylation. • These reactions add hydroxyl groups to the amino acids proline or lysine in the collagen molecule via prolyl hydroxylase and lysyl hydroxylase, both requiring vitamin C as a cofactor. • Hydroxylation allows the collagen molecule to assume its triple helix structure and making vitamin C essential to the development and maintenance of scar tissue, blood vessels, and cartilage. • are necessary for synthesis of carnitine. • Carnitine is essential for the transport of fatty acids into mitochondria for ATP generation. • The remaining three have the following functions in common but do not always do this: • -dopamine beta hydroxylase participates in the biosynthesis of norepinephrine from dopamine.
  • 26. • -another enzyme adds amide groups to peptide hormones, greatly increasing their stability. • -one modulates tyrosine metabolism. • DAILY REQUIREMENTS: • 75 milligrams per day: the United Kingdom's Food Standards Agency • 45 milligrams per day: the World Health Organization • VITAMIN B. - a.B1(thiamin)deficiency leads to impaired collagen formation - b.B1 stores in the body are adequate for upto 9 months - c.B2(riboflavin) concentration is higher in fetus than in mother - d.B6(pyridoxine)requirement in pregnancy is 25mg/day - e.niacin is synthesised in the body from tryptophan. • ANSWERS - a-T- - b-F- the body contains only 30mg(average adult requirement is 1-1.5) - c-T- as with other water soluble vit, vit B crosses the placenta by active mechanisms, wch results in higher concentration in fetus - d.F- the true figure is 2.5mg/day an din non pregnant adult is 2mg/day. - e-T • VITAMIN A - a.requires bile for its absorption. - b.deficiency leads to night blindness.
  • 27. - c.excess leads to xerophthalmia - d.stored in liver - e.daily dietry requirement during pregnancy is 50mg/day • ANSWERS - a.F - b.T - c.F- xerophthalmia is due to its deficiency. hypervitaminosis A is charachterised by anorexia , headache, hepatosplenomegaly,patchy loss of hair and hyperostosis. - d.T - e.F-during pregnancy the requirement is 1000 - microgm/day. • Vitamin A is a vitamin which is needed by the retina of the eye in the form of a specific metabolite, the light-absorbing molecule retinal. • This molecule is absolutely necessary for both scotopic and color vision. • Vitamin A also functions in a very different role, as an irreversibly oxidized form retinoic acid, which is an important hormone-like growth factor for epithelial and other cells. • Vitamin A can be found in two principal forms in foods: • -retinol • -carotenes • Vitamin A is found naturally in many foods: • liver (beef, pork, chicken, turkey, fish)
  • 28. • carrot • broccoli leaf • sweet potato • kale • butter • spinach • pumpkin • collard greens • cantaloupe melon • egg • apricot • papaya • mango • pea • broccoli • Vitamin A plays a role in a variety of functions throughout the body, such as: • Vision • Gene transcription • Immune function • Embryonic development and reproduction • Bone metabolism • Haematopoiesis • Skin health • Antioxidant Activity • DEFICIENCY leads to • -night blindness • -xerophthalmia. • First there is dryness of the conjunctiva (xerosis) as the normal lacrimal and mucus secreting epithelium is replaced by a keratinized epithelium.
  • 29. • This is followed by the build-up of keratin debris in small opaque plaques (Bitot's spots) and, eventually, erosion of the roughened corneal surface with softening and destruction of the cornea (keratomalacia) and total blindness. • Other changes include impaired immunity, hypokeratosis (white lumps at hair follicles), keratosis pilaris and squamous metaplasia of the epithelium lining the upper respiratory passages and urinary bladder to a keratinized epithelium. • With relations to dentistry, a deficiency in Vitamin A leads to enamel hypoplasia. • Adequate supply of Vitamin A is especially important for pregnant and breastfeeding women, since deficiencies cannot be compensated by postnatal supplementation. • However, excess Vitamin A, especially through vitamin supplementation, can cause birth defects and should not exceed recommended daily values. • VITAMIN E - a.present in animal food stuffs only. - b.deficiency may cause intrauterine fetal death - c.potentiates the action of coumarin anticoagulants - d.used in the treatment of infertility - e.dietry requirement is 10mg/day • ANSWERS - a.F-present in most foods - b.F-proven only in animals and not in humans - c.T
  • 30. - d.F-there is no evidence that vit e increases virility, or plays any role in treatment of infertility or recurrent abortion. - e.T • VIT D - a.water soluble - b.is stored in body fat - c.is absorbed from large intestine - d.deficiency leads to rickets - e.dietry req is 10mg/day • ANSWERS - a.F - b.T - c.F-it is absorbed from small intestine - d.T - e.T • VITAMIN K - a.mainly found in green leafy veges - b.in adults no external supplements necessary - c.hypervitaminosis is charactererised by anemia - d.exists in 2 forms k1 n k2 - e.is stored in large quantities in the liver • ANSWERS - a.T - b.T-except in pregnant pts on anti epilactics who require vit k in last months of pregnancy and in new borns. - c.T - d.T - e-Falthough accumulates in liver initially but its hepatic concentration declines rapidly. • REGARDING metabolism - a.the metabolic rate is the amount of energy liberated per unit of time
  • 31. - b.anabolism is defined as the formation of substances which can store the energy. - c.basal metabolic rate is defined as the metabolic rate determined at rest in a room at 12-14hrs after the last meal. - d.the BMR of a man is about 500kcal/day - e.the metabolic rate is decreased after consumption of a meal that is rich in protein. • ANSWERS - a.T - b.T - c.T - d.F-2000kcal/day - e.F-the metabolic rate is increased after consumption of a meal rich in protein or fat. • ANABOLISM - the set of metabolic pathways that construct molecules from smaller units. - These reactions require energy. • BASAL METABOLIC RATE • The primary organ responsible for regulating metabolism is the hypothalamus. • REGARDING METABOLISM - a.oxidation is the comination of a substance with either oxygen or hydrogen. - b.Co-factors r essential for certain enzyme reactions. - c.A co-enzyme is a protein substance wch acts as a carrier for products of reaction. - d.Co-enzyme A is a high energy compound wch is formed from adenine,ribose pentotothenic acid and thioethanol amine. - E.a calorie is defined as the amount of heat energy needed to raise the temperature of 1gm of water by 1 degree ,from 15 to 16degreesC.
  • 32. • ANSWERS • a.F-oxidation is the combination of a substance with oxygen or loss of hydrogen or loss of an electron. reduction is the reverse of this. • b.T • c.F-it is an organic non-protein substance. • d.T • e.T • ENZYMES - a.are proteins - b.heating usually results in a complete loss of enzyme activity - c.a change in pH has no effect on the activity of an ezyme. - d.are present in all cell organelles. - e.organic solvents will usually destroy an enzymes activity. • ANSWERS - a.T - b.T - c.F-a change in pH has an effect on enzyme activity. - d.T - e.T • PROTEIN METABOLISM - a.proteins contain abt 40%nitrogen - b.chains containing>100 amino acid residues are called proteins - c.proteins yield 4 calories /gm absorbed. - d.during pregnancy there is a rise in the plasma concentration of triglycerides. - e.during pregnancy there is a rise in the plasma concentration of albumin. • ANSWERS - a.F - b.T - c.F - d.T
  • 33. - e.F EMQ: • ant division of ant. remi of S2-4 • ant division of post.rami of S2-4 • genitofemoral nerve • post division of ant. remi of S2-4 • post division of post. remi of S2-4 • sensory supply of perineal nerve • inferior rectal nerve • ischial tuberosity • ischial spine • A lady is about to deliver and you are about to give her a pudendal block.. • Q1- what is the root value of pudendal nerve? • Q2- what nerve supplies lower part of vagina (I think)? • Q3- why do u give local skin infiltration before episiotomy? • Q4- or what is the nerve supply of peranal area .? • (2).. If we take the day of fertilization as Day’0’ then… • Day2 • Day4 • Day 8 • Day 10 • Day12 • Day 14 • Day 18 • Day20 • Day 22 • Day 24 • Day 26 • Day 42 [
  • 34. • Day 70 • Q1- Which represents 4 cell stage ? • Q2- conceptus implants completely? • Q3- fetal heart pulse seen on ultrasound? • Q4- vertebra form completely? • A Cytoplasmic transcription factor receptor • B G protein coupled receptor on cell membrane • C G protein coupled receptor on Golgi complex • D Mega subunit ligand gated ion channel • E Multisubunit antibody receptor on cell membrane • F Multisubunit ligand gated ion channel on cell membrane • G Nuclear protein kinase receptor • H Nuclear transcription factor receptor • I Protein kinase receptor on cell membrane • J Receptor protein complex (intracytoplasmic) • K Transcription factor receptor on cell membrane • Select the receptor which binds the molecules referred to in the itemS below. Each option may be used once, more than once or not at all. - insulin . - progesterone . - ostradiol . - prostaglandin . • (4) • PCR • Fish test • Chromosomal linkage analysis • Telomer analysis • Sum hereditary crosslinkage chromosomal option (dnt remember)
  • 35. • Screen for some known gene mutation • Sweat test • Saliva test • -A family with one kid presumed to have cystic fibrosis however cftr gene mutation was not detected. Family members willing to give samples if needed.mother wants definite diagnosis as she is 11wks and wants to know diagnosis for her baby.parents are cousins with 1st degree relatives having cystic fibrosis. • Invasive diagnostics can be applied to reach final results by? • you have Guthrie spot of their son how will u diagnose him for cystic fibrosis? • families in UK with cystic fibrosis how will u find f508? • (5) o Endoplasmic reticulum o Golgi apparatus o Lysosomes o Microtubules o Mitochondria o Nucleolus o Plasma membrane o Ribosomes • Select the most appropriate organelle that matches the following descriptions Contains enzymes capable of digesting cells and cellular material
  • 36. 'Reads' the mRNA and builds protein . Modification of lipids and proteins with storage of material prior to export out of the cell. • (6)[i][b] screening test for following syphilis treponemal antibody test syphilis hemaglutination test syphilis immobilization test HB-electrophorisis mcv mchc MCh serum iron paul-bennel test[/b][/i] • alpha thalessemia • folic acid deficiency • iron deficiency anemia • primary syphilis in early untreated period • (7) on taking a large protein meal? • Decrease decrease • Increase increase • Increase decrease • Deacrease increase • Nochange no change • Nochange increase • Nochange decrease • Insulin & Glucagon excretion what will happened ...? • (8)
  • 37. o 3-hydroxyisovaleric acid . o 17-alpha hydroxyprogesterone . o 17-delta hydroxyprogesterone . o acetic acid . o cortisol . • The following level of certain metabolities in amniotic fluid changes significantly of the fetus has an inborn error of the metabolism .select the single metabolite from list of the options above whose level in amniotic fluid is altered by inheretiance disease in item below. • congentiat adrenal hyperplasia • (9) Adrenaline Calcitonin • C Cholecystokinin Cortisol Glucagon Insulin Progesterone Somatostatin • IT- estosterone • Select the appropriate hormone from the list for the following structures that produce it. Each answer may be used once, more than once, or not at all. Adrenal Cortex . Adrenal Medulla . Pancreatic alpha cell . Pancreatic D Cells .
  • 38. • (10) • AAnterior cerebral artery • BAnterior communicating artery • CAnterior inferior cerebellar artery • DBasilar artery • EInternal auditory artery • FInternal carotid artery • GMiddle cerebral artery • HPosterior cerebral artery • IPosterior communicating artery • JSuperior cerebellar artery • KVertebral artery • Complete the diagram of the circle of Willis using the options given: • this question came with diagrame in september 2007 • so you should study circle of weils very well if I get the pictures I will post it . • Akeratinising stratified squamous epithelium • Bnon-keratinising stratified squamous epithelium • Cpseudostratified columnar epithelium • Dsimple columnar epithelium • Esimple cuboidal epithelium • Ftransitional epithelium • Which of the epithelial types described above are present in the anatomical regions below: ectocervix . endocervical canal . cervical ectropion . transformation zone of cervix .
  • 39. vagina . uterus . Bladder . Trachea . Labia Majora . Bowel . • 11-Fallopian tube . Vulva . anal canal . • Autosomal co-dominant • Autosomal dominant • Autosomal recessive • Polygenic • Single gene defect • X linked dominant • X linked recessive • [Select the most likely mode of inheritance for the following patients’ conditions: - A 27 -year old female developed gestational diabetes mellitus. Her uncle and grandmother also had diabetes mellitus2- A mothr is concerned regarding her baby who has developed fractures which appear to occur with minimal trauma. He has blue sclera. - An 18-year old female underwent caries tooth extraction and developed profuse bleeding. On history she revealed menorrhagia. Her mother and her grandfather had the same disease.
  • 40. WARFARIN . HEPARIN . AMPICILLIN . METHYLDOPA . PENCILLIN . CARBIMAZEPINE . METRONIDAZOLE . NON OF THE ABOVE . ALL OF THE ABOVE . • For each of the following choose the single most appropirate statement from the above list of options . - Drug contraindicated in breast feeding . - Drug does not cross the placenta . • A. • Calcitonin • B. • Cortisol • C. • Glucagon • D. • Growth hormone • E. • Insulin • F. • Oestradiol 17-¦Â¦Â • G.
  • 41. • Oxytocin • H. • Parathyroid hormone • I. • Prolactin • J. • Thyroxine • Instructions: For each action described below, choose the single most likely causative hormone from the above list of options. Each option may be used once, more than once, or not at all. • Question 3: Stimulates deposition of cartilage at the ends of bones • Question 4: Raises blood glucose levels through the breakdown of fat and protein • Question 5: Antagonises the effect of parathyroid hormone to minimise bone density loss • Question 6: Stimulates the release of milk from the breast • Question 7: Stimulates the urinary secretion of calcium • Options • A. • Common iliac artery • B. • External iliac artery • C. • Inferior epigastric artery • D. • Inferior vesical artery • E.
  • 42. • Internal iliac artery • F. • Middle rectal artery • G. • Ovarian artery • H. • Superior vesical artery • I. • Umbilical artery • J. • Uterine artery • K. • Vaginal artery • Instructions: After a forceps delivery a 30-year-old primigravida sustains a primary post partum haemorrhage of four litres. Although the uterus appears well contracted the bleeding continues, and a decision is made to identify and treat the bleeding point radiologically. For each question posed below, choose the single most appropriate option from the above list. Each option may be used once, more than once or not at all. • Question 8: What vessel runs up the broad ligament? • Question 9: Which other vessel arises from the uterine artery? • Question 10: Which other vessel anastomoses with the uterine artery? Question 11: Which vessel does the uterine artery arise from? • Question 12: Which vessel runs anterior and superior to the ureter? - vitamin A • B- vitamin E • C- vitamin D • vitamin B12 • vitamin B6 • vitamin C • folic acid
  • 43. • riblflavine • vitamin K • thiamine - fat soluble vitamin sysnthesised in the intestinal wall from the beta carotene . - fat soluble vitamin synthesised by large intestinal bacteria . - deficiency of this vitamin in childhood causes rickets . - fat soluble vitamin synthesis by the kidney is regulated by parathyroid hormone . - deficiency of this vitamins typicaly occurs in women with hyperemesis gravidarum . - water soluble vitamin with anti-oxidant effects . - absorption of this vitamin requires the presence of intrinsic factor . - deficiency of this vitamin causes megaloblastic anaemia and neurological disorders . - fat soluble vitamin produced in the skin by photo-activiation of 7- dehydrocholesterol . - fat soluble vitamin with anti-oxdant effects .
  • 44. - fat soluble vitamin deficiency associated with raised serum alkaline phosphatase . - deficiency of this vitamin associated with osteomalasia . - maternal intake of this vitamin around the time of coception has been shown to reduce incidance of neural tube defects . - fat soluble vitamin , deficiency occur within few a days of cessation of bile secretion . - this vitamin is a cofactor in synthesis of prothrombin . Candida spp Herpes simplex virus Human immunodeficiency virus Gardnerella vaginalis Chlamydia trachomatis Human papilloma virus Treponema pallidum Neisseria gonorrhoeae Trichomonas vaginalis • Select the micro-organism described in the following cases: - An obligate intracellular gram negative bacterium .... - The cause of lymphogranuloma venereum ......... - Neutrophils containing gram negative diplococci .....
  • 45. Double Y Syndrome Down’s Syndrome Edward’s Syndrome Fragile X Syndrome Klinefelter’s Syndrome Patau’s Syndrome Turner’s Syndrome • Please select the most appropriate syndrome from the options below. You may use each option once, more than once, or not at all. - An infant girl is born with a webbed neck, hypertension and audible ejection systolic murmur. - A neonate boy is born with cleft lip and palate, low set ears and polydactyly. The child survives for only 3 weeks. - An infant boy is born with hypotonia, epicanthic folds and single palmer creases. Adrenal arteries Celiac trunk Common iliac arteries Gonadal (ovarian or testicular) arteries Inferior mesenteric artery Lumbar arteries Median sacral artery Renal arteries Superior mesenteric artery • Select the most appropriate option: - Gives rise to the left gastric, splenic and hepatic arteries.
  • 46. - Gives rise to the intestinal, middle colic and right colic arteries. - Gives rise to the left colic, sigmoid and superior rectal arteries. - An unpaired branch that arises from the aortic bifurcation. - Arises from the aorta between the level of L1 and L2 vertebrae, and forms five segmental arteries that do not anastomose. • B- Chlamydia trachomatis • C- Herpes simplex virus • D- Human immunodeficiency virus • E- Gardnerella vaginalis • F- Human papilloma virus Neisseria gonorrhoeae Treponema pallidum Trichomonas vaginalis • Select the micro-organism described in the following cases: - A thin motile, gram negative spiral shaped bacterium ... - An anaerobic flagellated protozoan ... - A single strand RNA virus .. ACTH CRH Dopamine FSH GnRH Growth Hormone LH Prolactin Somatostatin
  • 47. TSH • Please select the correct hormone from the options for each of the descriptions below. You may use each option once, more than once, or not at all. - This hormone acts on cartilage and liver to release IGF-1... - This hormone promotes iodination of tyrosine residues... - This hypothalamic hormone inhibits the secretion of growth hormone... - In males, this hormone facilitates the generation spermatozoa... - This hormone inhibits galactorrhoea...
  • 48. • Deep inguinal Inferior mesenteric Internal iliac Para-aortic Superficial inguinal • Lymph from the following structures passes to the above group of lymph nodes first. Please choose the most appropriate answer from the list above. - Rectal . - Vulva . - Hallux . - Cervix . - Upper anal canal . Bias Correlation Error False positive False negative Null hypothesis Power Regression Sensitivity Specificity • Which of the above statistical terms is described by the following statements:
  • 49. - A random source of inaccuracy... - A type 2 error... - Ability to exclude a true negative... Clostridium perfringens clostridium difficile Escherichia coli Haemophilus influenzae Staphylococcus aureus streptococcus pyogenes none of the above • Select the most likely causative organism for the following infections. - Chronic osteomyelotis after implant surgery .. - Gas gangrene .. - Pseudomembranous colitis .. Mean Median Mid-range Mode Range Spread Standard Deviation Standard Error • Select the appropriate term from the list for the following definitions. Each answer may be used once, more than once, or not at all.
  • 50. - Most frequent value .. - Spread of estimates of sample means around the true population mean . - The measure of spread of values around the mean . - The mid value when all values are listed in ascending order . - The sum of all the values divided by the number of values . endothelial growth factor human chorionic gonadotrophin human placental lactogen IGF-1 Insulin leptin oestrogen progesterone relaxin • Select the most appropriate hormone for the following statements. - The hormone that is the basis of most pregnancy tests. - Hormone produced by synctiotrophoblast to regulate nutrient storage in the final stages of pregnancy. - Hormone that is secreted by decidual cells of the placenta .
  • 51. - The hormone that is lactogenic, regulates glucose metabolism and promotes fat breakdown . • 1) • ant division of ant. remi of S2-4 • ant division of post.rami of S2-4 • b)genitofemoral nerve • post division of ant. remi of S2-4 • post division of post. remi of S2-4 • sensory supply of perineal nerve • inferior rectal nerve • ischial tuberosity • h)ischial spine • A lady is about to deliver and you are about to give her a pudendal block.. • Q1- what is the root value of pudendal nerve? a • Q2- what nerve supplies lower part of vagina (I think)? e/a • Q3- why do u give local skin infiltration before episiotomy? e • Q4- or what is the nerve supply of peranal area .? • (2).. If we take the day of fertilization as Day’0’ then… • Day2 • Day4 • Day 8 • Day 10 • Day12 • Day 14 • Day 18 • Day20 • Day 22
  • 52. • Day 24 • Day 26 • Day 42 [ • Day 70 • Q1- Which represents 4 cell stage ? day 2 • Q2- conceptus implants completely? Day 10/12 • Q3- fetal heart pulse seen on ultrasound? Day 22 • Q4- vertebra form completely? Day 42 • PCR • Fish test • Chromosomal linkage analysis • Telomer analysis • Sum hereditary crosslinkage chromosomal option (dnt remember) • Screen for some known gene mutation • Sweat test • Saliva test • -A family with one kid presumed to have cystic fibrosis however cftr gene mutation was not detected. Family members willing to give samples if needed.mother wants definite diagnosis as she is 11wks and wants to know diagnosis for her baby.parents are cousins with 1st degree relatives having cystic fibrosis. - Invasive diagnostics can be applied to reach final results by? ??? - you have Guthrie spot of their son how will u diagnose him for cystic fibrosis? PCR??? - families in UK with cystic fibrosis how will u find f508? SWEAT TEST???? • 5) Endoplasmic reticulum Golgi apparatus Lysosomes Microtubules Mitochondria Nucleolus Plasma membrane Ribosomes
  • 53. • Select the most appropriate organelle that matches the following descriptions - Contains enzymes capable of digesting cells and cellular material c - 'Reads' the mRNA and builds protein . h - Modification of lipids and proteins with storage of material prior to export out of the cell. a • (6) screening test for following - syphilis treponemal antibody test - syphilis hemaglutination test - syphilis immobilization test - HB-electrophorisis - mcv - mchc - MCh - serum iron - paul-bennel test alpha thalessemia 4 folic acid deficiency 6 iron deficiency anemia 8 primary syphilis in early untreated period 1 • on taking a large protein meal? Decrease decrease Increase increase Increase decrease Deacrease increase Nochange no change Nochange increase Nochange decrease
  • 54. • Insulin & Glucagon excretion what will happened ...? both will increase in secretions… • (8) • 3-hydroxyisovaleric acid . • 17-alpha hydroxyprogesterone . • 17-delta hydroxyprogesterone . • acetic acid . • cortisol . • The following level of certain metabolities in amniotic fluid changes significantly of the fetus has an inborn error of the metabolism .select the single metabolite from list of the options above whose level in amniotic fluid is altered by inheretiance disease in item below. • congentiat adrenal hyperplasia b • (9) Adrenaline Calcitonin • C Cholecystokinin Cortisol Glucagon Insulin Progesterone Somatostatin • IT- estosterone • Select the appropriate hormone from the list for the following structures that produce it. Each answer may be used once, more than once, or not at all.
  • 55. o Adrenal Cortex . d o Adrenal Medulla . a o Pancreatic alpha cell . e o Pancreatic D Cells . h • Akeratinising stratified squamous epithelium • Bnon-keratinising stratified squamous epithelium • Cpseudostratified columnar epithelium • Dsimple columnar epithelium • Esimple cuboidal epithelium • Ftransitional epithelium • Which of the epithelial types described above are present in the anatomical regions below: - ectocervix . - endocervical canal . simple columnar - cervical ectropion . - transformation zone of cervix . - vagina . A - uterus . E - Bladder . F - Trachea .F - Labia Majora . - Bowel . simple columnar • 11-Fallopian tube . ciliated epi - Vulva . - anal canal . simpe columnar Autosomal co-dominant Autosomal dominant Autosomal recessive
  • 56. Polygenic Single gene defect X linked dominant X linked recessive • [Select the most likely mode of inheritance for the following patients’ conditions: - A 27 -year old female developed gestational diabetes mellitus. Her uncle and grandmother also had diabetes mellitus D - A mother is concerned regarding her baby who has developed fractures which appear to occur with minimal trauma. He has blue sclera. B - An 18-year old female underwent caries tooth extraction and developed profuse bleeding. On history she revealed menorrhagia. Her mother and her grandfather had the same disease. G • EMQ • 1) PGH2 • 2)PGG2 PGE2 PGF2alpha Arachidonic acid Calcium Magnesium • Q1) Synthesis of prostaglandins by? • Q2) the first Prostaglandin produces? • Q3) Oxytocin uses this as its intermediate precursor? • 2) Shapes of the pelvic
  • 57. • Round shaped pelvic brim with transverse diameter more than AP • Round shaped pelvic brim with AP diameter more than the transverse diameter • C)Oval shaped pelvic brim with transverse diameter more than AP • Oval shaped pelvic brim with AP diameter more than the transverse diameter • Heart shaped pelvic brim with transverse diameter more than AP • Heart shaped pelvic brim with AP diameter more than the transverse diameter • Q1) Gynecoid • Q2) Android • 3) DNA and RNA • DNA—Deoxyadenine------Double standed • Deoxythymine • Deoxycytosine • Deoxyguanine • DNA—Deoxyadenosine------Single standed • Deoxytyrosine • Deoxycytosine • Deoxyguanine • RNA—Adenine-----Double stranded • Thymine • Cytosine • Uracil • ) RNA—Adenine-----Single stranded • Thymine • Cytosine • Uracil
  • 58. • Q1) Messenger RNA? • Q2) DNA? • Q3) Human papilloma virus? Separated recalled items: • Hormone increased in Prolactinaemia • Hormone produced by posterior pituatary • Baby born with right sided umbilical hernia • Baby born with stiff limbs and mother had oligohydramniosis • Atomic mass of technetium (99) • Atomic mass of cobalt (58, but the nearest possible on the list 60) • Origins of pudendal nerve- ant s2 s4 • Functions of 21 and 17a hydroxlase (convery what to what) • POstitive predictive value and negative predictive value • Linear regression • Hep B and Hep C- DNA/ RNA viruses • Lidocaine- side effects, shorter that bupivicaine • Anatomy of pelvic bones • Fetal skull • Arterial supply of anus • Lasers- ? red light faster that UV, ? cut with red light • MRI- not radioactive, movement of H ions, resolution • Matching disease with organism- chancre, • Cancers with serum markers • Amyloidosis- ?RA ?intracellular • Bronchodilators • Staph aureus- coagulase, toxin • Time closure of neural tube, time mid gut move from abdo wall • origin of pudendal nerve • nerve supply of the skin of perinium • the structure that can differentiate between body of uterus and cervix in c/s • struture must be identified during c/s to avoid injury • embryology: by this time if the mid gut not withdrawl to abdomen the fetus will devolp exomphilus
  • 59. • by this time the an and post neroun was closed ,they give many options • obstructive jaunduce. • histopathology of endometrium, hyperplasia & adenocarcinoma. • HIV, HBV. • selective COX 2 inhibitors & antiprogesterone. • steroidogenesis pathways & enzymes. • tumour markers. • sarcoidisis. • asbestosis. • PUDENDAL NERVE ROOT VALUE S1,2,3 ANT PRIMARY RAMI • N SUPPLY OF PERIANAL AREA • LENGTH OF UMBILICAL CORD • AUTOSOMAL RECESSIVE TAY SACH'S DISEASE • AUTOSOMAL DOMINANT ACHONDROPLASIA • ANTIINFLAMMATORY MUSCLE RELAXANT PROSTAGLANDIN • STAPH AUREUS COAGULASE POSITIVE, PIGMENTED COLONY,GRAM POSITIVE • GENTAMYCIN BACTERICIDAL,TOXIC TO 8 TH NERVE DIURETIC ACTING ON DCT BENDROFLUAZIDE
  • 60. MCQ questions that actually appeared on MRCOG part 1exams between 2005-2009. 1) The pelvic surface of the sacrum? a) gives origin to the piriform muscles b) gives origin to the levator ani muscles c) is broader in the male than in the female d) trnasmit dorsal rami of sacral nerves e) is in contact with the anal canal 2)Prolactin. a) release is stimulated by TRH b) plasma levels are raised in the first trim of preg. c) release is increased by suckling. d) maybe produced by decidua. e) release is inhibited by metoclopromide. 3) The foll disorders and org are correctly paired. a) opthalmia neonatorum: chlamydia trachomatis b) chancroid: Haemophilus ducreyi. c) sleeping sickness: Leishmania donovani. d) ringworm: Trichenella spiralis. e) non-specific urethritis: Toxoplasma gondii. 4) The heart rate typically increases in response to: a) pain. b) hypoxia. c)ventilatory expiration d) increase in Intracranial pressure e) decrease baroreceptor activity 5) Antibodies. a) are proteins. b) are formed in the fetus before 12 weeks of Intrauterine life. c)have an average molecular weight of around 10 000 daltons. d) of the rhesus type are genetically transmitted. e) are produced by the ribosome of plasma cells.
  • 61. 6) the foll disorders have an X linked pattern of inheritance. a) G6PD Deficeincy. b) Kleinfelter syndrome c) adrenogenital syndrome d) haemophilia B e) familial hypercholestroleamia. 7)The foll are derived from the urogenital sinus: a) the bladder trigone. b) the ureters. c) the female urethra. d)greater vestibular glands. e) paraurethral glands. In the fetal CVS a) the heart arises from endoderm b) the heart is formed by fusion of endocardial tubes. c) Cardiac pulsation is present by the 30th day after fertilization. d) oxygenated blood is tranferred to the left atrium through the foramen ovale. e) the ductus arteriosus closes during the last 4 weeks of pregnancy. 9)Arginine vasopressin a) reduces GFR. b) controls water loss in the Proximal renal tubule. c) is synthesised by the post pituitary gland. d) is released in response to rise in plasma osmolality. e) is released in response to fall in circulating plasma volume. 10) Renin a) is secreted by the zona glomerulosa of the adrenal coretx. b) is a proteolytic enzyme. c) is secreted at an increased rate if the renal perfusion pressure falls. d) acts upon circulating angiotensinogen. e) is released in response to an increase in extracellular fluid volume
  • 62. 11) Actinomyces israelii a) is a rickettsia. b) forms yellow granules in pus. c) is a commensal in the mouth. d) is a commensal in the vagina. e) is usually resistant to penicillin. 12) Diseases caused by spirochaetes include. a) Weil's disease. b) lymphogranuloma venereum c) pinta d) Vincent's angina. e) bilharzia 13) The foll are inherited as autosomal recessive conditions: a) tuberous sclerosis. b) phenylketonuria. c) achondroplasia d) sickle cell anaemia. e) Von gierke's disease. 14) Antibodies play an important part in the development of: a) phagocytosis. b) Mantoux responce. c) erythroblastosis fetalis. d) hyperemesis gravidarum e) anaphylaxis 15) chemical mediators concerned in the production of an inflammatory response include: a) 5-hydroxytryptamine b) aldosterone.
  • 63. c) glucocorticoids. d) bradykinin e) leukotreines 16) The parathyroid gland. a) originate from the pharyngeal cleft ectoderm. b) secrete parathyroid hormone via the chief( principal) cells. c) secrete calcitonin via the oxyphil cells. d) may become hyperplastic in the presence of intestinal malabsorption. e) may develop adenomas in association with islet cell tumour of the pancreas 17) In the abdominal wall: a) the rectus abdominis muscle is attched to the crest of the pubis. b) the post border of the external oblique muscle ends in the linea semilunaris. c) the aponeurosis of the external oblique muscle takes part in the formation of the conjoint tendon. d) the inferior epigastric artery is a branch of the internal iliac artery. e) the conjoint tendon blends medially with the anterior layer of the rectus sheath. 1 The urogenital sinus in the female gives rise to the following: a) ureter b) paraurethral glands c) Bartholin's gland d) urachus e)Gartner's duct. 19) Concerning sex hormone: a) the ovary secretes androstenedione. b) The ovary secretes testosterone c) The ovary secretes dihydrotestosterone.
  • 64. d) SHBG conc. are higher in women more than men e) Androgens bound to protein have high biological activity. 20) Features of congenital rubella include: a) excretion of virus by the neonates. b) hepatomegaly c) excessive production of growth hormone. d) cataract e) deafness. 21) Clomifene citrate: a) is an anti-androgens. b) does not stimulate ovulation directly. c) can produce visual disturbance. d) is genereally prescribed throughtout the proliferative phase of the menstrual cycle. e) in the treatment of ovulation increases the risk of multiple pregnancy. 22) The foll are cytotoxic alkylation agents: a) Cyclophosphamide. b) mercaptopurine c) chlorambucil d) fluorouracil e) methotrexate. Most of the statistic questions posted are right. so will post only the ones which are missing from wht i remember. 23) If a distribution of results is markedly skewed to the left: a) the mean is the same as the 50th centile. b) the same number of values lie on either side of the median. c) the mode is equal to the median. d) the student's t test should be used to compare the distribution with
  • 65. another. e) logarithmic transformation of the result will produce a distribution closer to the normal. 24) Corcerning the analysis of clinical trials: a) the 95% confidence interval indicates the range within which 19 out of 20 values will lie. b) The P value illustrates how often the result would be expected to occur by chance. c)b The conventional level of statistical significance is set of P<0.005 d) In a randomised trial, there must be equal numbers of results in each arm of the study. e) A relative risk reduction of 60% is significant irrespective of the value of P. 25) The following substance are normally synthesized in the liver: a) glucagon b) vitamin A c) cholesterol. d) immunoglobulins e) prothrombin 26) The pineal gland: a) is situated at the anterior end of the 3rd ventricles. b) is innervated by the parasymphathetic nervous system. c)produces melatonin. d)maybe calcified in the adults. e) is most active during daylight. 27)In congenital adrenal cortical hyperplasia. a) The commonest deficiency is C18 hydroxylase. b) plasma cortisol concentration is raised. c) urinary excretion of 17 oxysteroids is elevated.
  • 66. d) dexamethasone will suppress the urinary excretion of 17 oxysteroids. e) there is no virilising effects. 28)Foll are RNA containing virus: a) coxsackie b) influneza c) mumps d) herpes simplex. e) cytomegalovirus. 29) Listeria monocytogenes: a) is a gram negative organism. b) is sensitive to ampicillin. c) may cause a transplacental infection. d) is sexually transmitted. e) can be cultured from a high vagina swab. 30) actinomyces israelii. a)is a fungus. b) forms yellow granules in pus. c)is a mouth commensal d) occurs in association with IUCD. e) is resistant to penicillin. 31) The following drugs may cause enlargement of the fetal thyroid gland: a)methyldopa b) thyroxine c) carbimazole d) propranolol e) propylthiouracil 32) The foll statements about anticoagulant are correct:
  • 67. a)Heparin inhibits the action of thrombin b) The action of heparin is antagonised by vitamin K c) Heparin increases antithrombin III activity. d) The effects of anticoagulants are decreased by metronidazole. e) Warfarin is greater than 80% protein bound in plasma 33) The following drugs and side effects are associated: a) methydopa: depression b) paracetamol: thromboembolism c) indomethacin: peptic ulcer d) prednisolone: osteoporosis. e) ritodrine: hypoglycemia 34) Haematopoiesis in the fetus: a) results in nucleated erythrocytes early in development. b) occurs in the yolk sac in the first month. c) does not occur in the bone marrow until term. d) is predominantly hepatic during the 4th month. e) does not require folic acid. 35)Early blood borne dissemination is characterised feature of: a) carcinoma of the endometrium. b) osteosarcoma c) basal cell carcinoma d) carcinoma of the cervix e) choriocarcinoma. 36) The following cells maybe phagocytic: a) neutrophils. b) kupffer cells c) monocytes d) Hofbauer cells.
  • 68. e) plasma cell. 37) The following are premalignant conditions: a) diverticular disease of the large bowel. b) ulcerative colitis. c) pulmonary asbestosis. d) Paget's disease of the bone. e) condylomata of the vulva 3 In the pituitary gland: a) the anterior lobe is smaller than the posterior lobe. b) the posterior lobe is ectodermal in origin. c) the acidophil cells produces oxytocin. d) the basophil cells produce growth hormone. e) the blood supply is derived from the internal carotis artery. 39)The obturator artery: a)branches from the posterior trunk of the internal iliac artery. b) passes through the greater sciatic foramen. c) is crossed by the ureter. d)supplies the hip joint. e) may be replaced by a branch of the superior epigastric artery. 40) In congenital adrenal hyperplasia: a) the commonest cause is a deficiency of 21 hydroxylase. b) the plasma cortisol conc is increased. c) there may be excessive secretion of 17 alpha hydroxyprogesterone. d) sodium retention is characteristic. e)blood cathecholamine conc are increased. 41) In the fetal lung: a)bronchial cartilage formation commences at 18-24 weeks of gestation.
  • 69. b) type II alveolar cells first appear at 16-20 weeks gestation. c) sphingomyelin is the most common phospholipid present at term d) phospholipid release is increased by endogenous adrenaline. e) phospholipid production is decreased by exogenous corticosteroids. 42) In radiotherapy a) 1 gray is equivalent to 1 joule/kg. b) the skin usually receives a greater dose of radiation than the underlying tissues. c) the major effect of radiation energy is to damage the cytoplasm of the cell. d)cells in tissues which are hypoxic are more vulnerable to radiation. e)Radiation induced changes in tissues may take 6 weeks to develop. 43) Concerning the adrenal glands a) cortex is derived from neural crest cells. b) Zona fasiculata secretes aldosterone. c) Cortical adenomas may cause Cushing syndrome. d) Neuroblastoma arise in the medulla. e) Addison's disease may result from autoimmune destruction of the cortex. 44)The following are autosomal recessive: a) neurofibromatosis. b) cystic fibrosis. c) phenylketonuria d) polyposis coli e) sickle cell anaemia 45) Uterine fibroids: a) are defines histologically as fibromyxomas. b) arise from endometrial stroma c) maybe associated with polycythamia.
  • 70. d) predispose to endometrial hyperplasia. e) are liable to sarcomatous change in about 5% of cases. 46) growth of the foll tumors are hormone dependent: a) squamous cell carcinoma of the cervix. b) breast adenocarcinoma. c) uterine leiomyoma. d) prostatic adenocarcinoma e) testicular carcinoma 47)Surfactants: a) is formed mainly in the placenta b) levels in amniotic fluid diminish after 33 weeks of gestation c) formation can be inferred from the lecithin-sphingomyelin ratio in amniotic fluid. d) contains palmitic acid e) decreases the surface tension in pulmonary alveoli. 4 Intracellular fluid differs from Extracellular fluids in that: a) it forms the major proportion of total body water. b) its volume can be measured easily. c) it has a higher concentration of potassium than of sodium. d) its volume is regulated primarily by the kidneys. e) it has a higher phosphate concentration. 49) The Anal canal: a) has an upper part which is innervated by the inferior hypogastric plexus. b) has a lower part which is supplied by the superior rectal artery c) drains lymph to the superficial inguinal nodes from its upperpart. d) has its internal sphincter innervated by the infecrior rectal nerve. e) has a superficial part of its external sphincter attached to the coccyx.
  • 71. 50) The right ovarian artery: a) Arises from the abdominal aorta above the renal artery. b) passes posterior to the 3rd (horizontal) part of the duodenum c) passes post to the genitofemoral nerve. d) supplies the right ureter. e) anastomoses with the right uterine artery. 51) Vulva supplied by: a) Internal pudendal artery. b) Inferior rectal artery. c) Genitofemoral artery d) obturator artery e) femoral artery.. This is a question outside past papers..All r false Except internal pudendal artery.. 52) The vagina: a) has an anterior wall longer than the post wall. b) contains mucus secreting glands in its epithelium. c) is related in its lower third to the bladder base. d) during reproductive life has an acid pH. e) is derived from mesonephric duct. 53) The right ureter: a) is approx 50cm in length. b) ia partly covered by duodenum c) crosses the genitofemoral nerve. d) enters the bladder anteromedially. e) receives part of its blood supply from the uterine artery. 54) After birth:
  • 72. a) allantois froms median umblical ligament b) umbilical vein forms medial umbilical ligament. c) umbilical artery forms superior vesical artery. d) ductus venosus forms the ligamentum teres. e) ductus arteriosus forms the arch of the aorta. 55)In spermiogenesis: a) primary spermatocytes undergo reduction division. b) primary spermatocyte gives rise to 4 spermatids. c) whole process of spermatogenesis in man takes 6-7 days. d) grossly abnormal spermatozoa may be found in fertile semen. e) spermatids are haploid. 56) In the human male, dihydrotestosterone: a) is a precursor of testosterone. b) has one-tenth of the ptency of testosterone. c) is responsible for involution of the Mullerain system. d) is responsible of the male external genitalia. e) binds to an intracellular receptor. 57) In Human lactation: a) estrogens promote development of breast lobules. b) estrogen promotes milk producing effect of prolactin on the brest, c) human placental lactogen is essential for milk synthesis. d) prolactin stimualtes gonadotrophin release. e) oxytocin causes milk ejection, 58)Concerning testicular hormones: a) testosterone reduces plasma LH conc. b) Inhibin stimulates LH production. c) Estrogen are formed in the testis. d) Testosterone is converted to dihyrotestosterone by 5 alpha reductase. e) Testosterone in plasma is predominantly bound to albumin.
  • 73. 59) Epidermal growth factor: a) is mitogenic. b) synthesis is stimulated by estradiol. c) is a steroid molecule. d) is found in endometrium. e) binds to a receptor on the nuclear membrane. 60) folic acid: a) deficiency causes megaloblastic bone marrow.. b) is hydroxycobalamin. c) is present in green vegetables. d) is predominantly absorbed from the large intestine. e) is destroyed by boiling water. 61) Doppler Ultrasound: a) is used to monitor fetal breathing. b) is used in fetal HR monitors. c) can be used to measure blood velocity in the fetus. d) measure proton relaxation times. e) requires injection of contrast agents. 62) The following are structural abberation of chromosomes: a) deletions. b) inversions. c) aneuploidy. d) polyploidy. e) translocation. 63) Messenger RNA a) synthesis is dependant on RNA polymerase. b) is an exact copy of sense DNA.
  • 74. c) contains exons. d) is measured by western analysis. e) translationoccurs in the nucleus. 64) In tumors of the bones: a) primary malignancy is more common than 2ndry malignancy. b) osteoma rarely present in skull bones. c) osteosarcoma is associated with Paget's disease of bone. d) lymph node metastases are unusual. e) simple bone cysts have a strong tendency to recur. Questions about • endometrial hyperplasia • side effect of drug (carboplatin)-(taxal) • arias stella if its specific in pregnancy • chlamydia • varicella • hpv ,wart • hcg concentration in pregnancy • citric acid cycle diagram • hormones diagram levels of, fsh,lh,estrogen,testosterone • by transvaginal u/s yolk sac appear in which week • genetic about pcr, fish test • laser • totipotent cell,tumour like mass • voiding presure • ca requirement in pregnancy • test used to diagnose folic acid deficency • thalasthmia • cervical ectropion cells are columnar cell • mri contraindicated in early pregnancy • stellate instability in dna • transverse abdominus ms attached to lumbar transverse process • cut s1 lead to autonomic bladder
  • 75. • cancer which are hormone dependent -breast -prostate -thyroid • guthre test • cystic fibrosis and diagnosis -swet test -karyotyping-fish test • disinfection and sterilization • complication of contraceptive pills • innervation of all pelvic organs. • lining of ureter • erythropoeitin and renin q from john duthie • complications of cisplatin paclitaxel • statistics -std error calculation mean in a normal distribution • occipito frontal diameter • hcg titre at detection by tv u/s 10 at 3 days after fertilization • crl in scan is 60mm what is gestational age ,crl at term ? • parietal suture is between • kuffer cells are –phagocytic • estradiol receptors • progestrone receptors -we know that these are intra nuclear but there were confusing options with intranuclear , one option had intranuclear kinase . • anticonvulsant with description of phenytoin • vitamin deficiency causing hyperemesis • macrocytic anemia • xerophtalmia • cystic fibrosis • complement causes • b thalasemia detection • folate def detection • basal cell ca - local malignant i think • tumor like –hamaroma • cystic fibrosis guthic spot test • receptors in apoptosis caspa • arias stella reaction
  • 76. • blood picture in PET and in preg is inflam like • in statistics mostly repeated questions..example given..calculate sensitivity,specificity,+ve predictive value..in right sided skewed distribution curve median will be on which side of mean? • microsatellites alleles - cd4 -nk cells • down syndrome • pulmonary embolism • apoptosis or endometrial receptors. • anal canal • vagina • ovary • Phenylketonuria • inhibin • passive transport • erythropoietin • renin • hcg levels • syphylis • rubella • hormone dependent malignancies(testicular carc._sertoli leydig- clear cellcarc.ofkidney-thyroid carc.) • pigd • congenital heart • ring Y chromosome • passive diffussion not depend on . • concentration gradient • molecular size • following tissue are capable of regeneration spinal cord liver
  • 77. epidermis myocardium bone marrow • double blind trial • level of HCG in the urine pregnancy test • APOPTOSIS • methods of disinfection and • BOWIE DICK test • progesterone receptors • syphilis , toxoplasma , hpv , immune responses • down syndrome associated with duodenal atresia true associated with ambig genetalia false only maternal chromosome F only paternal chromosome F • oogonia • miosis starts at puberty • miosis before mitosis • during S phase chromosomes are doubled. • to calculate sensitivity and specificity. • standard deviation • standard error of the mean. • mean, mode median in normal distribution and skewed data. • T test • MRI... type of radiation, causes effect in pregnancy women should be placed left tilted during scan. • dose of radiation . 1 gray = joules/ kg . absorbed dose. • dopplers USG •
  • 78. EMQ questions that actually appeared on MRCOG part 1exams between 2007-2009. EMQ about • vitamins (hyperemesis), vit B6 and (xerophthalmia). • paclitaxel carboplatin side effects • complex partial seiizure gum hypertrophy,acne ,facial coarsening,vit K TO BE given..?drug used • oestrogen progesterone receptor site • vitamins-xerophthalmia..morning sickness..macrocytic anaemia. • rubella incubation period,specific immunity within 15 days.. • HEPATOCELLULAR carcinoma,..?? • lining epithelium of ureter,..? • mantoux rection.. • ovulatory DUB • aneurysm- 10% due 2 inflam.,syphilitic aneu,thoracic vessels,marfan recessive. • if a mother has a child with cystic fibrosis ,she is pregnant and will do amniocentesis a. which test to be done (every possibility mentioned) b. all relatives will volunteer to do gutic spot test ,which test you will carry on the blood . • crown heel lenghth at birth crown/rump length 6 cm =? Weeks • type of hpv causing benign wart type of hpv causing cancer cx
  • 79. • .B cell .plasma cell .dendritic cell .mast cells . t cell QUESTION . ANTIBODY PRODUCTION? . ANTIGEN PRESENTATION? • Options 1. oval inlet transversal diameter longer than anterposterior . 2. oval inlet anterposterior dia longer than trans. 3. heart shape inlet anterposterior dia. longer than trans. 4. heart shape inlet trans. diam. longer than anterposterior 5. rounded inlet .. T>A. 6. rounded inlet ..A > T Qus. a. Gynaecoid pelvis. b. Android pelvis . ------------------------------------------------------------ a. Adenine monophosphate Guanine monophosphate Cytosine monophosphate duple helix Thymine monophosphate b. Adenine monophosphate Guanine monophosphate Cytosine monophosphate duple helix Uracil monophosphate c. Adenine monophosphate Guanine monophosphate Cytosine monophosphate single helix
  • 80. Uracil monophosphate d. Adenine monophosphate Guanine monophosphate Cytosine monophosphate single helix Thymine monophosphate e. Adenine monophosphate Guanine monophosphate Cytosine monophosphate duple helix Thymine monophosphate f. Diadenine monophosphate Diguanine monophosphate Dicytosine monophosphate duple helix Diuracil monophosphate c. DiAdenine monophosphate DIGuanine monophosphate DICytosine monophosphate single helix DIUracil monophosphate d. DIAdenine monophosphate DIGuanine monophosphate DICytosine monophosphate single helix DIThymine monophosphate e. Adenine monophosphate Guanine monophosphate Cytosine monophosphate duple helix Thymine monophosphate b. Adenine monophosphate Guanine monophosphate Cytosine monophosphate duple helix Uracil monophosphate
  • 81. c. Adenine monophosphate Guanine monophosphate Cytosine monophosphate single helix Uracil monophosphate d. Adenine monophosphate Guanine monophosphate Cytosine monophosphate single helix Thymine monophosphate 1. DNA. 2. mRNA 3. Nucliac acid o HPV. • Options Somatotroph, acidophil Somatotroph, basophil lactotroph, acidophil lactotroph, basophil corticotroph, acidophil corticotroph, basophil gonadotroph, acidophil gonadotroph, basophil thyrotroph, acidophil thyrotroph, basophil Q1. Growth hormone Q2. Prolactin number 1: a. pelvic brim oval in shape, transverse diameter more than anteropost diameter. b. pelvic brim oval in shape, anteropost diameter more than transverse diameter. c. pelvic brim heart shaped, transverse diameter more than anteropost
  • 82. diameter. d. pelvic briim heart shaped, anteropost diameter more than transverse diameter. e. pelvic brim circular, transverse diameter more than anteropost diameter. f. pelvic brim circular, anteroposr diameter more than transverse. 1) gynecoid pelvis. 2) android pelvis. number 2: a.double stranded, adenosine monophosphate, guanine monophosphate, cytosine monophosphate, uracil monophosphate. b. double stranded, adenosine monoph, guanine monph, cytosine monoph, thymine monoph. c. single stranded, adenine monoph, guanine monoph, cytosine monoph, thymine monoph. d. single stranded, adenine monoph, guanine monoph, cytosine monoph, uracil monoph. e. single stranded, deoxyadenine monoph, deoxyguanine monoph, deoxycytosine monoph, deoxyuracil monoph. f. single stranded, deoxyadenine monoph, deoxyguanine monoph, deoxycytosine monoph, deoxythymine monoph. g. double stranded, deoxyadenine monoph, deoxyguanine monoph, deoxycytosine monoph, deoxyuracil monoph h. double stranded, deoxyadenine monoph, deoxyguanine monoph, deoxycytosine monoph, deoxythymine monoph, 1) DNA. 2) RNA. 3) genome of HPV. number 3: a. edwards syndrome. b. exomphalus.
  • 83. c. gastroschiasis. d. patau's syndrome. e. down's syndrome. f. kleinfeltr's syndrome. 1) abnormality in the anterior abdominal wall, usually to the right of the umbilicus, other genetic abnormalities rarely associated. 2) abnormality in chromosome 18. number 4: a. azithromycin. b. amoxycillin. c. cefuroxime. d. benzylpenicillin. e. metronidazole. 1) best treatment of acute upper UTI in third trimester. 2)treatment of chlamydia in a non-pregnant woman. number 5: a. L1,2,3 b. L1 c. L2,3,4 d. L1,2 e. L3,4,5 1) ilioinguinal nerve. 2) genitofemoral nerve. number 6: a. streptococus, aerobic, G+ve. b. streptococus, anaerobic, G+ve. c. staphylococus, aerobic, G+ve. d. staphlococus, anaeobic, G-ve.
  • 84. e. pseudomonas, aerobic, G+ve. f. pseudomonas, aerobic G-ve. g.pseudomonas, anaerobic, G-ve. 1) toxic shock syndrome. 2) hospital cross infection. number 7: a. CMV. b. listeria monocytogens. c. treponame pallidum. d. HIV. e. human leucocytic virus. f. staphlococus aureus. g. streptococus. h. varicella zoster. 1) a pregnant woman developed a "flue-like" illness with fever and general malaise, her baby was born with hepatosplenomegaly and jaundice. 2) a woman has a history of 2 previous stillbirths, is now pregnant and at 37 weeks complained of fever and rash, her son was born healthy but at age of one year he had abnormal incisors and later deafness. number 8: a. hepatitis C, DNA. b. hepatitis C, RNA. c. herpes simplex. d. HPV. e. i cant recall the other options. 1) associated with high incidence of hepatocellular carcinoma. 2) papular skin rash. number 9:
  • 85. a. estrogen. b. progesterone. c. DHEA-sulphate. d. cortisol. e. cortisone. 1) hormone maintains uterine quisence throughout pregnancy. 2) hormone produced by the placenta and fetal adrenal gland, promotes fetal lung maturation. number 10: a. allantois. b. cloaca. c. mesonephros. d. yolk sac. 1) rectum develops from the posterior portion if this structure. 2) germ cells arise from it. number 11: a. B cell b. T CD4 cell c. T CD8 cell d. dendritic dells. 1) major antigan presenting cell. 2) could not recall it. EMQ 1) 1) PGH2 2)PGG2 3) PGE2 4) PGF2alpha
  • 86. 5) Arachidonic acid 6) Calcium 7) Magnesium Q1) Synthesis of prostaglandins Q2) the first Prostaglandin produces. Q3) Oxytocin uses this as its intermediate precursor 2) Shapes of the pelvic A) Round shaped pelvic brim with transverse diameter more than AP B) Round shaped pelvic brim with AP diameter more than the transverse diameter C)Oval shaped pelvic brim with transverse diameter more than AP D) Oval shaped pelvic brim with AP diameter more than the transverse diameter E) Heart shaped pelvic brim with transverse diameter more than AP F) Heart shaped pelvic brim with AP diameter more than the transverse diameter Q1) Gynecoid Q2) Android 3) DNA and RNA A) DNA—Deoxyadenine------Double standed Deoxythymine Deoxycytosine Deoxyguanine B) DNA—Deoxyadenosine------Single standed Deoxytyrosine Deoxycytosine Deoxyguanine C) RNA—Adenine-----Double stranded Thymine
  • 87. Cytosine Uracil D) ) RNA—Adenine-----Single stranded Thymine Cytosine Uracil Q1) Messenger RNA Q2) DNA Q3) Human papilloma virus 4) 1) L1 2) L1, L2 3) L1, L2, L3 4) L1, L2, L3, L4 5) L2, L3, L4 6) L3, L4 Q1) Ilioinguinal Q2) Genitofemoral. ) Given in a single dose A) AZITHROMYCIN B) Amoxicillin C) Cephalosporin D) Vancomycin E) Gentamycin F) Benzylpenicillin Q1) A female with a chlamydial infection, and non-pregnant, which is the best medication? Q2) A female presents with acute upper UTI? 6)Infections A) Staph
  • 88. B) Strept C) Pseudomonas aeruginosa Q1) most common cause wound infection in hospital Q2) hospital acquired infection 7) Statistics Another question on calculation for specificity and sensitivity.. People preparing for March 2010, please learn the formulas well. 8-THE FOLLOWING STATEMENTS ABOUT THE ADRENAL GLAND ARE CORRECT a-they lie anterior to the diaphragm ...........True. b-the left adrenal gland lies behind the pancreas.....True. C-lymphatic drainage is to the superficial inguinal nodes.....False d-the adrenal cortex contains chromaffin cells .........False. e-the adrenal medulla is derived from mesoderm ....False the ovary a- is attached to the ant surface of the broad ligament b- lies on the genitofemoral nerve c- lies in the angle between the ureter and the external iliac vessel d- - has visceral afferent fibres from the pelvic splanchnic nerve e- has lymphatic drainage to the superficial inguinal lymph nodes concerning the uterus a- it is formed from the mesonephric duct b- it has a lymphatic drainage in part to the inguinal glands c- the uterine artery passes below the ureter d- the uterine vein communicate with the vesical plexus of veins e-pain sensation from the body of uterus is carried by the pelvic splanchnic nerve about this stem am not sure if it was like that in the exam Obturator artery a- branches from the posterior trunk of the internal iliac artery
  • 89. b- passes through the greater sciatic foramen c- is crossed by the ureter d- supplies the hip joint e- may be replaced by a branch of the superior epigastric artery The external iliac artery a-enters the thigh anterior to the inguinal ligament b- at its origin is crossed by ureter c- at its origin is crossed by ovarian vessels d-lies medial to the external iliac vein at its distal end e-give rise to the deep external pudendal artery . The pelvic splanchnic nerves.. A- Are derived from the posterior rami of the sacral spinal nerves B- Supply afferent fibres C- Unite with branches of the synpathetic pelvic plexus D- Supply the ascending colon with motor fibres E- Supply the uterus with parasympathetic fibres In the anterior abdominal wall... A- Rectal muscle is intersected tranversely by three bands B- The posterior rectus sheath below the arcuate line consists of transversalis fascia only C- Above the costal margin the posterior rectus sheath is deficient D- The superior epigastric artery arises from the internal thoracic artery. E- The inferior epigastric artery arises from the femoral artery The pelvic surface of the sacrum. A- Gives origin to the piriforms muscle B- Gives origin to the levator ani muscle C- Is broader in the male than in the female D- Transmits the dorsal remi of sacral nerves E- Is in contact with the anal canal.
  • 90. The rectum A- Is supplied in part by the inferior rectal artery B- Is innervated by the inferior rectal nerve C- Is lined by stratified squamous epithelium D- Has its lymphatic drainage to the superficial inguinal nodes E- Possesse a complete layer of longitudinal muscle The Spleen A- has a notched posterior border B- lies in front of the costo-diaphragmatic recess C- is in contact with the body of the pancrease D- lies under the cover of the 9th to the 11th ribs E- is innervated from the renal plexus THE VULVA a-internal pudendal nerve b-anterior cutaneous of thigh c-inferior rectal d-illioinguinal e-obturator the pineal gland a-is situated at the anterior end of the third ventricle b-is innervated by parasympathetic nervous system c-produce melatonin d-may be calcified in the adult e-is most active during daylight screening test for following 1- syphilis treponemal antibody test 2- syphilis hemaglutination test 3- syphilis immobilization test 4- HB-electrophorisis 5- mcv
  • 91. 6- mchc 7- MCh 8- serum iron 9- paul-bennel test Q1) Thalassemia Q2) Syphilis Sep. 2009 Options 1. oval inlet transversal diameter longer than anterposterior . 2. oval inlet anterposterior dia longer than trans. 3. heart shape inlet anterposterior dia. longer than trans. 4. heart shape inlet trans. diam. longer than anterposterior 5. rounded inlet .. T>A. 6. rounded inlet ..A > T Qus. a. Gynaecoid pelvis. b. Android pelvis . ------------------------------------------------------------ a. Adenine monophosphate Guanine monophosphate Cytosine monophosphate duple helix Thymine monophosphate b. Adenine monophosphate Guanine monophosphate Cytosine monophosphate duple helix Uracil monophosphate c. Adenine monophosphate Guanine monophosphate Cytosine monophosphate single helix Uracil monophosphate d. Adenine monophosphate
  • 92. Guanine monophosphate Cytosine monophosphate single helix Thymine monophosphate e. Diadenine monophosphate Diguanine monophosphate Dicytosine monophosphate duple helix Diuracil monophosphate f. Diadenine monophosphate Diguanine monophosphate Dicytosine monophosphate duple helix Dithymine monophosphate g. DiAdenine monophosphate DIGuanine monophosphate DICytosine monophosphate single helix DIUracil monophosphate h. DIAdenine monophosphate DIGuanine monophosphate DICytosine monophosphate single helix DIThymine monophosphate 1. DNA. 2. mRNA 3. Nucliac acid o HPV. STRUCTURE OF BASES AND NOT THE NUCLEOTIDE OR NUCLEOSIDES gynecoid and android pelvis and the DNA 1) a) ant division of ant. remi of S2-4
  • 93. b) ant division of post.rami of S2-4 b)genitofemoral nerve c) post division of ant. remi of S2-4 d) post division of post. remi of S2-4 e) sensory supply of perineal nerve f) inferior rectal nerve g) ischial tuberosity h)ischial spine A lady is about to deliver and you are about to give her a pudendal block.. Q1- what is the root value of pudendal nerve? Q2- what nerve supplies lower part of vagina (I think)? Q3- why do u give local skin infiltration before episiotomy? Q4- or what is the nerve supply of peranal area .? (2).. If we take the day of fertilization as Day’0’ then… Day2 Day4 Day 8 Day 10 Day12 Day 14 Day 18 Day20 Day 22 Day 24 Day 26 Day 42 [ Day 70 Q1- Which represents 4 cell stage ?
  • 94. Q2- conceptus implants completely? Q3- fetal heart pulse seen on ultrasound? Q4- vertebra form completely? (3) A Cytoplasmic transcription factor receptor B G protein coupled receptor on cell membrane C G protein coupled receptor on Golgi complex D Mega subunit ligand gated ion channel E Multisubunit antibody receptor on cell membrane F Multisubunit ligand gated ion channel on cell membrane G Nuclear protein kinase receptor H Nuclear transcription factor receptor I Protein kinase receptor on cell membrane J Receptor protein complex (intracytoplasmic) K Transcription factor receptor on cell membrane Select the receptor which binds the molecules referred to in the itemS below. Each option may be used once, more than once or not at all. 1- insulin . 2- progesterone . 3- ostradiol . 4- prostaglandin . (4) PCR Fish test Chromosomal linkage analysis Telomer analysis Sum hereditary crosslinkage chromosomal option (dnt remember) Screen for some known gene mutation Sweat test
  • 95. Saliva test -A family with one kid presumed to have cystic fibrosis however cftr gene mutation was not detected. Family members willing to give samples if needed.mother wants definite diagnosis as she is 11wks and wants to know diagnosis for her baby.parents are cousins with 1st degree relatives having cystic fibrosis. 1- Invasive diagnostics can be applied to reach final results by? 2- you have Guthrie spot of their son how will u diagnose him for cystic fibrosis? 3- families in UK with cystic fibrosis how will u find f508? (5) A- Endoplasmic reticulum B- Golgi apparatus C- Lysosomes D- Microtubules E- Mitochondria F- Nucleolus G- Plasma membrane H- Ribosomes Select the most appropriate organelle that matches the following descriptions 1- Contains enzymes capable of digesting cells and cellular material 2- 'Reads' the mRNA and builds protein . 3- Modification of lipids and proteins with storage of material prior to export out of the cell. (6)[i][b] screening test for following 1- syphilis treponemal antibody test
  • 96. 2- syphilis hemaglutination test 3- syphilis immobilization test 4- HB-electrophorisis 5- mcv 6- mchc 7- MCh 8- serum iron 9- paul-bennel test[/b][/i] A- alpha thalessemia B- folic acid deficiency C- iron deficiency anemia D- primary syphilis in early untreated period (7) on taking a large protein meal? A- Decrease decrease B- Increase increase C- Increase decrease D- Deacrease increase E- Nochange no change F- Nochange increase G- Nochange decrease Insulin & Glucagon excretion what will happened ...? (8) a- 3-hydroxyisovaleric acid . b- 17-alpha hydroxyprogesterone . c- 17-delta hydroxyprogesterone . d- acetic acid . e- cortisol . The following level of certain metabolities in amniotic fluid changes significantly of the fetus has an inborn error of the metabolism .select the single metabolite from list of the options above whose level in amniotic fluid is altered by inheretiance
  • 97. disease in item below. congentiat adrenal hyperplasia (9) A- Adrenaline B- Calcitonin C Cholecystokinin D- Cortisol E- Glucagon F- Insulin G- Progesterone H- Somatostatin IT- estosterone Select the appropriate hormone from the list for the following structures that produce it. Each answer may be used once, more than once, or not at all. 1- Adrenal Cortex . 2- Adrenal Medulla . 3- Pancreatic alpha cell . 4- Pancreatic D Cells . (10) AAnterior cerebral artery BAnterior communicating artery CAnterior inferior cerebellar artery DBasilar artery EInternal auditory artery FInternal carotid artery GMiddle cerebral artery HPosterior cerebral artery IPosterior communicating artery
  • 98. JSuperior cerebellar artery KVertebral artery Complete the diagram of the circle of Willis using the options given: this question came with diagrame in september 2007 so you should study circle of weils very well if I get the pictures I will post it . Akeratinising stratified squamous epithelium Bnon-keratinising stratified squamous epithelium Cpseudostratified columnar epithelium Dsimple columnar epithelium Esimple cuboidal epithelium Ftransitional epithelium Which of the epithelial types described above are present in the anatomical regions below: 1- ectocervix . 2- endocervical canal . 3- cervical ectropion . 4- transformation zone of cervix . 5- vagina . 6- uterus . 7- Bladder . 8- Trachea . 9- Labia Majora . 10- Bowel . 11-Fallopian tube . 12- Vulva . 13- anal canal . A- Autosomal co-dominant B- Autosomal dominant
  • 99. C- Autosomal recessive D- Polygenic E- Single gene defect F- X linked dominant G- X linked recessive [Select the most likely mode of inheritance for the following patients’ conditions: 1- A 27 -year old female developed gestational diabetes mellitus. Her uncle and grandmother also had diabetes mellitus2- A mothr is concerned regarding her baby who has developed fractures which appear to occur with minimal trauma. He has blue sclera. 3- An 18-year old female underwent caries tooth extraction and developed profuse bleeding. On history she revealed menorrhagia. Her mother and her grandfather had the same disease. A- WARFARIN . B- HEPARIN . C- AMPICILLIN . D- METHYLDOPA . E- PENCILLIN . F- CARBIMAZEPINE . G- METRONIDAZOLE . H- NON OF THE ABOVE . I- ALL OF THE ABOVE . For each of the following choose the single most appropirate statement from the above list of options . 1- Drug contraindicated in breast feeding . 2- Drug does not cross the placenta . A. Calcitonin
  • 100. B. Cortisol C. Glucagon D. Growth hormone E. Insulin F. Oestradiol 17-¦Â¦Â G. Oxytocin H. Parathyroid hormone I. Prolactin J. Thyroxine Instructions: For each action described below, choose the single most likely causative hormone from the above list of options. Each option may be used once, more than once, or not at all. Question 3: Stimulates deposition of cartilage at the ends of bones Question 4: Raises blood glucose levels through the breakdown of fat and protein Question 5: Antagonises the effect of parathyroid hormone to
  • 101. minimise bone density loss Question 6: Stimulates the release of milk from the breast Question 7: Stimulates the urinary secretion of calcium Options A. Common iliac artery B. External iliac artery C. Inferior epigastric artery D. Inferior vesical artery E. Internal iliac artery F. Middle rectal artery G. Ovarian artery H. Superior vesical artery I. Umbilical artery J. Uterine artery K. Vaginal artery Instructions: After a forceps delivery a 30-year-old primigravida sustains a primary post partum haemorrhage of four litres. Although the uterus appears well contracted the bleeding continues, and a decision is made to identify and treat the bleeding point radiologically. For each question posed below, choose the single most appropriate option from the above list. Each option may be used once, more than once or not at all.